Download as pdf or txt
Download as pdf or txt
You are on page 1of 334

Nội dung

NĂM 2009 .................................................................................................................................. 3


Đề 1 ....................................................................................................................................... 3
Đề 2 ....................................................................................................................................... 9
Đề 3 ......................................................................................................................................14
Đề 4 ......................................................................................................................................18
Đề 5 ......................................................................................................................................24
Đề 6 ......................................................................................................................................33
NĂM 2010 .................................................................................................................................42
Đề 1 ......................................................................................................................................42
Đề 2 ......................................................................................................................................49
Đề 3 ......................................................................................................................................55
Đề 4 ......................................................................................................................................62
Đề 5 ......................................................................................................................................68
NĂM 2011 .................................................................................................................................73
Đề 1: Đại cương. Vô cơ.........................................................................................................73
Đề 1: Hữu cơ.........................................................................................................................80
Đề 2: Đại cương. Vô cơ.........................................................................................................84
Đề 2: Hữu cơ.........................................................................................................................89
Đề 3: Đại cương. Vô cơ.........................................................................................................94
Đề 3: Hữu cơ.........................................................................................................................98
Đề 4: Đại cương. Vô cơ.......................................................................................................105
Đề 4: Hữu cơ.......................................................................................................................112
Đề 5: Đại cương. Vô cơ.......................................................................................................115
Đề 5: Hữu cơ.......................................................................................................................119
Đề 6: Đại cương. Vô cơ.......................................................................................................124
Đề 6: Hữu cơ.......................................................................................................................129
NĂM 2012 ...............................................................................................................................134
Đề 1: Đại cương. Vô cơ.......................................................................................................134
Đề 1: Hữu cơ.......................................................................................................................139
Đề 2: Đại cương. Vô cơ.......................................................................................................146
Đề 2: Hữu cơ.......................................................................................................................152
Đề 3: Đại cương. Vô cơ.......................................................................................................157
Đề 3: Hữu cơ.......................................................................................................................161
Đề 4: Đại cương. Vô cơ.......................................................................................................167
Đề 4: Hữu cơ.......................................................................................................................171
Đề 5: Đại cương. Vô cơ.......................................................................................................176
Đề 5: Hữu cơ.......................................................................................................................180
NĂM 2014 ...............................................................................................................................188
Đề 1: Đại cương. Vô cơ.......................................................................................................188
Đề 1: Hữu cơ.......................................................................................................................194
Đề 2: Đại cương. Vô cơ.......................................................................................................199
Đề 2: Hữu cơ.......................................................................................................................206
Đề 3: Đại cương. Vô cơ.......................................................................................................212
Đề 3: Hữu cơ.......................................................................................................................220
Đề 4: Đại cương. Vô cơ.......................................................................................................226
Đề 4: Hữu cơ.......................................................................................................................232
NĂM 2015 ...............................................................................................................................236
Đề 1: Đại cương. Vô cơ.......................................................................................................236
Phản ứng sinh ra supephotphat giúp cây cỏ tốt tươi. ..........................................................243
Đề 1: Hữu cơ.......................................................................................................................244
Đề 2: Đại cương. Vô cơ.......................................................................................................251
Đề 2: Hữu cơ.......................................................................................................................257
Đề 3: Đại cương. Vô cơ.......................................................................................................263
Đề 3: Hữu cơ.......................................................................................................................269
Đề 4: Đại cương. Vô cơ.......................................................................................................274
Đề 4: Hữu cơ.......................................................................................................................280
NĂM 2016 ...............................................................................................................................285
Đề 1: Đại cương. Vô cơ.......................................................................................................285
Đề 1: Hữu cơ.......................................................................................................................292
Đề 2: Đại cương. Vô cơ.......................................................................................................297
Đề 2: Hữu cơ.......................................................................................................................304
Đề 3: Đại cương. Vô cơ.......................................................................................................310
Đề 3: Hữu cơ.......................................................................................................................318
Đề 4: Đại cương. Vô cơ.......................................................................................................323
Đề 4: Hữu cơ.......................................................................................................................328
NĂM 2009
Đề 1

Bài 1
a)

b) Có 4 nguyên tử Ag trong một đơn vị mạng.


4M 4M 3 4 .107,8682
c) ρ= ⇒ a3 = ⇒ a = √10,5.6,022142.1023 = 409 pm
VNA ρNA
a
d) r = = 144 pm
2√2

Bài 2
Các phản ứng xảy ra:
Mg + CO2 = MgO + C
(A) (B) (C)
C + O2 = CO2
Mg + N2 = Mg3N2
(D) (E)
CO2 + 2NH3 = (NH2)2CO + H2O
Bài 3
a) pH = 2,9
b) pH ≥ 7,2
Bài 4
𝐝𝐏𝐍𝐎𝟐 𝟐 𝟏 𝟏
a) Do phản ứng là bậc 2 nên ta có: = −𝐤𝐏𝐍𝐎𝟐
hay = 𝐨 + 𝐤𝐭
𝐝𝐭 𝐏 𝐍𝐎𝟐 𝐏𝐍𝐎
𝟐
𝐨
Với 𝐏𝐍𝐎𝟐
là áp suât́ NO2 ban đầu.
𝟏
b) Ta ̉ m t = 𝐭 𝟏 thì: 𝐏𝐍𝐎 =
̣ i thờ i điê 𝐨
𝐏𝐍𝐎
𝟐 𝟐 𝟐
𝟐
𝟏
Do pha ̣ c 2 nên ta có: k =
̉ n ứ ng là bâ 𝐨 .𝐭 = 0,422 L.atm-1.ph-1
𝐏𝐍𝐎
𝟐 𝟏
𝟐
Bài 5 Do CO là phối tử trường mạnh nên sẽ đẩy các electron của Fe về trạng thái cặp
đôi. Lúc này 1 AO 3d trống sẽ tổ hợp với 1 AO 4s trống và 3 AO 4p trống thành 5 AO lai
hóa dsp3 hướng về 5 đỉnh của một lưỡng tháp tam giác.
Bài 6
a) ∆Go = ∆Ho - T∆So
∆Ho = 2∆Hos(MgCl2, l) + ∆Hos(Zr, r) - ∆Hos(ZrCl4, k) - 2∆Hos(Mg, l)
∆Hos(MgCl2, l) = ∆Hos(MgCl2, r) + ∆Honc(MgCl2, r) = -641 + 43 = -598 kJ.mol-1
∆Hos(ZrCl4, k) = ∆Hos(ZrCl4, r) + ∆Hoth(ZrCl4, r) = -980 + 106 = -874 kJ.mol-1
∆Hos(Mg, l) = ∆Hos(Mg, r) + ∆Honc(Mg, r) = 0 + 9 = 9 kJ.mol-1
∆Ho = 2.(-598) + 874 – 2.9 = -340 kJ
∆So = 2So(MgCl2, l) + ∆So(Zr, r) - ∆So(ZrCl4, k) - 2∆So(Mg, l)
So(MgCl2, l) = So(MgCl2, r) + ∆Honc/Tnc(MgCl2, r) = 89,59 + (43.103/981) = 133,42 J.K-1
mol-1
So(ZrCl4, k) = So(ZrCl4, k) + ∆Hoth/Tth(ZrCl4, k) = 181 + (106.103/604) = 356,5 J.K-1 mol-1
So(Mg, l) = So(Mg, r) + ∆Honc/Tnc(Mg, r) = 32,68 + (9.103/923) = 42,43 J.K-1 mol-1
∆So = 2(133,42) + 39,0 – 356,5 – 2(42,43) = -135,5 J.K-1
∆Go = -340 + 0,1355T (kJ)
b) ∆G = ∆Go - RTlnQp = -340 + 0,1355.1073 + 8,314.10-3.1073ln(1/0,10) = -171,07 kJ
< 0. (Phản ứng tự phát)

Bài 7
̣ a theo ti ́nh bền cu
a) Dư ̉ a anion enolat ta ̣ o thà nh (da
̣ ng bazơ).
b) Chât́ B tồn tạ iơ ̣ ng lươn
̉ da ̣ c (có vò ng thơm):
̃ g cư

̣ y momen lưỡng cư
Do vâ ̣ c cu
̉ a B > A.
c)
Bài 8
a) Tạo thành hỗn hợp raxemic do đi qua trung gian là cation vòng ba cạnh dưới ảnh hưởng
nhóm kề phenyl:

̣ o thà nh cacbocation trung gian phẳng có


b) Tạo thành hỗn hợp hai đồng phân e và a do ta
cấu trúc như hình vẽ:

́ công đến trung tâm bất đối:


c) Không thay đổi cấu hình do không có sự tân

Bài 9

a)

b)

c)
Bài 10
a) Mannan là một poly-D-mannopyranosit.

Tấn công tại C1 Tấn công tại C1, C3 Tấn công tại C1, C2

Tấn công tại C1, C6 Tấn công tại C1, C2, C6


Trung bình mỗi mắt xích có một nhánh. Có ba cấu tử khác nhau luân phiên đồng đều theo
mạch qua cầu nối C1, C6 (nhưng không phải là chắc chắn, bằng chứng cho thấy một
lượng nhỏ hợp chất 2,3,4-tri-O-metyl) tạo nhánh C2:
b)

́ trúc vò ng.


c) Đecapeptit có câu

d) Phản ứng của melamin với fomandehit là phản ứng cộng nucleophin của nhóm –NH2 vào
nhóm cacbonyl.
Với A: C4H8N6O - C3H6N6 = CH2O. Vậy A là sản phẩm phản ứng cộng 1 : 1
Với B: C9H18N6O6 - C3H6N6 = 6CH2O. Vậy B là sản phẩm phản ứng cộng 1 : 6
Đề 2

Bài 1: (3,0 điểm)


a) Theo mô hình VSEPR xác định cho các hợp chất loại này thì góc cực đại được cho rằng
là 109,5o (0,75)
b) Thực tế trị số góc được xác định là không lớn do cặp electron trên P cần có nhiều không
gian hơn nên sẽ đẩy cặp electron trong liên kết P – X làm giảm trị số góc liên kết (0,75)
c) Khi đi từ I đến F thì trị số góc liên kết giảm dần khi tăng độ âm điện của X (0,25)
Lý do được giải thích là khi tăng độ âm điện thì cặp e càng lệch về phía X do khoảng
không gian mà nguyên tử trung tâm chiếm phụ thuộc vào mật độ e ở đó. Khi mật độ e
ở nguyên tử trung tâm giảm thì cặp e tự do sẽ càng có thêm nhiều không gian nên góc
liên kết sẽ giảm đi (0,5)
d) Do P còn AO trống nên sẽ có một liên kết phụ p → d, liên kết lúc này sẽ mang một phần
tính chất liên kết đôi. Do vậy mật độ electron tập trung ở liên kết sẽ lớn hơn so với PH3
nên các liên kết trong PF3 sẽ đẩy mạnh hơn
Bài 2: (2,0 điểm)
a) Ta có biểu thức ∆G = ∆H - T∆S.
Ở 220K thì ∆G220 = ∆H - 220∆S ⇒ ∆S = (∆H - ∆G220)/220
Ở 190K thì ∆G190 = ∆H - 190∆S
Do ∆H và ∆S không phụ thuộc nhiều vào nhiệt độ nên lắp biểu thức tính ∆S vào biểu
 190  o 190
thức ∆G190 sẽ thu được biểu thức như sau:  r G190   r H 1     r G220 (0,5)
o o

 220  220
Thay số vào biểu thức này thu được các kết quả lần lượt cho monohydrat, dihydrat và
trihydrat là 57,2; 85,6 và 112,8 kJ/mol (0,5)
b) Q = p(H2O)n.p(HNO3), ∆rG = ∆rGo + RTlnQ (0,25)
Lắp các trị tính được ở câu a vào biểu thức ∆rG ta sẽ thu được các kết quả lần lượt cho
monohydrat, dihydrat và trihydrat lần lượt là: -2,0; 1,3 và 3,5 kJ/mol (0,5)
Như vậy ở điều kiện địa cực thì trihydrat sẽ bền vững nhất (0,25)
Bài 3: (2,0 điểm)
a) Với axit monoclo pH = 2,51, còn triclo thì pH = 2,02 (Đúng mỗi kết quả 0,25)
b) Giả sử ban đầu ta có 1L dung dịch axit monoclo và x L dung dịch axit triclo. Lúc này
tổng thể tích dung dịch sẽ là 1 + x L
Như vậy nồng độ của monoclo và triclo trong hỗn hợp là (0,25)
0, 01
Cmono 
x 1
0, 01x
Ctri 
x 1
Với [H3O+] = 10-2,3M thì
3
CClCH COO .102,3 2,183.103
1, 4.10  2
 CClCH COO  M (0,25)
0, 01
 CClCH COO 
2
x 1
x 1 2

CCl CHCOO .102,3 9, 756.103 x


0, 2  3
 CCl CHCOO  M (0,25)
0, 01x
 CCl CHCOO 
3
x 1
x 1 3

Áp dụng định luật bảo toàn proton ta có:


2,183.103 9, 756.103 x
+ = 10-2,3 (0,25)
x 1 x 1
Giải phương trình này thu được x = 0,596 (0,25)
Vậy tỉ lệ thể tích Vmono / Vtri = 1 / 0,596 hay 1,68 : 1 (0,25)
Bài 4: (3,0 điểm)
1. Động hóa học
a) Vđâu = ∆C/∆t
Theo đồ thị thì trong 10 phút đầu tiên ta có
0, 02 x 0, 004
  x  0, 016 M  C  0, 004M  V   4.104 mol.L1. ph 1 (0,5)
6, 65 5,3 10
b) Dựa vào đồ thị ta có thể thấy được nồng độ chất phản ứng giảm một nửa trong một
khoảng thời gian như nhau nên phản ứng này có bậc 1 (0,25)
Áp dụng các biểu thức động học bậc 1 thu được k = 0,023 ph -1
(0,25)

2. Điện hóa học


a) Bán phản ứng ở A: MnO4- + 8H+ + 5e → Mn2+ + 4H2O
Áp dụng phương trình Nernst vào phương trình này ta có:
8,314.298 0, 004.(104 )8
E A  1, 491  ln  1,108V (0,25)
5.96500 0, 01
Từ đây ta sẽ suy ra EA – EB = 0,573V ⇒ EB = 0,535 V (0,25)
Ở đây không thể suy ra được điều ngược lại vì đó sẽ dẫn đến kết quả EB = 1,679V là
không thể chấp nhận do nồng độ ion Ag+ trong trường hợp này sẽ vượt quá 1M (0,25)
Lúc này ta có
8,314.298
EB  0,800  ln C Ag 
96500
lnC(Ag+) = (0,535 – 0,800).96485 / (8,314.298) = 3,296.10-5M (0,25)
C(CrO42-) = 8.10-3M (0,25)
Ksp(Ag2CrO4) = 8,68.10-12 (0,25)
b) Cần phải cho pH của A để tính thế ở bán phản ứng A (0,25)
pH ở B phải được biết vì CrO42- chỉ tồn tại ở môi trường kiềm, cho pH để xác định trong
B không có Cr2O72-
Bài 5: (2,0 điểm)
1. Hóa học các nguyên tố
Dựa vào các dữ kiện đã cho có thể xác định được A là CuO (0,25)
Các phản ứng xảy ra (Mỗi phản ứng đúng cho 0,25)
CuO + 2HCl → CuCl2 + H2O
CuCl2 + Zn → Cu↓ + ZnCl2
2Cu + O2 → 2CuO
2. Phức chất
Xác định đúng cấu trúc của các chất được 0,5
Cấu trúc các đồng phân phức 0,5 điểm

̉ m)
Bài 6 (2.0 điê
1. Cacbocation Cl3C+ sẽ rất kém bền do sự hút e mạnh của Cl nên khó có khả năng tạo
thành do Cl có khả năng tham gia liên hợp làm bền cacbocation không cao. Mặt khác
CCl4 cũng khó có khả năng tham gia SN2 do sự án ngữ của 3 clo là quá lớn cho sự tấn
công sau. Nhưng Si thì khác với C, Si có AO 3d trống nên có thể tham gia cơ chế cộng-
tách trong đó H2O sẽ tấn công vào AO 3d để tạo thành trung gian Cl4Si- - OH2+ rồi sau
đó mất HCl. Phản ứng sẽ xảy ra tiếp tục cho đến khi toàn bộ Cl bị thay thế bằng OH
(0.75)
2. Tách được, do pKa của HCOOH lớn hơn axit benzoic nhưng bé hơn axit o-clobenzoic.
(0,5)
3. Liên kết Si – C dài hơn C – C nên cản trở không gian của nhóm t -Bu không còn quan
trọng (0,75)
Bài 7 (1,5 điểm)
1. Cấu trúc mỗi chất đúng được 0,25

2. Cấu trúc mỗi chất đúng được 0,25


̉ m)
Bài 8 (3.0 điê
1. Đúng mỗi chất được 0,25

2. Cơ chế SN2 (0,5)


Do sự cứng nhắc của hệ thống nên chất của ta không có đồng phân hình học (0,25)
3. Nitro hóa hai lần, sau đó diazo hóa, chuyển thành dibrom. Sau đó chuyển tiếp thành cơ
Mg, cho tác dụng với HCHO và oxy hóa
̉ m)
Bài 9 (1.5 điê
1. Các chất chưa biết (0,5)

2. Gly-Arg-Tyr-Leu-Ala hay Gly – Tyr – Arg – Leu - Ala đều được (0,5)
3. Fructose (0,5)
Đề 3
Bài 1
a) Khối lượng phân tử của khí B là M(B) = 1,59.29,0 = 46,1 g/mol. Từ giả thiết B có màu
nâu cho phép ta khẳng định rằng khí đó là NO2. Như vậy chất lỏng không màu A là N2O4.
Hợp chất D chính là muối nitrat của kim loại C do nhiệt phân D sinh ra một oxit.
Chúng ta có:
62,01n
MD = = 91,49 (g/mol) (với n là số oxy hóa của kim loại C trong nitrat). Từ
10,3222
đây ta có khối lượng phân tử của C sẽ là 29,48n, điều này cho kết qủa là n = 2 và
C là Co. (0,25)
Như vậy D là Co(NO3)2. Do phản ứng của oxit E với HCl cho khí G là Cl2 nên oxit
E phải là Co3O4 còn F là CoCl2.6H2O (muối hexahydrat này có màu hồng, màu này
bị mất đi khi đun nóng). Kết luận:
A = N2O4; B = NO2; C = Co; D = Co(NO3)2; E = Co3O4; F = CoCl2.6H2O; G = Cl2
(0,75)
b) Các phản ứng xảy ra: (Đúng tất cả được 0,75, đúng 4 phản ứng 0,5, đúng dưới 4 được
0,25)
N2O4 ⇌ 2NO2
Co + 2 N2O4 = Co(NO3)2 + 2NO
3Co(NO3)2 = Co3O4 + 6NO2 + O2
Co3O4 + 8HCl = 3CoCl2 + Cl2 + 4H2O
CoCl2.6H2O = CoCl2 + 6H2O
c) Phức Co2+ dễ bị oxy hóa bởi oxy không khí để tạo phức Co3+. Điều này cho phép ta kết
luận X chính là [Co(NH3)6]Cl3. Cation X có cấu trúc: (0,25)

Các phản ứng xảy ra là: (0,25)


4CoCl2 + 4NH4Cl + 20NH3.H2O + O2 = 4[Co(NH3)6]Cl3 + 22H2O
6[Co(NH3)6]Cl3 = 6CoCl2 + 6NH4Cl + N2 + 28NH3
Bài 2
a) τ1 = 370 ngà y và τ2 = 240 ngà y (0,25 cho mỗi đáp án đúng)
b) Xem hình ve:̃ (1,5. Trong đó 0,75 điểm cho lập luận chuẩn và 0,75 cho công thức)
Bài 3 v = k[ BrO3-][Br-][H+]2 (0,25 cho việc tìm ra mỗi bậc riêng phần, và 0,25 cho
biểu thức)
Bài 4
a) ΔH = -291,3 kJ (0,5 điểm cho kết quả đúng, 0,5 điểm cho bước tính toán)
b) β1,4 = 1018,92 (0,5 điểm cho đáp số, 1,5 cho lập luận)
Bài 5 0,033 % Ba2+ (Mỗi phương trình phản ứng 0,25, 0,5 cho đáp số đúng và 1 điểm
cho tính toán)
Bài 6
a) (Viết đúng 3 phản ứng được 0,5. Viết đúng 2 phản ứng được 0,25)
H2(k) + HgO(r) = H2O(l) + Hg(l)
Cực âm: H2(k) + 2OH- = 2H2O(l) + 2e
Cực dương: HgO(r) + H2O + 2e = 2OH- + Hg(l)
b) KP = 5,63.10-11 , (O2) = 3,21.10-19Kpa , ∆rHom = 90,2 kJ/mol (Đúng trị số KP được 0,25,
đúng hai trị số còn lại mỗi trị số 0,5)
c) T = 799K (0,25)
́ tương tác không gian và hiệu ứng liên hợp. Cụ thể do nhóm NO2 cồng
Bài 7 Chú ý đên
kềnh nên sẽ có tương tác van der Waals với hai nhóm NO2 còn lại nên các nhóm sẽ lệch
ra khỏi mặt phẳng liên hợp làm giảm tính axit, với nhóm CN thì không có tương tác đẩy
này do nhóm CN lai hóa sp có dạng hình học phẳng (1,5). Kết luận được tính axit của
hai chất mới không khác với thứ tự ban đầu được 0,5
Bài 8
a) 1,0

b) 0,5 điểm cho mỗi cấu tạo đúng


c) Thay hai nguyên tử hiđro linh động của etylaxetoaxetat bằng một nhánh metyl và một
nhánh etyl. Tiếp theo đecacboxyl hóa sản phẩm thu được và cuối cùng iođofom
metylxeton này. (1,0. Nếu dài quá 6 phản ứng sẽ bị trừ nửa số điểm)
Bài 9
a) 1,0 điểm

b) Xác định đúng cấu trúc được 0,25 cho mỗi cấu trúc. Giải thích đúng được 0,25

Bài 10
a) Tổng hợp đúng được 1 điểm, nếu dài quá 10 bước (tổng hai chất) thì sẽ chỉ còn 0,5

b) Phe-Ala-Leu-Gly-Val (0,5 điểm cho thứ tự đúng)


c) Cơ chế phản ứng
Công thức cấu tạo của 4 polyme là (0,25, sai một công thức không cho điểm)

Quá trình tạo thành hợp chất X (0,25)


Đề 4
Bài 1: (2,0 điểm)
A chắc chắn là một oxit kim loại do nó khó tan trong nước (phương trình 4).
(0,25)
Giả thiết A có x nguyên tử oxy và số oxy trong B là y. Nếu chúng ta đếm số nguyên tử
oxy trong phương trình 1 hay 2 đều dẫn đến hệ thức
2y = x + 5. (0,25)
Từ hai phương trình 1 và 4 thì hợp chất C không hề chứa lưu huỳnh. Bằng cách đếm số
oxy trong phương trình 3, có thể rút ra được C chứa 4 nguyên tử oxy. Như vậy với việc
sử dụng phương trình 4 chúng ta có hệ thức
(0,25)
x+y=7 (0,25)
Từ hệ phương trình này ta có kết quả x = 3 và y = 4. (0,25)
Bây giờ chúng ta đã đoán được A ở dạng M2O3, B: Na2MO4, C: Na4MO4 và D: MSO4.
(0,25)
Sử dụng các giá trị đề cho ta có thể tính được số mol NaOH trong dung dịch:
n(NaOH) = c·V = 10(pH-14)·0,1 = 0,00157 mol. Tức là
n(Na4MO4) = 0,000472 mol và M(Na4MO4) = 211.85 g/mol. M là Fe.
(0,25)
A – Fe2O3
B – Na2FeO4
C – Na4FeO4
D – FeSO4

̉ m)
Bài 2 (2,0 điê
(a) CO2 (aq) + H2O ⇌ HCO3- (aq) + H+ (aq). Ka = 10-6,1
Ta có:

Ka 
 H   .  HCO3  6,1
 10 
 CO2  107,4
 6,1  101,3  0, 05 0,50
CO2   HCO3  10

(b) Trong dung dịch đệm, nồng độ của HCO3- lớn hơn nồng độ của CO2 nhiều 0,50
nên chống lại sự thay đổi của axit tốt hơn
CO2( aq ) 
(c) (i) K H   2, 25.104  CO2( aq )   2, 25.104.5,3  1,19.103 (M )
P CO2 ( k ) 0,50

(ii) Từ
CO2   0, 05 và [CO2(aq)] = 1,19.10-3M nên [HCO3-] = 2,38.10-2 M
 HCO3  0,50
̉ m)
Bài 3 (3,0 điê
p 2 ( H 2O). p(O2 ) ( p1 ) 2 .0,5. p.1
1. 2 H2O ⇌ 2 H2 + O2 K ' p    7, 63.1021. p (0,25)
2
p ( H 2O ) p (1  1 )
2 2

(0,5 2 ) 2
2 HCl ⇌ H2 + Cl2 Kp "   3, 03.1011
(1   2 ) 2

p 2 ( H 2O). p 2 (Cl2 )
4 HCl + O2 ⇌ 2 Cl2 + 2 H2O Kp 
p(O2 ). p 4 ( HCl )
p 2 ( H 2O ) p 2 ( H 2O). p 2 (Cl2 )
Kp  2
. 4
 K p "2 / K p ' KP = 0,12.p-1 (0,25)
p(O2 ). p ( H 2 ) p ( HCl )
K = KP.p0 = 0,12 (do pchuẩn = p0) (0,25)
∆G = -RTlnK = 17,6 kJ/mol
(0,25)
2.
a) Phương trình phản ứng: BBr3 + PBr3 +3H2 → BP + 6HBr
(0,25)
b) Bậc của phản ứng là 2 (Tính đúng mỗi bậc riêng phần được
0,25)
Biểu thức tốc độ phản ứng: v = k[BBr3][PBr3] (0,25)
c) k800 = 4,60.10-8/2,25.10-8.9,00.10-6 = 2272L2.s-1.mol-1 (0,25)
k880 = 19,60.10-8/2,25.10-8.9,00.10-6 = 9679L2.s-1.mol-1
(0,25)
d) Phương trình Arrhenius có dạng: lgk = lgA – Ea/2,3RT
Ta có:
lgk1 = lgA – Ea/2,3RT1 (1)
lgk2 = lgA – Ea/2,3RT2 (2)
Trừ (1) cho (2) ta được:
E 1 1  1 1  k1
lg k1  lg k 2   a     E a  2,3R   lg (0,25)
2,3R  T1 T2   T1 T2  k2
Thay số vào ta tính được Ea = 186kJ.mol-1.
(0,25)

̉ m)
Bài 4 (2,0 điê
1. Hydro tồn tại trong các hợp chất này dưới dạng các ion hydrua. Lý do là độ âm
điện của Na và Mg đã quá thấp so với hydro
(0,50)
Phản ứng: Điện phân nóng chảy các muối này thấy thoát ra hydro ở anot hoặc
cho các muối tác dụng với nước sẽ có hydro thoát ra
(0.25)
2. x = 2: Diboran (BH3)2. Do BH3 là hợp chất 6e nên với việc tạo thành diboran nó
mới có đủ 8e để có thể tồn tại bền vững
(0,25)
y = : Polymer nhôm hydrua. Do Al có nhiều vị trí phối trí hơn (số phối trí 6
khác với bo chỉ 4) nên nó sẽ tạo 6 liên kết quả chuối (bo chỉ 2). Và như vậy
nhôm hydrua có thể tụ hợp nhiều phân tử lại với nhau tạo thành một polymer
nhôm hydrua dạng rắn chứ không như diboran ở dạng khí
(0,5)
3. Tính axit trong chu kỳ tăng dần từ trái sang phải và tính base trong nhóm VA
giảm từ trên xuống dưới
(0,5)
̉ m)
Bài 5 (3,0 điê
1. Cấu trúc các phức (Đúng mỗi cấu trúc được 0,25)

2.
a) Phản ứng: 2 Sb + 6 OH- = Sb2O3 + 3H2O + 6e (0,25)
b) pH = 12 ⇒ [OH-] = 10-2, pH = 12,7 ⇒ [OH-] = 10-1,3
RT 16 RT 102
E  Eo  ln , E 12,7 – E12,0 = ln 1,3  0, 041V (0,50)
6 F [OH  ]6 F 10
3. Ô mạng cơ sở gồm 4 anion oxy và 4 anion titan như hình vẽ
a) Khối lượng riêng của TiO (0,25)
(16  48)
 23 7 3
 5, 73 g/cm3
6, 022.10 .(0, 42.10 )
b) Xây dựng chu trình Born – Haber (0,25)
-523
Ti(r) + ½O2(k) TiO (r)
425
½. 498 Uml
Ti(k) + O (k)
658 -
141,5 1310 +
Ti+(k) + O- (k) Ti2+(k) + O2-(k)
797,5
Tính được kết quả Uml = -3821 kJ/mol (0,25)
̉ m)
Bài 6 (2,0 điê
1. Do F gây hiệu ứng –I mạnh nên sẽ làm cho các nguyên tử H ở C trở nên có tính
axit. Nguyên tử H có tính axit cao hơn sẽ là nguyên tử ít bị thế hơn. Vì vậy nên
base sẽ ưu tiên tấn công vào vị trí đó tạo sản phẩm Hofmann
(0,75)
2. Do khả năng solvat hóa anion sinh ra từ axit này sẽ yếu hơn vì độ phân nhánh
cao sẽ ngăn cản sự tiếp cận của phân tử dung môi đến anion này
(0,5)
3. Sản phẩm có cấu trúc như sau (0.25)

Phản ứng này xảy ra theo cơ chế SN2 với sự tạo thành anion xiclopentadienyl là
hệ thống thơm bền vững (6e, thỏa mãn quy tắc Huckel về tính thơm)
(0,5)

̉ m)
Bài 7 (2,0 điê
1. Xác định đúng công thức phân tử của hydrocarbon là C8H14 (0,25)
Xác định chính xác cấu tạo của hydrocarbon là bixiclo[2.2.2]octan
(0,25)
Xác định được cấu trúc hai sản phẩm monoclo (Chất B quang hoạt nên sẽ có hai
đồng phân quang học) (Mỗi cấu trúc đúng
được 0,25)

2. Khi đun nóng B trong axit tạo ra xeton spiro, điều đó có nghĩa C là sản phẩm
chuyển vị pinacol của B. Suy ra được B là diol hai vòng 6 cạnh giáp, từ đó suy ra
A là hydrocarbon dẫn xuất của decalin có nối đôi ở cạnh chung.
(0,5)
Đúng mỗi cấu trúc được 0,25 điểm

̉ m)
Bài 8 (3.0 điê
Đúng cấu trúc các chất được 0,25 điểm. C có mặt phẳng đối xứng nên không tồn
tại đồng phân quang học, còn II do cấu trúc cứng nên sẽ có 2 đồng phân. Xác định
đúng số đồng phân được 0,5 điểm (0,25 cho mỗi chất)
Đúng cơ chế được 0,5 điểm
O OEt O OEt

O O
N OEt N
N N
-OEt CO2Et CO2Et
C
EtO2C H EtO2C I

O
*
+
N
H Cl-
II X – H2N-NH2
N
1-azabicyclo[2.2.2]octane
*

Sơ đồ tổng hợp hợp lý được 1 điểm. Nếu số phản ứng vượt quá 10 chỉ còn 0,5

̉ m)
Bài 9 (2,0 điê
1. Công thức cấu tạo các chất
a) Đúng mỗi chất A và B được 0,25
b) Đúng cấu trúc được 0,5 điểm

2. L-gulose và D-glucose cho cùng một axit aldaric nên Fischer kết luận rằng với D-
glucose (nhóm CHO ở đầu) và L-gulose (nhóm CHO ở cuối) đều có chung một cấu
hình ở trung tâm bất đối. Điều đó có nghĩa là nếu biết cấu trúc L-gulose sẽ biết
được cấu trúc D-glucose, cũng như C2 epimer của glucose là mannose
(1,0 điểm)
Đề 5
̉ m)
Bài 1 (3.0 điê
1.
a) Có hai cấu trúc vàcủa NO2 như sau: (0,25 cho mỗi cấu trúc)

Ba cấu trúc tương ứng của N2O4 như sau: (0,25 cho mỗi cấu trúc)

b) Dạng tồn tại chính ở pha lỏng là dạng a + a. Liên kết N - N trong cấu trúc này kém
bền hơn trong N2H4 là do lực đẩy Coulomb giữa hai nguyên tử N cùng mang điện tích
dương. Khi chuyển từ N2O4 sang N2H4 thì độ dài liên kết N - N giảm. (0,75)
2. Mỗi loại ion tạo ra một mạng lập phương tâm mặt, hai mạng đó lồng vào nhau, khoảng
𝑎
cách giữa hai mạng là và tam giác tạo thành bởi hai cạnh góc vuông a, a còn cạnh
2
huyền là đường chéo d, khi đó:
d2 = 2a2 → d = a√2
𝑎√2
d = 4r(Cl-) → r(Cl-) = 4
= 182 pm (0,5)
Xét một cạnh a: a = 2r(Li+) + 2r(Cl-) → r(Cl-) = 75 pm (0,5)
̉ m)
Bài 2 (2.0 điê
1.
a) Do phản ứng là bậc 2 nên ta có:
dPNO2
 kPNO
2

dt 2

1 1
 o
 kt
PNO2 PNO 2

Với PNO2 tương ứng với áp suất NO2 ban đầu.


o
(0,25)
b) Tại thời điểm t = t1/2 thì:
1 o
PNO2  PNO2
2
Do phản ứng là bậc hai nên từ đó ta có:
1
k o
 0,422 L.atm 1 ph 1 (0,25)
P t NO 2 1 / 2

2. Bán phản ứng xảy ra ở điện cực hydro trong amoniac lỏng sẽ là:
H2 + 2 NH3 ⇌ 2 NH4+ + 2e
Thế tương ứng của nó là:
2
RT CNH 4 / Co
E(NH4 /H2) = E (NH4 /H2) 
+ o
ln+
với Eo(NH4+/H2) = 0V (0,25)
2F pH 2 / po
Ở đây điện cực hydro sẽ đóng vai trò là anod (vì nếu ngược lại thì lượng NH4+ sẽ vô lý)
∆E = Ecatot – Eanot ⇒ 0,820 = 0,681 – E(NH4+/H2) ⇒ E(NH4+/H2) = -0,139V
Thay vào biểu thức Nernst của cặp NH4+/H2 cho ta giá trị C(NH4+) = 9,9.10-4M (0,25)
Điều này có nghĩa là axit axetic phân ly hoàn toàn trong amoniac lỏng ( = 0,99) nên
nó là một axit mạnh (0,25)
Tính toán tương tự với HCN cho ta kết quả E(NH4+/H2) = -0,156V, C(NH4+) = 4,22.10-4M
(0,25)
Chỉ 42,2% HCN phân ly nên nó sẽ là một axit yếu trong amoniac lỏng (0,25)
Trong dung dịch nước thì axit axetic là một axit yếu (pKa = 4,75) và HCN là một axit rất
yếu (pKa = 9,25). Tuy nhiên do tính base mạnh của amoniac làm tăng tính axit của các
axit yếu này (0,25)

̉ m)
Bài 3 (1.5 điê
a) Có cân bằng: H2S + 2 H2O ⇌ 2 H3O+ + S2-
CH2 O .CS 2
K= 3
 K a1.K a 2 ⇒ C(S2-) = 1,58.10-17M (0,25)
CH 2 S
Từ đó ta có các kết quả tương ứng
MeS = 10-5.1,58.10-17 = 1,58.10-22 (0,25)
Me2S = (10-5)2.1,58.10-17 = 1,58.10-27 (0,25)
b) Với CdS thì khi hòa tan vào HCN ta có các cân bằng sau
CdS ⇌ Cd2+ + S2-
Cd2+ + 4 CN- ⇌ [Cd(CN)4]2-
Gọi C(Cd2+) = x, C(S2-) = y và C(phức) = z sẽ dẫn đến hệ thức
Ksp = x.y (1); β = z / (x.14) (2) y = x + z (3) ⇒ y = 8,37.10-6M (0,25)
Với Cu2S thì khi hòa tan vào HCN ta có các cân bằng sau
CuS ⇌ 2 Cu+ + S2-
Cu+ + 4 CN- ⇌ [Cu(CN)4]3-
Gọi C(Cu+) = x, C(S2-) = y và C(phức) = z sẽ dẫn đến hệ thức

Ksp = x2.y (1); β = z / (x.14) (2) y = ½x + ½z (3) ⇒ y = 271M


(0,25)
So sánh các giá trị trên ta thấy Cu2S tan hoàn toàn trong KCN 1M còn CdS thì
không. Vì vậy việc kết tủa phân đoạn các chất này là hoàn toàn có thể
(0,25)
̉ m)
Bài 4 (1.5 điê
c) ∆Go = ∆Ho - T∆So
∆Ho = 2∆Hos(MgCl2, l) + ∆Hos(Zr, r) - ∆Hos(ZrCl4, k) - 2∆Hos(Mg, l)
∆Hos(MgCl2, l) = ∆Hos(MgCl2, r) + ∆Honc(MgCl2, r) = -641 + 43 = -598 kJ.mol-1
∆Hos(ZrCl4, k) = ∆Hos(ZrCl4, r) + ∆Hoth(ZrCl4, r) = -980 + 106 = -874 kJ.mol-1 (0,25)
∆H s(Mg, l) = ∆H s(Mg, r) + ∆H nc(Mg, r) = 0 + 9 = 9 kJ.mol
o o o -1

∆Ho = 2.(-598) + 874 – 2.9 = -340 kJ


∆So = 2So(MgCl2, l) + ∆So(Zr, r) - ∆So(ZrCl4, k) - 2∆So(Mg, l)
So(MgCl2, l) = So(MgCl2, r) + ∆Honc/Tnc(MgCl2, r) = 89,59 + (43.103/981) = 133,42 J.K-1
mol-1 (0.25)
S (ZrCl4, k) = S (ZrCl4, k) + ∆H th/Tth(ZrCl4, k) = 181 + (106.10 /604) = 356,5 J.K-1 mol-1
o o o 3

So(Mg, l) = So(Mg, r) + ∆Honc/Tnc(Mg, r) = 32,68 + (9.103/923) = 42,43 J.K-1 mol-1


∆So = 2(133,42) + 39,0 – 356,5 – 2(42,43) = -135,5 J.K-1 (0,25)
∆G = -340 + 0,1355T (kJ)
o
(0,25)
d) ∆G = ∆G - RTlnQp = -340 + 0,1355.1073 + 8,314.10 .1073ln(1/0,10) = -171,07 kJ
o -3

< 0. (Phản ứng tự phát) (0,5)


̉ m)
Bài 5 (2.0 điê
1. Hóa vô cơ nguyên tố
a) Các halogenua axit phản ứng với nước theo phương trình:
3 y
SOxClHaly + H2O = HCl + y Hhal + H2SOx + 0,5y + 1,5
2
Ở thí nghiệm thứ nhất, kết tủa sẽ chứa bari sunfat hoặc sunfit. Ở thí nghiệm thứ hai sản
phẩm sẽ gồm AgCl và một bạc halogenua khác. Bạc halogenua đó không thể là AgF do
khối lượng kết tủa ở thí nghiệm thứ hai lớn hơn thí nghiệm thứ nhất (vì AgF tan trong
nước). Như vậy X chỉ có thể là Br và I. (0,25)
Do số mol của kết tủa thí nghiệm 1 bằng với số mol kết tủa thí nghiệm 2 nên ta có hệ
thức
mkt 2 m1 m 143  y (108  M Hal )
  1, 419  kt 2 
M AgCl  yM AgHal mBaSO2 x y3 mkt1 137  32  16( x  0,5 y  1,5)
2

Điều này dẫn đến kết quả


131  23  96,5 y
M Hal  (0,25)
y
Dựa trên sự phân tích số oxy hóa của các nguyên tố trong SOxClHaly thì ta nhận thấy x
chỉ có thể có hai giá trị 1 hay 2, còn y có thể là 1, 2 hay 3. Áp dụng các phép thử ta thu
được kết quả x = 2 và y = 1. Hợp chất của ta là SO2ClBr
Vậy phản ứng của ta là (Viết đúng hai phản ứng được 0,25)
SO2ClBr + Ba(OH)2 = BaClBr + BaSO4↓ + 2 H2O
SO2ClBr + 2 AgNO3 + 2H2O = AgCl↓ + AgBr↓ + H2SO4 + 2 HNO3
b) Sơ đồ tổng hợp (0,5 điểm – Nếu dài quá 6 phản ứng cho 0,25)
2 P + 5 Cl2 = 2 PCl5
2 P + 5 Br2 = 2 PBr5
H2SO4 + PCl5 = HSO3Cl + POCl3 + HCl
HSO3Cl + PBr5 = SO2ClBr + POBr3 + HBr
2. Phức chất
a) Tên: Cloropentaamin coban (III). Số oxy hóa: +3
Công thức phức:

(0,25)
b) [CoCl3(NH3)3]: Hai đồng phân lập thể không đối quang (0,25)

̉ m)
Bài 6 (2 điê
1. Cấu trúc vòng cứng sẽ có tính axit kém hơn do carbanion ở đầu cầu không thể được an
định bởi cộng hưởng (AO p của carbanion sẽ không thể song song để xen phủ với MO 
của nhóm CO để làm bền carbanion) (0,50)
2. Hóa lập thể
a) Ký hiệu đúng 2 trung tâm được 0,25, sai một trung tâm không có điểm
OH

* CH3
S S
*

NHCH3

b) Các công thức của pseudoephedrin (0,5)


CH3

CH3
H3CHN H
HO H
hay
H3CHN H
HO H
Ph

Ph
Công thức Fischer
Ph CH3

H OH H NHCH3
hay
H3CHN H HO H

CH3 Ph
c) Cấu trúc của methcanthion (2) (0,25)

d) Cấu trúc của sản phẩm đồng phân (0,25)


OH

CH3

NHCH3

Giải thích lập thể phản ứng (0,25)


CH3
CH3
H3C

H N H
H3CHN H
Li+ 1) H+
Li
2 2) H2O
HO Ph
O
Ph

H
-
H
̉ m)
Bài 7 (1,5 điê
1. Phản ứng 0,25. Đúng 3, 4 cấu trúc được 0,25. Đúng 5 cấu trúc được 0,5. Đúng dưới 3
cấu trúc không cho điểm

2. Đúng mỗi cấu trúc được 0,25

̉ m)
Bài 8 (2.0 điê
1. Tổng hợp hữu cơ
a) Đúng mỗi cấu trúc được 0,25

b) Xác định đúng số công thức cộng hưởng được 0,5

c) Xác định đúng có 101 công thức cộng hưởng của T được 0,25
2. Cơ chế phản ứng (0,75)

̉ m)
Bài 9 (1.5 điê
1. Xác định đúng mỗi chất được 0,25

2. Đúng cơ chế được 0,75 điểm (chỉ cần giải thích được sự tạo thành NH3)
̉ m)
Bài 10 (3.0 điê
Đúng các chất từ B tới E được 0,5 điểm, 3 chất được 0,25, 2 chất trở xuống 0
điểm
Đúng các chất tới I được 0,5, 3 chất được 0,25, 2 chất trở xuống 0 điểm
- Đúng tất cả các chất từ I đến X được 2 điểm
- Đúng 7-8 chất được 1,75
- Đúng 5,6 chất được 1,5.
- Đúng 3,4 chất được 1 điểm
- Đúng 1,3 chất được 0,5
Đúng đến đâu tự cho điểm đến đó. Nếu một chất trong dãy sai thì
nguyên đoạn sau không cho điểm nữa
Đề 6
̉ m)
Bài 1 (3.0 điê
1. Tinh thể
a) Khối lượng riêng được tính theo công thức
m n.137
 
V N A .a 3
Do Ba có mạng tinh thể lập phương nên ta sẽ có bảng sau: (0,75)
Lập phương đơn Lập phương tâm Lập phương tâm
giản khối mặt
Chiều dài cạnh ô 2r = 434,8.10 cm
-10
4r/ 3 =502,1.10- 2 2 r=614,9.10-
mạng cơ sở 10cm 10cm

Số nguyên tử Ba 1 2 4
có trong một ô
mạng
Kết quả tính khối 2,77 g/cm3 3,60 g/cm3 3,92 g/cm3
lượng riêng
Dựa vào kết quả ta thấy Ba có mạng tinh thể lập phương tâm khối (0,25)
b) Số phối trí đều là 6 (0,25)
2- 2-
c) Trong bari peroxid thì anion O sẽ bị thay thế bằng O2 định hướng thẳng nên
nó sẽ được sắp xếp song song theo một trục, điều này làm cho cấu trúc lập
phương bị lệch một cách đáng kể như hình vẽ
(0,5)

2. Cấu trúc phân tử


a) Cấu trúc hình học của anion:
(0,5)

Anion này có dạng đường thẳng, vì chỉ khi ở dạng này các e trên oxy mới có thể
ở vị trí cách xa đến mức tối đa. Ngoài ra cũng có ozonid mà anion này ở dạng
góc, đáp án này cũng được chấp nhận
(0,25)
b) Một số cấu trúc đẳng electron khác như NF2, S3-…
(0,25)
c) Hệ thống này có 19e, và như vậy nó sẽ là một gốc tự do vì thế nên ta có thể
đoán được chúng dễ bị dime hóa. Thực sự điều này đã được kiểm chứng ở một
số trường hợp như S62-; N2F4…
(0,5)
̉ m)
Bài 2 (2.5 điê
1. Nhiệt động học
a) Hằng số cân bằng của phản ứng được tính theo biểu thức K = Kthuận / Knghịch
Kết quả tính toán các hằng số cân bằng được cho ở bảng
(0,25)
T (K) 400 500 600 700 800
K 258 127 80,1 57,4 44,6
Nhiệt độ càng tăng thì Kcb càng giảm nên theo nguyên lý Le Chartelier phản ứng
là tỏa nhiệt
(0,25)
b) Từ phương trình Van’t Hoff ta có:
K H  1 1 
ln 2      . Như vậy ta có thể vẽ được đường biểu diễn lnK như một
K1 R  T2 T1 
hàm của nhiệt độ (lnK = f(1/T).
Theo đề bài ta sẽ có các giá trị sau
T (K) 400 500 600 700 800
1/T (K )
-1 2,50.10 -3 2,00.10 -3 1,67.10-3 1,43.10-3 1,25.10-3
lnK 5,55 4,85 4,38 4,05 3,80
Từ đó ta có đồ thị biểu diễn hàm lnK = f(1/T) (0,25)
Từ giá trị ở 400 và 800K ta có thể tính được ∆H = -11,7 kJ/mol
(0,25)
∆G = -RTlnK ⇒ ∆S = (∆H - ∆G) / T
T (K) 400 500 600 700 800
∆G (kJ/mol) -18,5 -20,1 -21,9 -23,6 -25,3
lnK 16,9 16,9 16,9 17,0 16,9
-1
∆S = 16,9 J.K .mol -1

(0,25)
c) Gọi  là độ phân ly của HI ở 600K thì từ biểu thức Kc ta dễ dàng rút ra biểu thức
CHI2
4 2
K  = 80,1 ⇒  = 0,955 (0,25)
CH 2 .CI2 1  
Số tiểu phân trước và sau phản ứng không đổi nên áp suất không ảnh hưởng
đến cân bằng
(0,25)
2. Động học
a) Nếu n = 1. thì 3.33•10 -5/7.69•10 -6 = 4.33 ≈ 1.67•10 -2/3.85•10 -3 = 4.34
Nếu n = 2. thì 2.00.10 -4/3.33•10 -5 = 6.00 ≈ 0.100/1.67•10 -2 = 5.99
(0,25)
b) Tính toán: k = 3.85•10 -3/7.69•10 -6 = 501 L/mol.s
k = 1.67•10 -2/3.33•10 -5= 502 L/mol.s
k = 0.100/2.00·10 -4= 500 L/mol.s
kM = 501 L/mol.s (0,25)
̉ m)
Bài 3 (2.5 điê
1. Cân bằng ion
Anilin là base được ký hiệu là B, còn axit sunfanylic là axit được ký hiệu A
Giá trị pH được xác định dựa trên tỉ số [B] / [BH+] hay [A-] / [HA]
Trong dung dịch có cân bằng sau
B + HA ⇌ BH+ + A-
Ban đầu 0,02 0,015
Cân bằng 0,02 – x 0,015 – x
Ta có biểu thức
 BH    A 
K
 B  HA
(0,25)
Ka được xác định bằng con đường như sau
 H    A   A  Ka
Ka   
 HA  HA  H  
 BH   OH    BH   K b  H  
Kb     (0,25)
 B  B KW
 BH    A  K .K
 a b  4, 47
 B  HA KW
Thay trị Ka vào ta tìm được giá trị x = 0,01151M
(0,25)
Dùng biểu thức tính pH cho dung dịch đệm xác định được pH = 4,50
(0,25)
2. Điện hóa Nano
a) Thế của nửa pin được mô tả bằng phương trình Nernst:
RT C (Oxh)
E  Eo  ln
nF ( Kh)
Tổng điện thế U bằng E(catot) – E(anot)
E – Eo + RTF-1lnC(Ag+)
RT C2 ( Ag  )
U1 = E2 – E1 và U1 = ln
F C1 ( Ag  )
Với U1 = 0,170 V; C2(Ag+) = 0,01M; C1(Ag+) = x M
8,314.298,15 0, 01
0,170   ln → x = 1,337.10-5M
96485 x
Như vậy Ksp = 1,788.10 -10 (0,5)
b) Nửa pin bên trái của (II): E(AgCl) = 0,8 + RTF-1ln(1,337.10-5) = 0,512V
U = E(AgCl) – E(Agn, Ag+) mà
E(Agn, Ag+) = Eo(Agn, Ag+) - RTF-1ln(0,01)
Với cụm nano Ag10 thì E(Ag10 / Ag+) = 0,512 – 0,430 = 0,082V. Như vậy thế
chuẩn của cụm nano Ag10 là Eo(Ag10 / Ag+) = 0,200V
(0,25)
Với cụm nano Ag5 thì E(Ag5 / Ag) = 0,512V – 1,030V = -0,518V. Như vậy thế
chuẩn của cụm nano Ag5 là Eo(Ag5 / Ag) = -0,400V
(0,25)
c) Thế tiêu chuẩn tăng theo kích thước hạt đến khi đạt giá trị lớn ứng với một kích
thước nhất định nào đó. Tập hợp các hạt bé hơn có thế thấp hơn do chúng có bề
mặt rộng hơn và quá trình kết tinh kém thuận lợi hơn về mặt năng lượng đối với
những nguyên tử trên bề mặt. Do đó năng lượng tự do tạo thành bạc kim loại
lớn hơn (ít âm hơn) với các hạt nhỏ, vì vậy thế tiêu chuẩn thấp hơn. Hiệu ứng
này giảm khi kích thước hạt tăng do sự giảm lượng tương ứng các nguyên tử
trên bề mặt. (0,5)
Chú ý thêm: Tuy nhiên thế không tăng mãi theo sự tăng của kích thước. Thế điện
hóa của một số tập hợp nhỏ có kích thước nhất định có thế cao hơn. Điều này do các
tập hợp này được chắn hoàn toàn (tập hợp có chứa một “số kỳ ảo” (magic number) các
nguyên tử) làm cho chúng bền hơn
̉ m)
Bài 4 (3 điê
1. Dãy chuyển hóa phức chất (Đúng mỗi chất được 0,25 điểm)

2. Hóa nguyên tố
- Dựa vào phản ứng với HCl ta có thể xác định được 1 và 3 có thể là Au hay Hg.
Điều này được tái khẳng định dựa vào sự tạo thành hỗn hống. Hg là kim loại có
nhiệt độ nóng chảy thấp nhất và dựa vào luận điểm 2 có nhiệt độ nóng chảy
thấp hơn 3 thì 3 sẽ là Au và 1 là Hg
(0,5)
- 4 và 6 sẽ là Na hoặc Ca do chúng phản ứng mãnh liệt với nước và Al và Sn sẽ có
thể là 2 và 5 (do chúng là các kim loại lưỡng tính). Để phản ứng được với kiềm
thì cần phải đun nóng 2, nên 2 là Sn và 5 là Al
(0,5)
- Ca và Al là chất lỏng ở 1000oC, còn Na là chất khí do Na có nhiệt độ sôi thấp hơn
1000oC (883) nên 4 là Ca và 6 là Na. Hg dĩ nhiên là khí còn lại. Au và Fe là các
chất rắn ở nhiệt độ này. Kim loại 7 phản ứng được với HCl nhưng không cho
hiện tượng với kiềm nên 7 là Fe.
(0,75)
- Fe có khối lượng riêng thấp hơn Au (7,9 < 19,3 g/cm3) nhưng nhiệt độ nóng
chảy lại cao hơn (1538 > 1064oC). Sn có khối lượng riêng thấp hơn Fe (7,3 < 7,9
g/cm3) và nhiệt độ nóng chảy thấp hơn Au (232 < 1064oC)
̉ m)
Bài 5 (2.0 điê
1. Do trong phản ứng SN2 thì cấu trúc bậc 1 sẽ bị ảnh hưởng không gian lớn hơn so
với cấu trúc bậc 3 (0,5)
2. HMPA và DMSO là các dung môi solvat hóa tốt các cation, cation càng bị solvat
hóa tốt thì tính base của anion RO- tương ứng càng mạnh. Vậy trong mọi trường
hợp anion t – BuO- có tính base mạnh hơn MeO-
(0,5)
3. Do có thể tạo được phức bền, như cấu trúc phức đồng như sau (0,25 cho lập
luận, và 0,25 cho hình vẽ)
4. Mạch cacbon càng dài tính kỵ nước càng tăng, thắng lực liên kết hydro. Thể tích
hơi giảm đi vì liên kết hydro kéo các phân tử lại gần nhau hơn khoảng cách vốn
có giữa chúng, tức là khoảng cách giữa etanol và nước sẽ nhỏ hơn khoảng cách
giữa hai phân tử etanol trong etanol tinh khiết và hai phân tử nước trong nước
tinh khiết (0,5)
̉ m)
Bài 6 (2.5 điê
1. Do phản ứng được với thuốc thử axit fuchsinsunfurơ chứng tỏ C5H10O là một
andehit, điều đó có nghĩa là C5H12O là một ancol bậc 1
(0,25)
Việc tạo thành hai dẫn xuất brom đồng phân và sự thu về 2 ancol đồng phân
cho thấy phản ứng của ta sẽ có sự chuyển vị
(0,25)
Việc oxy hóa mãnh liệt hydrocarbon C cho ta chỉ CH3COOH và CO2 chứng tỏ
trong cấu trúc hydrocarbon sẽ có bộ khung (CH3 – C(R)= )
Từ tất cả các lập luận này ta có thể xác định được A là Me3C – CH2OH, B là Me3C
– CHO và C là Me2C = CH – CH3 (0,25 cho mỗi cấu
trúc đúng)
2. Chất C5H8O3 của ta là một este, và axit tương ứng khi đun nóng sẽ mất CO
chứng tỏ nó là một xetoaxit
(0,25)
Việc cho kết quả dương tính với thuốc thử haloform chứng tỏ có sự hiện diện của
CH3CHOH (0,25)
Từ đó suy ra C5H8O3 chính là este etylpyruvat (0,25)
Như vậy C10H16O4 chính là sản phẩm dietyl este của axit fumaric chứ không phải
là maleic (axit B), vì phải đun nóng một thời gian lâu mới cho ra sản phẩm
(0,25)
Sự cis – hydroxyl hóa hỗn hợp axit fumaric và maleic sẽ cho ra hỗn hợp 3 đồng
phân lập thể
(0,25)
̉ m)
Bài 7 (3.0 điê
1. Sơ đồ tổng hợp như sau (Đúng mỗi chất được 0,25)
2. Cơ chế phản ứng, ở đây không cần quan tâm lập thể của cơ chế (Đề nghị chính
xác cơ chế được 1 điểm)

3. Sơ đồ tổng hợp (Nếu tổng hợp từ 6-8 phản ứng cho 0,5, dài trên 8 phản ứng chỉ
0,25)
HOOC – COOH + EtOH → (COOEt)2

̉ m)
Bài 8 (1.5 điê
1. Sơ đồ tổng hợp ornitin (Đúng mỗi chất được 0,25, đúng pHI được 0,25)

2. Sản phẩm A là sản phẩm tách nước của D-altrose khi nhóm CH2OH ở C6 và nhóm
OH ở C1 cùng chiếm vị trí axial trong cấu trúc vòng 6 cạnh dạng ghế
(0,5)
NĂM 2010
Đề 1
Câu 1: (1,75 điểm): Đúng mỗi ý cho 0,25, sai không cho điểm nào
a)

b) 8 hốc tứ diện .
Be2B
4 đơn vị công thức (Be8B4)
c) Số phối trí(bo) = 8; (Be) = 4
d)
a 0 2  2.329
a 0  465pm
1
(8  9.012  4  10.811).
m 6.022  10 23  1.90g / cm3
e)  
V (465  10 10 ) 3

f)
1
.a 0 3  201pm
4
g)
x*
y* z*

2p 2p
xb

yb zb

s*
2s 2s

sb
Câu 2: (2,5 điểm): Đúng mỗi ý câu 1 được 0,25 – Đúng câu 2 được 1 điểm
1. SCl2: dạng AL2E2. Phân tử có dạng góc.
SO3: dạng AL3E0. Phân tử có dạng tam giác đều, phẳng.
SO2ClF: dạng AL4E0. Phân tử có dạng tứ diện với S ở tâm tứ diện. Do các phối tử khác nhau nên
chất này có dạng tứ diện lệch chứ không phải tứ diện đều
SF4: dạng AL4E1. Có dạng cái bập bênh.
SBrF5: dạng AL6E0, Bát diện lệch.

2. Khi tạo sản phẩm cộng với pyridin thì cấu trúc xung quanh nguyên tử bo trung tâm đã trở thành
cấu trúc lai hóa tứ diện. Sự chuyển đổi cấu trúc này dẫn đến ảnh hưởng không gian lớn xung
quanh nguyên tử bo nhất là khi X là nhóm có kích thước lớn (chẳng hạn iot) làm cho sản phẩm
cộng được tạo thành không được ưu tiên. Do đó BF3 cho khả năng tạo sản phẩm cộng lớn nhất.
(BF3 có tính axit Lewis mạnh nhất)

X X

N: + B N B

X X X
X

sp2 sp3
Câu 3: (4 điểm): Câu 1 đúng mỗi câu a, b được 1 điểm – Câu 2 1 điểm, câu 3 đúng mỗi ô được 0,25 (Hai
ô sinh nhiệt của C và H phải đúng hết mới được 0,25)
1.

a)
87
37 Rb  38
87
Sr 01 
b) 87Sr
lúc này = 87Srt=0 + 87Rbt=0 – 87Rblúc này
87Rb = 87Rbt=0 exp(-t)
lúc này

(87Rbt=0 / 87Rblúc này) = exp(t)


87Sr
lúc này = 87Srt=0 + 87Rblúc này(exp(t) – 1)
(87Sr lúc này / 86Sr) = (87Sr t = 0 / 86Sr) + (87Rblúc này / 86Sr)(exp(t) – 1)
y = c + x(m)
Trong phép tính gần đúng thì (87Sr lúc này / 86Sr) với A và B như nhau
m = (0.709 – 0.699) / (0.180 – 0.004) = 0.0568 = (exp(t) – 1)
t = ln(2)t / t1/2
t1/2 = 4.8.1010 năm
t = (4.8.1010)ln(1.0568) / ln2 = 3.8.109 năm

2. Ta có:

[Cu(NH3)2]+ ⇌ Cu+ + 2NH3

Cu    NH 3 
2

KD   1011
Cu ( NH 3 ) 2 

Biết Eo(Cu+/Cu) = 0,52V suy ra biểu thức Eo([Cu(NH3)2]+ / Cu) = 0,52 – 0,059pKD = -0,14V (1)
[Cu(NH3)4]2+ + 2e  Cu + 4NH3 Eo = -0,02V
[Cu(NH3)2]+ + e  Cu + 2NH3 Eo = -0,14V
Từ đó ta suy ra Eo([Cu(NH3)4]2+ / [Cu(NH3)2]+ = 2(-0,02) – (-0,14) = 0,10V (2)
So sánh (1) và (2) rút ra kết luận ion phức [Cu(NH3)2]+ bền vững trong dung dịch

3.
∆Hf của than chì và H2 là 0kJ/mol
Ta có sơ đồ sau:
C3H6(xiclopropan)
+4,5O2
Đồng phân hóa
3CO2 + 3H2O
Đốt cháy

C3H6(propen)
Dựa vào sơ đồ trên ta tính được ∆H (đốt cháy propen) = -2061,5kJ/mol
+4,5O2
Ta cũng có sơ đồ sau:
3C + 3H2 + 4,5O2

Hình thành
3CO2 + 3H2O

Đốt cháy

C3H6(xiclopropan)

+ 4,5O2
Dựa vào sơ đồ trên ta tính được ∆Hf (xiclopropan) = 53,2kJ/mol
Phép tính tương tự đối với propen cho kết qủa:∆Hf(propen)=20,3kJ/mol

Câu 4: (2 điểm): Câu 1, 2: 0,25 điểm – Đúng mỗi câu 3, 4, 5 được 0,5, sai không cho điểm
1) Tiểu phân phản ứng trước hết là: A2-
Sản phẩm là HA-
2) Số mmol sản phẩm = 1,00.0,300 = 0,300mmol.
3) HA- + H2O ⇌ H2A + OH-
4) Tại pH = 8,34 = (pKa1 + pKa2)/2 tất cả A2- đều bị proton hóa thành HA-.
Do đó số mmol A2- có mặt trong dung dịch lúc đầu = 3,00mmol
Tại pH = 10,33 hệ là một dung dịch đệm với tỉ lệ [A2-]/[HA-] = 1. Như vậy:
[HA-]lúc đầu + [HA-]tạo thành = [A2-]lúc đầu - [HA-]tạo thành
Như vậy số mmol HA lúc đàu = 3,00 – 0,300 – 0,300 = 2,40mmol.
5) VHCl = [(2.3,00) + 2,40]/0,300 = 28,00mL

Câu 5: (2 điểm)
a) Na2N2O2: natri hyponitrit
b) Về nguyên lý thì anion [N2O2]2- có thể có đồng phân hình học. Thực nghiệm chứng tỏ đồng phân trans ưu
thế hơn
c) Sản phẩm có thể bị lẫn NaNO3, NaNO2, NaHCO3 hay Na2CO3
d) 2Na2N2O2 + 4Na + 2H2O  Na2N2O2 + 4NaOH
e) Na2N2O2 + CO2  Na2CO3 + N2O

Câu 6 (2 điểm): Giải thích đúng mỗi ý được 1 điểm


1. Trong dung môi dioxan xảy ra sự giữ nguyên cấu hình do dung môi dioxan đã solvat hóa carbocation tạo
thành ở phía đối diện với nhóm đi ra
ROH + SOCl2  ROSOCl + HCl
Trong pha khí hay khi có mặt pyridin sẽ xảy ra sự tấn công sau của anion Cl- tạo sản phẩm quay cấu hình:
C5H5N + HCl  C5H5NH+ + Cl- (khi có mặt pyridin) hay HCl  H+ + Cl- (pha khí)

2. Đối với hợp chất allen thì nguyên tử carbon trung tâm lai hóa sp, và hai AO p còn lại phân bố vuông góc
với nhau. Điều này gây ra sự cản quay của các obitan p quanh trục phân tử và dẫn đến sự tạo thành các
đồng phân quang học như hình vẽ:

Điều kiện để allen có đồng phân quang học là a khác b và c khác d. C,d có thể trùng với a, b
Đối với allen có số nối đôi lẻ thì các nhóm thế lại trở đồng phẳng, và xuất hiện đồng phân hình học như
hình vẽ dưới:

Câu 7 (2 điểm): Xác định đúng cấu trúc các chất được 0,25, vẽ đúng công thức Fischer và xác định R, S
đúng cho mỗi công thức được 0,25.
Câu 8 (2 điểm): Xác định đúng mỗi cấu trúc được 0,25 – Đề nghị cơ chế đúng 1 điểm – Tổng hợp không
quá 5 bước cho 1 điểm

Cơ chế: Enol hóa rồi este hóa


Tổng hợp:
Câu 9 (2 điểm): Câu 1: 0,25 – Câu 2 đúng mỗi chất 0,25 – Câu 3 so sánh đúng mỗi ý được 0,5
1. Cấu trúc đường thỏa mãn yêu cầu:

2. Cấu trúc các chất trong dãy:

3. So sánh:
(i) Piridin có đôi electron (thuộc obitan sp2) sẵn sang để dùng chung với axit; trong khi pirole chỉ có
thể kết hợp với proton khi đánh đổi tính thơm của vòng.
(ii) Có thể xảy ra sự phân tán các electron không liên kết của nhóm –NH2 vào nhân. Hệ qủa là có sự
tăng mật độ electron trên nguyên tử nitơ của dị vòng, do đó có sự tăng tính bazơ ở vị trí này.

NH2 NH2 NH2 NH2


N N N N
Đề 2
Bài 1: (0,5 cho mỗi liên kết hydro trong bazơ nitơ - 1 điểm cho cấu trúc hexame)
H

H3C O H N N
H

Adenin
N
H N H N
R
Thymin N N

R O H

H N H O N
H

Guanin
N
H N H N
R
Cytosin N N

R O H N

H
F
H H
F F

H H

F F
H H
F

Bài 2: (1,0 cho ý 1 – 1 điểm cho nêu đúng dạng hình học các anion – 0,5 cho giải thích đúng)
1. ClO- : Đường thẳng
ClO2- : Gấp khúc
ClO3- : Chóp tam giác đều
ClO4- : Tứ diện đều
Do số cấu trúc cộng hưởng tăng dần từ ClO- đến ClO4- nên độ bền tăng, và khả năng oxy hóa
giảm.
2. Ở nhiệt độ thường phân tử N2F2 tồn tại ở dạng trans có momen lưỡng cực bằng 0. Dạng này bền
vững để tránh tương tác lưỡng cực, lưỡng cực của phân tử. Còn ở nhiệt độ cao phân tử tồn tại
chủ yếu ở dạng cis có momen lưỡng cực lớn
Bài 3: (0,5 điểm cho câu 1 – 1,0 điểm cho mỗi câu a, b – 0,5 điểm cho mỗi câu điện hóa)
1. Ta có:

k (T2 ) Ea  1 1 
ln    
k (T1 ) R  T1 T2 
1 1 ln 3.R
    1,5.103
T2 T1 Ea
 T2  667 K
2. K1 = PCO2 = 0,2 atm
2
PCO
K2   PCO  2.02  0, 632atm
PCO2
Gọi x, y là số mol CaCO3 và CO2 đã phản ứng. Từ đó suy ra số mol các chất ở trạng thái cân
bằng là: 1 – x mol CaCO3 , x mol CaO , x – y mol CO2 , y mol C và 2y mol CO.
PCO2 .V PCO .V
x y   0, 05 mol CO2 , 2 y   0,158 mol CO
RT RT
 nCaO = 0,129 mol; nCaCO3 = 0,871 mol; nC = 0,921 mol
Khi xảy ra sự phân hủy hoàn toàn thì x = 1.
Lúc này nCO2 = 1 – y mol và nCO = 2y mol. Do áp suất CO2 và CO không đổi nên có hệ phương
trình:
0,632V – 2yRT
0,2V = (1 – y)RT
Giải hệ phương trình này cho V = 173,69 lít
3. 2NiO(OH) + 2H2O + 2e → 2Ni(OH)2(r) + 2OH- Eoc = -0,490V

E c  E co 
RT
2F

ln OH  
2

Cd(r) + 2OH- → Cd(OH)2(r) + 2e Eoa = 0,809V

RT 1
E a  E ao  ln
2F 
OH   2

nap
Cd(r) + 2NiO(OH)(r) + 2H2O 2Ni(OH)2(r) + Cd(OH)2(r)
phong

E = Eoa – Eoc = 1,299V

700mAh = 0,700A . 3600s = 2520C

nCd = 2520/2.96485 = 0,013mol  mCd = 0,013.112,4 = 1,47g


Bài 4: (Đúng mỗi câu được 1,0 điểm)
1. SO2 + H2O ⇌ H+ + HSO3- K1 = 10-2 (1)
HSO3- ⇌ H+ + SO32- K2 = 10-7 (2)
Sự kết tủa chỉ xuất hiện khi [SO32-] đạt 0,01M
Nồng độ [SO32-] trong dung dịch không nhỏ nên trong dung dịch coi như không tồn tại SO2
Suy ra: [HSO3-] = 0,02 – 0,01 = 0,01M
Từ cân bằng (2) suy ra [H+] = K2[HSO3-] / [SO32-] = 10-7M
pH = -lg[H+] = 7
Vậy cần áp đặt pH = 7 để bắt đầu quan sát được kết tủa CaSO3
2. Do hằng số tạo phức của Ag(S2O3)23- , Kf = (Kd)-1 = 1,667.1013 là rất lớn nên hầu hết Ag+ thêm
vào sẽ tạo phức hết với S2O32-
 [Ag(S2O3)23-] = 0,100M
Số mmol S2O32- tự do = 530 – (2.20) = 490 mmol
 [S2O32-] = 2,450M
Nồng độ ion Ag+ tự do được tính từ Kd

 Ag    S 2O32 
Kd   6, 0.1014
 Ag ( S 2O3 ) 2 
3

 [Ag+] = 1,0.10-15M
Từ Ksp = [Ag+][I-] = 8,5.10-17
 [I-] = 8,5.10-2M
 mmol KI = 17,0 mmol

Bài 5: (1,5 điểm)


a) ZnCO3(s) + S2–  ZnS(s) + CO32–
SO32– + CH2O + H+  CH2(OH)SO3–
2 S2O32– + I2  S4O62– + 2I–
b) S2O32–
c) n(S2O32–) = 2  5.20  0.01000 = 0.104 mmol (trong 20.00 mL dịch lọc)
c(S2O32–) = 0.104 / 20.00  50.00 / 20.00 = 0.0130 mol/L (trong dung dịch đầu) =
0.01300112.13 g/L = 1.46 g/L (1460 ppm)
Bài 6: (Đúng mỗi ý a, b được 0,5 điểm, xác định trị pK đúng được 0,5, giải thích đúng 0,5)
1. a) 1,3 – đicloxiclopentan có 3 đồng phân :

Đôi đối quang meso


b) 1 – metyl – 2,3 – đicloxiclopropan có 4 đồng phân :

Đôi đối quang Đôi đối quang

2. Cấu trúc của vitamin C


Vị trí hai nhóm chức có giá trị pKa tương ứng là

Nhóm OH enol bên trái (gần mạch nhánh) sẽ có trị pK cao hơn do base tương ứng được ổn định
mạnh do liên hợp với nhóm cacbonyl. Nhóm OH bên cạnh sẽ có pK thấp hơn vì sự mất một H ra khỏi
anion bền sẽ khó khăn hơn, mặt khác giữa H enol thứ hai và O- có sự tạo thành liên kết hydro nội phân
tử (tương tự như so sánh pK của axit fumaric và maleic)

Bài 7: (Cấu trúc các chất đúng 1 điểm, đúng cơ chế 0,75, giải thích 0,25 – Sơ đồ tổng hợp không quá 7
giai đoạn cho 1,0 điểm. Nhiều hơn chỉ được 0,5)
Anion xiclopentadienyl được tạo thành do anion có sự bền vững cao (hệ thơm bền), và nó là một
tác nhân nucleophin tốt.
2. Sơ đồ tổng hợp

Bài 8: (Đúng mỗi công thức được 0,25 điểm)


- Hợp chất A khi phản ứng với KMnO4 cho axit cacboxylic quang hoạt  A có chứa C* và có liên
kết đôi ở cacbon bậc 2.

- Hợp chất B tác dụng với KMnO4 tạo dixeton E không chứa C*. Vậy B không chứa C bất đối và có
liên kết đôi ở hai cacbon bậc 3.

- Hợp chất C tạo ra F vừa chứa nhóm cacboxyl vừa chứa nhóm xeton và cũng có C*  C có chứa
C* và có liên kết đôi ở cacbon bậc 2 và cacbon bậc 3
Bài 9: (Xác định đúng cấu trúc Q được 0,5 – Sắp xếp đúng trật tự amino axit 0,5, giải thích pHI 0,5)
1. Cấu trúc các chất

2.
a) Hexapeptit M có đầu N là Ala. Thủy phân M nhờ trypsin xác định được tripeptit có cấu tạo phù
hợp là Ala – Tyr – Arg. Dipeptit E có cấu tạo phù hợp là Arg – Phe và tripeptit G có cấu tạo phù
hợp là Arg – Phe – Ile. Do vậy amino axit đầu C là Gly. Từ đó xác định được cấu tạo của M là
Ala – Tyr – Arg – Phe – Ile – Gly.
b) pHI lớn nhất là của Arg, vì có nhóm guanidin có tính bazơ mạnh. Còn pHI nhỏ nhất của Phe do
có nhóm phenyl.
Đề 3
Bài 1: (0,25 cho xác định đúng số phố trí – 0,75 cho tính toán đúng hằng số mạng – 0,5 cho xác định
đúng ảnh hưởng với bảng tuần hoàn, và 0,5 cho viết đúng cấu hình e)
1.
a) Nguyên tử lưu huỳnh được phối trí bởi một nguyên tử lưu huỳnh khác và ba ion Fe2+ trong một
sự sắp xếp tứ diện (số phối trí 4)
b) Ô mạng nhỏ nhất ứng với hằng số mạng là ao chứa 4 nguyên tử Fe và 8 nguyên tử S. Điều này
dẫn đến:
4M(Fe)+8M(S)
ρ= 3
=5,011g/cm3
NAa o
Từ phương trình này chúng ta nhận được kết qủa ao = 541,8pm
2. Do thiếu chiều cao nên các obitan định hướng theo trục z sẽ không tồn tại. Bảng tuần hoàn lúc
này sẽ bị thiếu đi nhiều nguyên tố so với trong không gian ba chiều.
Cấu hình electron: 1s22s22p43s23p44s23d4

Bài 2: (Đúng mỗi câu được 1 điểm)


1. Năng lượng cần thiết để tạo thành anion từ O2 tăng theo thứ tự O2-  O22-  O2- và năng lượng
mạng tinh thể giải phóng trong quá trình tạo thành hợp chất ion tương ứng cũng tăng lên (Peroxit
có điện tích lớn hơn superoxit và oxit có kích cỡ bé hơn peroxit). Cation càng bé thì sự khác biệt
giữa các năng lượng mạng tinh thể càng lớn và cũng tương tự cho sự khác biệt giữa sự tạo
thành oxit, superoxit và peroxit. Trong trường hợp của Li thì sự khác biệt giữa năng lượng mạng
lưới lớn đến mức đủ bù cho năng lượng cần thiết để tạo thành oxit, như vậy Li2O sẽ là sản phẩm
bền nhất. Đối với các cation lớn hơn thì các năng lượng mạng lưới khác sẽ bé hơn và không đủ
bù trừ năng lượng nên các anion có nhiệt tạo thành không lớn sẽ được ưu tiên hơn
2. [Zn(H2O)4]2+ : ion tetraaqua kẽm (II)
[Zn(NH3)4]2+: ion tetraamin kẽm (II)
[Zn(CN)4]2-: ion tetraxianozincat (II)
[Zn(OH)4]2-: ion tetrahidroxozincat (II)
Zn2+: [Ar]3d10 → không có e độc thân → nghịch từ
Zn2+ có phối trí 4, không có obitan d trống → lai hóa sp3 → phức của Zn2+ có cấu trúc tứ diện
3. Nhận xét
- Phối trí 6 → Phức bát diện
- Áp dụng thuyết trường tinh thể
 P < ∆CN-: [Fe(CN)6]4- có phức spin thấp
 P > ∆H2O: [Fe(H2O)6]2+ có phức spin cao
∆>P ∆<P
[Fe(CN)6]4- [Fe(H2O)6]2+
- Không có e độc thân - Có 4 e độc thân

 2   2  3 
- Ebền = 6.      2 P - Ebền = 4.      2.   
 5   5  5 
= 528 kJ/mol = - 49,6 kJ/mol

⇒ [Fe(CN)6]4- bền hơn [Fe(H2O)6]2+

⇒ [Fe(H2O)6]2+ có tính khử và tính oxy hóa mạnh hơn [Fe(CN)6]4-

Bài 3: (5,0 điểm)


1. (1,5 điểm)
Áp dụng phương trình Henderson – Hassenbalch cho hệ đệm ta có:

4, 76  5, 0  log
 HOAc
 AcO  
 HOAc   0,5715
 AcO  
[HOAc] + [AcO-] = 0,500
 [AcO-] = 0,3182, [HOAc] = 0,1818
Vậy trong 300 mL có 95,45 mmol AcO- và 54,55 mmol HOAc
Số mmol hợp chất nitro bị khử là 3
Từ phương trình tỉ lượng ta thấy để khử 3 mol hợp chất nitro cần 12 mol H+ , lượng H+ này nhận
được bằng sự phân ly HOAc
Khi khử hoàn toàn hợp chất nitro thì
mmol HOAc = 54,55 – 12,00 = 42,55 mmol
mmol AcO- = 95,45 + 12,00 = 107,45 mmol
42,55
4, 76  pH  log
107, 45
 pH  5,16
2. (Đúng câu a được 0,5, đúng câu b được 0,5)
a) ∆H˚pứ = ∆H˚f(NH3) + ∆H˚f(CO2) - ∆H˚f(urea) - ∆H˚f(H2O)
= (2(-80.8) + (-412.9)) - ((-317.7 - 285.8))
= 29.0 kJ

∆S˚pứ = 2(110) + 121 - 176 - 69.9 = 95.1 JK-1


∆G˚pứ = ∆H˚pứ - T∆S˚pứ
= 29000 - 298(95.1)
= 660 J
-G o
Do ∆G˚ = -RTlnK, vậy K=e RT
 e -0.266  0.766
(0.1)(0.01)2
b) ∆G = ∆G˚ + RTlnQ và Q =
(1)(55.5)
Tức ∆G = 660 + 8.314(298)(-15.53) = -37.8 kJ
Do ∆G < 0 nên phản ứng thủy phân ure sẽ xảy ra theo chiều thuận ở điều kiện đang xét.
3. (Đúng câu a được 0,5 điểm, đúng mỗi ý câu b được 0,5)
d[P]
a) Cho rằng biểu thức tốc độ phản ứng có dạng:  k[X]x[Y]y[Z]z
dt
Từ những dữ kiện đã cho tính được x = 2, y = 0, and z = 1/2

1
d[P] 2
Như vậy  k[X] [Z]2
dt

b) Thay số ở thí nghiệm thứ nhất vào:

1
d[P] 2
0.002 M h-1 =  k[0.01M] [0.01M]2
dt
3
1
Cho kết quả k = 200 M 2 h

Do Y và Z rất dư so với X nên biểu thức tốc độ phản ứng có thể được đơn giản hóa trở thành tốc

độ = = k'[X]2 với k' = k[2.00]0.5

Áp dụng biểu thức động học của phản ứng bậc hai ta có:

1 1
t1   1 1  0.35h (21 min )
2 c 0 k' (0.01)(283M h )
4. (1,0 điểm)
nAg = 0,1 / 108 = 9,27.10-4 mol
nNH3 = 10-2 mol
Ta có:
Ag+ + e = Ag E1 = Eo1 + 0,059lg[Ag+]

0, 059 PO2
O2 + 2H2O + 4e = 4OH- E2  E2o  lg
4 [OH  ]4
Khi phản ứng tạo phức đạt đến cân bằng thì E1 = E2.
Trong dung dịch NH3 0,1M [OH-] = (Kb.C)1/2 = 1,32.10-3M. Thay vào tính được E2 = 0,561V
Từ đó tính được [Ag+] = 9,12.10-5V
Như vậy tổng nồng độ [Ag+] trong dung dịch sẽ là
S = [Ag+] + [Ag(NH3)+] + [Ag(NH3)2+] = [Ag+](1 + 1[NH3] + 2[NH3]2)
Dựa vào số mol các chất lúc khởi đầu có thể cho rằng [NH3]  0,1M  [Ag+] = 15,5M
Vậy Ag tan hoàn toàn trong dung dịch NH3 0,1M khi có mặt oxy không khí.

Bài 4: (0,25 điểm cho mỗi chất đúng và mỗi phản ứng đúng)
2S + Cl2  S2Cl2
S2Cl2 + Cl2  2SCl2
2SCl2 + 2O2  2SOCl2 + SO2Cl2
2SOCl2 + O2  SO2Cl2

Bài 5: (Giải thích đúng câu a 0,75 điểm, câu b 0,5)


1. Do có tương tác giữa các H xuyên vòng trong [14]-annulen nên các liên kết đôi trong vòng sẽ trở
nên không đồng phẳng, hệ không liên hợp. Tuy nhiên với việc xuất hiện thêm liên kết ba thì số
tương tác xuyên vòng giảm đi, hệ trở thành hệ thống liên hợp phẳng nên có tính thơm.
2. Đồng phân OH-axial có liên kết hydro nội phân tử nên tính axit giảm đi so với OH-equatorial.
Bài 6: (Xác định đúng cấu trúc các chất được 0,25)

Bài 7:
1. (Xác định đúng cấu trúc các chất được 0,25. Cơ chế đúng 0,5)
3) Tổng hợp profofol (dài quá 10 giai đoạn cho 0,5)

Bài 8: (Xác định đúng cấu trúc đường được 1,25. Tổng hợp đúng được 0,75)
Đề 4
Câu 1: (Đúng mỗi ý được 1,0 điểm)
1. Anion kim loại kiềm được tạo thành bằng cách cho kim loại kiềm nhận thêm 1e tạo cấu hình e
ns2 bền vững nhưng do Z* bé nên khả năng tồn tại không cao. Tuy nhiên trong môi trường
amoniac lỏng thì có sự tạo thành electron solvat hóa qua quá trình Na  Na+ + e. Electron tồn tại
trong môi trường này được một phân tử Na nhận tạo thành Na-. Mặt khác ion Na+ sẽ tạo phức
với ete crown dẫn đến sự phát hiện Na- ở dạng [Na+L]Na-.
2. Fe: [Ar]4s23d6 ; Fe2+: [Ar]3d6. Mặc dù obitan 4s có năng lượng cao hơn nhưng việc điền đầy
electron vào các obitan 3d sẽ tạo thành một lực đẩy electron rất lớn. Như vậy electron sẽ phải
điền vào phân lớp 4s trước rồi mới 3d. Khi tạo thành ion thì electron ở mức 4s có năng lượng
cao hơn sẽ bứt ra trước.

Câu 2: (Đúng mỗi ý được 1,0 điểm)


1. Dựa vào số electron độc thân của các phức có thể đoán được phức [NiCl4]2- tồn tại ở trường tứ
diện và [Fe(H2O)6]3+ tồn tại ở trường bát diện, tức phân tử có hai dạng hình học tương ứng là tứ
diện và bát diện. Thuyết VB chỉ có thể giải thích được hình dạng chứ không giải thích được từ
tính của phức chất.
2. Năng lượng của hai liên kết Si – O là 932 kcal / mol rất lớn hơn so với Si=O chỉ có 640 kcal/mol.
Như vậy cấu trúc polymer với các tứ diện Si – O ưu thế hơn.

Câu 3: (Đúng câu 1 được 1,5 điểm, câu 2 được 1,5 điểm)
1. Phản ứng: 2NO + Br2 2NOBr
Gọi số mol chất ở thời điểm đầu là no, còn ở lúc cân bằng là n
noNO = nNO + nNOBr
noBr2 = nBr2 + 0,5nNOBr
Áp suất chung bằng tổng các áp suất riêng phần, như vậy:
pNO + pBr2 + pNOBr = 230,0 mm Hg
Nếu phản ứng không xảy ra thì áp suất riêng phần ở 335 K là
pNO = 180*335/297 = 203,0 mm Hg và
pBr2 = 0,72*62400*335/(1200*2*80) = 78,4 mm Hg
Do áp suất riêng phần của các khí tỉ lệ với số mol của chúng nên.
pNO + pNOBr = 203,0 mm Hg và pBr2 + pNOBr = 78,4 mmHg.
Từ đó có các phương trình:
pNO + pBr2 + pNOBr = 230,0 (1)
pNO + pNOBr = 203,0 (2)
pBr2 + 0,5pNOBr = 78,4 (3), từ đó có kết quả
pBr2 = (1) – (2) = 230,0 – 203,0 = 27,0 mm Hg
pNOBr = (78,4 – 27,0)*2 = 102,8 mm Hg
pNO = 230,0 – 27,0 – 102,8 = 100,2 mm Hg
Giá trị hằng số cân bằng là:

Kp = p2NO*pBr2/p2NOBr = 100,22*27,0/102,82 = 25,7 mm Hg = 0,0338 atm


Áp suất riêng phần của NO ở trạng thái cân bằng là 100,2 mm Hg. Sử dụng phương trình pV =
nRT sẽ có kết quả số mol:
100,2*1200 = n*62400*335; n = 100,2*1200/(62400*335) = 5,75*10-3 mol.
2. (Đúng chất S được 0,25, đúng phần còn lại được 1,25)
a) S là axit sunfuric đặc
b) nCO = nHCOOH = PV/RT = 1,70.10-3 (mol) = 0,0783 g
Dựa vào phản ứng ta có thể giả thiết phản ứng phân hủy có bậc 1.
Với giả thiết phản ứng có bậc 1 ta sẽ có các biểu thức:

1 c co no ( HCOOH ) 1, 70.103
k  ln o Với  
t c c nt ( HCOOH ) 1, 70.103  nCO
Từ bảng số liệu của đề bài ta sẽ có bảng kết quả sau:
t (s) 0 50 100 150 200 
nt (mol.10-3) 1,70 1,23 0,874 0,633 0,457 0
ln(co/c) 0 0,324 0,665 0,988 1,31
K 6,48 6,65 6,59 6,55
Do các giá trị k tính được gần như bằng nhau nên giả thiết là chính xác.
Tính được k = 6,57.10-3 s-1

Câu 4: (2,0 điểm)


1. PbC2O4  Pb2+ + C2O42- Go,1
Pb2+ +2e  Pb Go,2
PbC2O4 + 2e  Pb + C2O42- Go = Go1 + Go2
Go,1 = -RTlnKsol = -8,314  298  2,303  log(8,50  10-10) = 51,75 kJ/mol  51,8 kJ/mol
Go,2 = -nFEo = 2*96485  0,126 = 24,31 kJ/mol
Go = 51,75 + 24,31 = 76,06 kJ/mol;
Eo2 = -Go/nF = -76060/(2  96485) = - 0,394 V
2. [Pb2+][C2O42-] = 8,50  10-10,
[Pb2+] = 8,50  10-10/0,025 = 3,4  10-8 M
3. Cách 1: Tính từ [Pb2+]:
E = Eo1 + (2,303RT/nF)  log[Pb2+] = -0,126 - 0,221 = - 0,347 V
Cách 2: Tính từ [C2O42-]:
E = Eo2 -(2,303RT/nF)  log[C2O42-] = -0,394 + 0,047 = - 0,347 V

Câu 5: (Xác định đúng thành phần phản ứng được 1,0 điểm. Viết được các phản ứng được 2,0 điểm)
100,0.0,0300
n(HCl) = = 0,0822 mol
36,5
n(NaOH) = 0,00696 . 7,5 = 0,0522 mol
n(NH3) = 0,0822 - 0,0522 = 0,0300 mol
n’(NaOH) = 0,00562 . 7,5 = 0,0422 mol
n’(NH3) = 0,0822 - 0,0422 = 0,0400 mol
0,467
n(SO42-) = . 5 = 0,0100mol
233,4
Sử dụng Ba(NO3)2 thay cho BaCl2 không làm thay đổi khối lượng kết tủa nên trong dung dịch
không thể có SO32- .
Các phản ứng:
NH4NO3 + NaOH = NaNO3 + NH3 + H2O
(NH4)2SO4 + 2 NaOH = Na2SO4 + 2 NH3 + 2 H2O
HCl + NaOH = NaCl + H2O
NH4NO3 + 4 Zn + 5 H2O = 4 Zn(OH)2 + 2 NH3
BaCl2 + (NH4)SO4 = BaSO4 + 2 NH4Cl
Ba(NO3)2 + (NH4)2SO4 = BaSO4 + 2 NH4NO3
( NH4NO3 + 8 FeSO4 + 5 H2SO4 = (NH4)2SO4 + 4 Fe2(SO4)3 + 3 H2O )
2NH4NO3 + 2 FeSO4 + 2 H2SO4 = (NH4)2SO4 + 2 NO2 + Fe2(SO4)3 + 2 H2O
( NH4NO3 + FeSO4 + 3 H2SO4 = NH4HSO4 + NO2 + Fe(HSO4)3 + H2O )
Fe2(SO4)3 + 12 KSCN = 2 K3Fe(SCN)6 + 3 K2SO4
( Fe(HSO4)3 +6 KSCN = 3 KHSO4 + K3Fe(SCN)6)
100 mL dung dịch A chứa 0,010 mol (NH4)2SO4 và 0,010 mol NH4NO3 . Nồng độ mỗi chất là
0,10 mol/L.

Câu 6: (Đúng mỗi ý được 1,0 điểm)


1. Phản ứng 1 cho sản phẩm e do hướng tấn công ưu tiên. Phản ứng 2 cho sản phẩm exo do ít bị
cản trở không gian hơn.
2. Góc lớn hơn bình thường là góc của nhóm CH2 do tương tác đẩy mạnh của hai nhóm t-Bu. Góc
bé hơn bình thường là góc HCH cũng ở nhóm này do chịu ảnh hưởng tương tự
Câu 7: (Viết đúng mỗi phản ứng được 0,25. Nêu được ý tưởng 0,25 điểm. Vẽ được cấu trúc polyuretan
0,5 điểm)
ROH + R’NCO → RO – CONHR’
Khi phản ứng với nước thì RNCO + H2O → RNH2 + CO2
RNH2 + RNCO → RNHCONHR
Khí CO2 thoát ra thổi phồng vật liệu. Phần còn lại của vật liệu biến thành RNH2 tiếp tục phản ứng
với phân tử isoxianat tiếp theo
Phản ứng của isoxianat với etylenglycol dẫn đến sự tạo thành polyuretan có cấu trúc như sau:

Câu 8: (Đúng mỗi cấu trúc được 0,25. Đúng mỗi hai cấu trúc lập thể được 0,25. Cơ chế 0,5. Tổng hợp
dưới 5 giai đoạn cho 0,75 điểm, nhiều hơn 5 bước chỉ cho 0,25)
1. Cấu trúc các chất:

2. Các đồng phân lập thể


HO PhOCH3 HO PhOCH3
N(CH3 )2 H
H N(CH3 )2

HO PhOCH3 HO PhOCH3
H N(CH3 )2
N(CH3 )2 H

3. Phản ứng Mannich


4. Quy trình tổng hợp:
Câu 9: (Đúng cấu trúc mỗi chất được 0,25 điểm. Đúng cấu trúc peptiy giả được 0,5 điểm)
1. Cấu trúc L, M, N

N M L
CH2OH

CH2OH CHO
H OH

MeO H MeO H MeO H

H OMe H OMe H OMe

H OH
CH2OH CHO

CH2OH

Số cấu trúc C thỏa mãn đề bài:

Số cấu trúc khả thi 2

1 2
COOMe H

O H(OH) O H(OH)
H H

H COOMe
H OH
OMe OMe

OMe OH(H) OMe OH(H)

H OH H H

2. Dựa vào phép tổng hợp biết B là Leu và dựa vào dữ kiện DNFB biết A là Gly
Vậy cấu trúc peptit giả sẽ là:
Đề 5
Câu 1: (Xác định chính xác khối lượng riêng được 0,5. Tính đúng mỗi trị cực đại được 0,5. Viết đúng công
thức Clathrat được 0,5)
1. Độ dài các đường biên giới của một ô mạng cơ sở (gồm chiều dài và chiều rộng) = 912pm  độ
dài cạnh ô mạng = 456 pm
Vô mạng = (4,56.10-10m)3
Khối lượng các tiểu phân (Co + 2Al + 4O) trong một ô mạng là
m = [(58,93 + 2.26,98 + 4.16,00)/NA]g

m 176,89 g.mol 1
 = m/V    23 1 10 3
 3097904 g / m 3  3,1g / cm 3
V 6,022 .10 mol .(4,56.10 )m
d) r(M2+) = Bán kính của hốc tròn tứ diện – bán kính của ion oxit

4.r(O2-) = độ dài đường chéo của ô mạng = 456 2 pm


r(O2-) = 114 2 pm = 161,22pm
Độ dài cạnh của hốc tứ diện a = 2r(O2-) = 322,44pm

Độ dài bán kính của hốc tròn tứ diện r = a 6 /4 = 179,45pm


r(M2+) = r – r(O2-) = 36,23 pm
r(M3+) = (độ dài cạnh của ô mạng – 2r(O2-) / 2 = 66,78pm
2. 3Ar.23H2O

Câu 2: (Đúng ý 1 được 0,5. Đúng 2 chất ở câu 2 được 0,5, 3 chất 1 điểm. Đúng câu 3 được 0,5)
1. Dạng trans, để tránh các tương tác lưỡng cực
2. SNF3: tứ diện (AX4E0), SF4: bập bênh (AX4E2) và KrF2: thẳng (AX2E3)
3. Pd: [Kr]4d10
Khi tăng điện tích hạt nhân thì electron sẽ bị hút mạnh hơn, điều đó làm hạ mức năng lượng.
Việc này ảnh hưởng mạnh đến các electron thuộc phân lớp từ 4d trở đi do các AO d chắn kém.
Kết quả là mức năng lượng của phân lớp 5s và 4d lúc này không chênh lệch nhau nhiều nên việc
chuyển e từ 5s sang 4d thuận lợi hơn do đạt cấu hình bão hòa bền.

Câu 3: (3,0 điểm)


1. (Đúng mỗi chất được 0,25)
Hợp chất Khả năng tồn tại Lý do
XeF4 Có
NeF2 Không Ne có năng lượng ion hóa quá cao
XeCl4 Không Tương tác giữa các nguyên tử clo làm giảm
tính bền
KrF2 Có
KrBr4 Không Br2 có năng lượng cắt đứt liên kết quá cao
XeOF Không Xe không có SOH +3
XeF9 Không Xe không có SOH +9
2. (Xác định đúng cấu trúc chất được 1,25 điểm)

101325Pa.0,086876m 3
n= =3,531mol
R.299,82K
Đây chính là lượng khí sinh ra từ 28,35g chất chưa biết. Như vậy 1 mol khí được sinh ra khi
8,028g chất này phản ứng với nước. Cùng với tính dễ cháy của chất khí này và màu sắc của
ngọn lửa cho phép chúng ta kết lụân rằng chất ban đầu chính là liti hydrua.
LiH + H2O = LiOH + H2

Câu 4: (Đúng câu 1 được 1,0 điểm. Đúng 2a được 0,5 điểm, 2b được 1 điểm, 2c được 0,5. Đúng câu 3
được 1,0 điểm. Đúng mỗi ý câu 4 được 0,5 điểm)
1. Để sự kết tủa hoàn toàn thì S = [Zn2+] còn lại phải dưới 1,0.10-6M
Trong trường hợp này ta có: S = [Zn2+] + [ZnC2O4] + [Zn(C2O4)22-] > [ZnC2O4]
Tuy nhiên [ZnC2O4] = 1[Zn2+][C2O42-] = 7,08.10-4.2,75.10-8 = 1,95.10-3
Như vậy sự kết tủa không thể xảy ra hoàn toàn được
2.
a) Ta có Kcb = k1 / k-1
Từ những giá trị của đề bài dẫn đến bảng sau đây
T (K) 400 500 600 700 800
Kcb 258 127 80,1 57,4 44,6
K giảm khi tăng nhiệt độ. Như vậy theo nguyên lý Le Chartelier phản ứng tỏa nhiệt

K2 H  1 1 
b) Từ phương trình Van’t Hoff : ln     lập được bảng giá tri lnK = f(1/T) với các
K1 R  T2 T1 
giá trị lần lượt là
T (K) 400 500 600 700 800
1/T K-1 2,50.10-3 2,00.10-3 1,67.10-3 1,43.10-3 1,25.10-3
lnK 5,55 4,85 4,38 4,05 3,80
Từ đó ta có đồ thị:

Từ giá trị ở 400K và 800K tính được ∆H = -11,7 kJ/mol


∆G = -RTlnK và ∆S = (∆H - ∆G)/T
Vậy ta có kết quả
T (K) 400 500 600 700 800
∆G (kJ.mol-1) -18,5 -20,1 -21,9 -23,6 -25,3
∆S (J.K-1.mol-1) 16,9 16,9 16,9 17,0 16,9
∆S = -16,9 J.K-1.mol-1
c)  = 0,955. Áp suất tăng cân bằng vẫn không thay đổi.
3. t = 48 h = 2 ngày đêm.
Áp dụng biểu thức tốc độ của phản ứng một chiều bậc một cho phản ứng phóng xạ, ta có: =
0,693/t1/2; Với t1/2 = 2,7 ngày đêm,  = 0,257 (ngày đêm)-1.
Từ pt động học p.ư một chiều bậc nhất, ta có:  =(1/t) ln N0/N.
Vậy: N/N0 = e-  t = e-0,257 x 2 = 0,598.
Như vậy, sau 48 giờ độ phóng xạ của mẫu ban đầu còn là: 0,598 x 4 = 2,392(mCi).
Do đó số gam dung môi trơ cần dùng là: (2,392 : 0,5) – 1,0 = 3,784 (g).
4. Cu2+ + 2e → Cu Eo1 = 0,34V
Cu2+ + e → Cu+ Eo2 = 0,15V
 Cu+ + e → Cu Eo3 = 0,34.2 – 0,15 = 0,53V
CuI + e → Cu + I- Eo4 = Eo3 + 0,059lg10-12 = -0,17V
Vậy có phản ứng: 2Cu + 2HI → 2CuI + H2
Cu+ + e → Cu Eo1 = 0,53V
Cu2+ + e → Cu+ Eo2 = 0,15V
2Cu+ → Cu + Cu2+ Eo = 0,53 – 0,15 = 0,38V
Vậy ion Cu+ không bền trong dung dịch

Câu 5: (1,0 điểm)


1. Khi pKa thấp tức tính axit tăng (Z là nhóm hút e), ion benzoat được ổn định hơn, như vậy cân
bằng sẽ chuyển dịch về phía tạo thành anion này, tốc độ phân ly tăng.
2. Do cation PhCH2+ sinh ra sẽ tấn công vào vị trí para của vòng thơm (SE) dẫn đến sự tạo thành
polymer nên không thể quan sát tốt tính bền của cacbocation trong trường hợp này.

Câu 6: (Xác định được cấu trúc 1,0 điểm. Xác định đúng cấu trúc E và F được 0,5)
1. Dựa trên các dữ kiện của đề bài có thể đoán được D là azulen.
2. Phản ứng tạo ra E là SE nên sản phẩm nằm ở vòng 5 là vòng có mật độ e cao, còn phản ứng tạo
ra F là SN nên sản phẩm nằm ở vòng 7 là vòng có mật độ e thấp.

Câu 7: (Đúng mỗi chất ở câu 1 được 0,25 điểm. Đề nghị cơ chế đúng 0,75 điểm. Tổng hợp đúng 0,5
điểm. Nếu dài quá 7 bước chỉ còn 0,5 điểm)
1. Cấu trúc các chất như sau:

2. Cơ chế axyloin
3. Con đường tổng hợp
Câu 8: (Đúng thứ tự sắp xếp peptit được 0,5 điểm, ước lượng chính xác pI được 0,25. Đúng cấu trúc
đường được 0,75. Dự đoán chính xác sản phẩm 0,5)
1. Dựa vào khối lượng phân tử của các mảnh thu được có thể kết luận chúng là các di hoặc
tripeptit. Khối lượng phân tử của peptit này được tính như sau:
M = M (amino axit 1) + M (amino axit 2) + ... – 18(n – 1)
Với dipeptit thì n = 2 có thể suy ra đoạn peptit 138 sẽ là Gly – Gly và đoạn 238 sẽ là Tyr – Gly
Tuy nhiên do đầu bài không cho số lượng amino axit là bao nhiêu, chỉ có thể biết là có 4 amino
axit mà thôi nên thực tế cấu trúc của peptit này không thể kết luận được.
2. Galactose
3. Phản ứng SNAr xảy ra ở nguyên tử F gắn trên C2
NĂM 2011
Đề 1: Đại cương. Vô cơ

Bài 1: 2 điểm: 1. 1 điểm; 2. 1 điểm

1.

a) Công thức cấu tạo hợp lý nhất là:

b) Dựa vào dữ kiện thực nghiệm, ta thấy liên kết giữa 2 nguyên tử nitơ vừa mang
một phần tính chất của liên kết đôi, vừa mang một phần tính chất của liên kết ba.
Tương tự, liên kết giữa nguyên tử nitơ và nguyên tử oxy vừa mang phần tính của
liên kết đơn, vừa mang phần tính của liên kết đôi. Do đó công thức cấu tạo của
N2O phải là:

2. Thực ra khả năng tạo thành mạng tinh thể ion không chỉ phụ thuộc vào năng lượng
ion hoá và ái lực electron của các giai đoạn, mà còn phụ thuộc vào độ bền của
mạng lưới ion. Do trong Mg2+O2- các ion có điện tích lớn hơn trong Mg+O- nên
Mg2+O2- sẽ bền hơn Mg+O-;năng lượng toả ra do hình thành mạng lưới Mg2+O2 có
thể bù đắp được sự thu năng lượng trong quá trình ion hoá (còn Mg+O- thì không
được như thế) nên hợp chất giữa MgO phải được cấu tạo từ Mg2+ và O2-.

Bài 2: 2 điểm: 1. 0,5 điểm; 2. 0,75 điểm; 3. 0,75 điểm

1. 1.
1 10.105 dynes/cm
ν  1680 cm 1
2 .3.10cm/s 12 g/mol 
2

2. 12 g/mol  .(6, 02.1023 mol1 )

2. Liên kết C – D có khối lượng thu gọn lớn hơn liên kết C – H, do đó năng lượng
dao động của liên kết C – D nhỏ hơn liên kết C – H.
Khi nguyên tử hiđro của nhóm O – H tạo liên kết hiđro thì mật độ electron của liên
kết O – H giảm xuống, dẫn đến năng lượng liên kết giảm, do đó hằng số lực liên
kết giảm. Chính vì vậy liên kết O – H riêng rẽ có năng lượng dao động lớn hơn
liên kết O – H có liên kết hiđro.
3. Nguyên tử hiđro liên kết với nguyên tử carbon ở trạng thái lai hoá khác nhau thì
Csp3  H
năng lượng liên kết tương ứng sẽ khác nhau, cụ thể là năng lượng liên kết
Csp2  H Csp  H
< < .
Như vậy hiển nhiên là năng lượng dao động của 3 loại liên kết này sẽ là khác
nhau. Điều này được giải thích là do khi obitan lai hoá có % đóng góp của AO – s
càng lớn thì khả năng xen phủ với các obitan khác càng cao, dẫn đến liên kết càng
bền vững.
Bài 3: 1,5 điểm: 1. 0,25 điểm; 2. 1,25 điểm; a) 0,75 điểm; b) 0,5 điểm

1. 2NaHCO3  Na2CO3 + H2O + CO2

2. Giả sử các hàm nhiệt động không thay đổ i nhiề u trong khoảng nhiệt độ khảo sát

Ta có:  r G o =  RTlnK p =  r H o  T rSo

r Ho  So
 lnK p =  + r (*)
RT R

r Ho
́ h với 1/T với hệ số góc là 
Như vậy lnKp phụ thuộc tuyế n tin và giao điể m của
R
 r So
đồ thi ̣ với trục tung là . Vậy ta sẽ dự ng một đồ thi ̣ là đường thẳ ng có phương
R
trin
̀ h như trên:

1/T (K-1) 3,30.10-3 3,10.10-3 2,92.10-3 2,68.10-3


lnKp -11,0 -7,85 -5,07 -1,47

y = -15369x + 39,759
R² = 0,9999

lnK

r Ho
Từ đồ thi,̣ ta thu đượ c  = -15369  r Ho = 128 kJ/mol
R

 r So
= 39,759  rSo = 330 J.mol-1.K-1
R

3. Ta có: pH2O = pCO2

Theo đề bài ta có pH2O + pCO2 = 2 atm

Do đó pH2O = pCO2 = 1 atm

Suy ra Kp = pH2O . pCO2 = 1 do đó lnKp = 0

Thế vào phương trin


̀ h (*), giải ra T = 387 K.

Bài 4: 3 điểm: 1. 1 điểm; 2. 1 điểm; 3. 1 điểm

1.
Ta có: [Ag+] = 2S

[CrO42-]0 = S = [CrO42-] + [HCrO4-] + [H2CrO4] =

Suy ra S = [CrO42-] + K a12 [CrO42 ][H  ] +  K a1 .K a 2  [CrO 42 ][H  ]2


1


Do đó S = [CrO 24 ] 1 + K a12 [H  ] +  K a1.K a 2  [H  ]2
1

S
 [CrO24 ] =
1  K [H ]   K a1K a 2  [H  ]2
1  1
a2

Ta lại có: Ksp = [Ag+]2.[CrO42-], từ đó suy ra:

1 + K 
Ksp
[H  ] +  K a1.K a 2  [H  ]2
1 1
S= 3
a2
4

pH = 4,0  pH = 10-4, S = 5,92.10-4 M

pH = 5,0  pH = 10-5, S = 2,77.10-4 M

pH = 8,0  pH = 10-8, S = 8,72.10-5 M

pH = 9,0  pH = 10-9, S = 8,64.10-5 M

(tính sai ít nhất ¾ trị không cho điểm, còn nếu chỉ sai 1 cho nguyên điểm)

2.
3. - ̣ A:
Dung dich HCl → H+ + Cl-

0,1 0,1 M

[H+] = 0,1 M  pH = -log[H+] = 1.

- ̣ B:
Dung dich

Nồ ng độ đầ u của HCl quá nhỏ nên không thể loại bỏ sự ảnh hưởng của cân bằ ng
phân ly của nước. Phương trin
̀ h bảo toàn điện tić h cho ta:

Kw
[H+] = [Cl-] + [OH-]  [H  ] = [HCl]o +
[H  ]
Từ đây ta thu đượ c phương trin
̀ h bậc hai theo [H+], giải ra ta đượ c [H+] = 1,62.10-
7 M

Do đó pH = 6,79

- ̣ C:
Dung dich

CH3COOH ƒ CH 3COO  + H  Ka
0,1  x x x

Bỏ qua cân bằ ng của nước, ta có:

[CH3COO ][H  ] x2
Ka = = = 104,75
[CH3COOH] 0,1  x

 x  1,32.103

 [H  ] = x = 1,32.103  pH = 2,88

- ̣ D:
Dung dich

Nồ ng độ đầ u của CH3COOH quá nhỏ nên không thể loại trừ sự ảnh hưởng của
nước. Phương trin
̀ h bảo toàn điện tić h cho ta:

[H+] = [OH-] + [CH3COO-]

 [CH3COO ] = [H ]  [OH ]

Ta lại có: [CH3COO-] + [CH3COOH] = [CH3COOH]o

 [CH3COOH] = [CH3COOH]o – [CH3COO-] = [CH3COOH]o – ([ H+] – [OH-])

[H  ][CH3COO  ] [H  ]([H  ]  [OH  ])


Ka = =
[CH3COOH] [CH3COOH]o  ([H  ]  [OH  ])

Thay [OH-] = Kw/[H+] ta thu đượ c một phương trin


̀ h bậc ba. Giải phương trin
̀ h
bậc ba này ta ra đượ c [H+] = 1,61.10-7 do đó pH = 6,79.

Như vậy pH của một dung dich


̣ axit mạnh với nồ ng độ rấ t loãng bằ ng pH của một
̣ axit yế u với nồ ng độ rấ t loãng.
dung dich
3.
Từ bảng số liệu ta thấy thời gian bán phản ứng thay đổi khi nồng độ đầu thay đổi, chứng
tỏ đây không thể là phản ứng bậc nhất.

Từ phương trình động học tích phân bậc tổng quát khác 1:

1  1 1 
 n 1  n 1   kt
n 1  C C0 

Thay C = C0/2, t = t1/2 ta có:

2n 1  1  2n1  1 
t1/2   ln t  ln    (n  1)ln C0
(n  1).k .C0n t  (n  1).k 
1/2

Ta có bảng số liệu sau:

ln t1/2 ln C0
5,52 -3,00
6,21 -3,69
6,91 -4,38
7,60 -5,08
Dựng đồ thị, ta được:

y = -0,9985x + 2,513
R² = 1

Trong đó trục tung là trục ln C0 , trục hoành là trục ln t1/2

Từ đó ta suy ra n = 2

 2n 1  1  -1
ln   = 2,513 suy ra k = 0,080 l.mol
 (n  1).k 
Bài 5: 1,5 điểm: 1. 1,25 điểm; 2. 0,25 điểm.

1. Gọi công thức của khí là XHn. Dựa trên dữ kiện % xác định được A có công thức
đơn giản nhất là BH3. Tuy nhiên phản ứng đầu chỉ ra rằng A phải ở dạng dime.
Vậy công thức của A chính là B2H6
Vậy công thức các chất sẽ là: B – BH3; C – B3H9; D – B3H7; E – B4H10; F – B5H11
BH3 chỉ có 6e, không đủ octet nên không thể bền vững. Việc tạo dime sẽ làm cho mỗi
nguyên tử B có đủ 8e xung quanh (octet) nên bền vững
Đề 1: Hữu cơ

Bài 1: 2 điểm: 1. 1,25 điểm; 2. 0,75 điểm

1.

a) Do O có độ âm điện lớn hơn S nên tính axit của A > B.

b) Tổng hợp A

Tổng hợp B

2.

(Vòng cứng nhắc dễ sonvat hóa hơn)


CH3CO3H < CH3COOH < CH3SO3H
(dựa vào sự giải tỏa của anion)

Bài 2: 2,5 điểm: 1. 1 điểm; 2. 1 điểm; 3. 0,5 điểm

1. Sơ đồ tổng hợp
Từ 5 – 7 phản ứng cho 0,5. Quá 7 phản ứng cho 0,25

2. Cơ chế của quá trình như sau:

3. Cơ chế phản ứng với natri metylat:

R càng nhỏ thì án ngữ không gian càng ít, vì thế mà phản ứng xảy ra nhanh hơn
với etyl axetat.
Cơ chế phản ứng với MeOH/H+ có thể là 1 trong 2 sơ đồ sau:

Cacbocation bậc 3 mới bền do đó khi R là t-butyl thì phản ứng xảy ra nhanh, tạo
axit và ete như sơ đồ sau. Còn R là etyl thì việc tạo cation bậc 2 không thuận lợi,
phản ứng đi theo cơ chế ở trên tạo este và rượu.

Bài 3: 2 điểm: 1. 1 điểm; 2. 1 điểm.


1. Sơ đồ tổng hợp

Từ 7 – 10 phản ứng cho 0,5. Quá 10 phản ứng cho 0,25

2. X có cấu trúc như sau:

Nếu như mô hình hóa, ta có thể thấy khi hóa rắn cấu trúc của Y cho phép sắp
xếp sít sao hơn (có thể coi dạng thẳng) trong khi cấu trúc của X lại gấp khúc (xem hình
biểu diễn sau):

Bài 4: 2 điểm: 1. 1 điểm; 2. 1 điểm.

1. Các phản ứng tách và thế nu thuộc loại phản ứng song song và cạnh tranh
nhau. Phản ứng tách thường có năng lượng hoạt hóa cao hơn phản ứng thế.
Vì lẽ đó khi tăng nhiệt độ tỷ lệ giữa các sản phẩm tách và thế E/S cũng tăng, như
trường hợp nêu ra ở đề bài, nhiệt độ tăng sẽ làm tăng hiệu suất anken.
2.

Bài 5: 1,5 điểm: 1. 0.5 điểm; 2. 1 điểm

1. Các cấu trúc có thể có:

2. Sơ đồ tổng hợp
Đề 2: Đại cương. Vô cơ

Bài 1: 3,5 điểm (1. 0,5 điểm; 2. 0,5 điểm; 3. 1,0 điểm; 4. 1,5 điểm)
1.
14 1 14 1
7N + 0 n → 6C + 1 H
2.
C-14 bị phân hủy chậm thành N-14. Do có cân bằng tạo thành và phân hủy nên nồng độ
C-14 trong khí quyển (tồn tại dưới dạng CO2) hầu như không đổi. Thực vật hấp thụ C-14
qua CO2, động vật ăn thực vật. Do đó hàm lượng C-14 trong sinh vật sống luôn được giữ
ổn định.
3.
1 λo 5730 100
t= ln = ln ≈2700 (năm)
k λ ln 2 72
4.
Do số nguyên tử của C-11 và C-14 bằng nhau:
λo (C-14) 5730
T12 (C-11)=T12 (C-14) = =3,79.10-5 (năm)
λo (C-11) 1,51.108
Tỉ lệ độ phóng xạ giữa C-11 và C-14 sau 12 giờ:
0,5

λ(C-11) 2 365.3,79.10−5
= 1,51.108 . ≈ 2.10−3
λ(C-14) −
0,5
2 365.5730

Bài 2: 3 điểm (1. 0,75 điểm; 2. 1,5 điểm; 3. 0,75 điểm)


1.
Dạng phân tử Dạng hình học Ví dụ
AX4 Tứ diện đều CH4
AX4E Tứ diện lệch SF4
AX4E2 Vuông phẳng XeF4
(Thí sinh đưa ví dụ chính xác nhưng khác với hướng dẫn vẫn được chấp nhận)
2.
Phân tử ClF3 ứng với cấu trúc AX3E2, có dạng hình chữ T. Có 2 loại liên kết Cl-F:
liên kết ở trục và liên kết ở xích đạo.
Liên kết ở trục có độ dài 169,8 pm và liên kết ở xích đạo có độ dài 159,8 pm.
Do liên kết xích đạo nằm xa cặp điện tử tự do hơn liên kết trục nên liên kết xích
đạo có độ dài ngắn hơn liên kết trục.
3) Với cùng một phối tử thì khi tăng dần số electron d theo chiều Mn2+ (d5); Fe2+ (d6), Ni2+
(d7) thì độ bền phức tăng do lúc này bán kính ion trung tâm có xu hướng giảm làm sự tạo
phức trở nên có hiệu lực hơn.
Vậy thứ tự độ bền sẽ là: [Mn(H2O)6]2+ < [Fe(H2O)6]2+ < [Ni(H2O)6]2+

Bài 3: 5 điểm (1. 1,25 điểm; 2. 1,25 điểm; 3. 1,5 điểm; 4. 1,0 điểm)
1. Xét phản ứng:
2NO + 2H 2  N 2 + 2H 2O
n0 a a mol
n x x 0,5x x mol
nt ax ax 0,5x x mol
Tại thời điểm t, áp suất tổng của hệ là:
P = ntotRT/V = (2a – 0,5x).RT/V = 2a.RT/V – 0,5x.RT/V = P0  PN2
dP dPN2
Suy ra  =
dt dt
Mà theo định nghĩa về tốc độ phản ứng, ta có:
dP dPH2 dPN 2 dPH 2O
R =  NO   = =
2dt 2dt dt 2dt
Do đó R = –dP/dt
Dễ dàng thiết lập được R = k.p 2NO .p H2
Trong đó k = 1,73.10-6 kPa-2.s2
Biểu thức –dP/dt chưa đủ để kết luận đó chính là tốc độ của phản ứng. Ai không qua bước
chứng minh ở trên bài làm trở nên vô nghĩa, vì không hiểu gì về động học. Do đó em đề nghị
bạn nào không chứng minh thì câu này không có điểm.
Mặt khác bạn nào không chú ý đến đơn vị của hằng số tốc độ phản ứng cũng phải bị trừ
điểm thật nặng!.
2. Ro = 1,73.10-6 kPa-2.s-1.(100 kPa)2.(100 kPa) = 1,73 kPa-2.s-1
Do k tương đối nhỏ, nên trong giai đoạn đầu của phản ứng, ta có thể giả sử áp suất riêng phần
của tác chất thay đổi không đáng kể. Do đó, một cách gần đúng ta cũng có thể giả sử rằng tốc độ
phản ứng trong suốt 1s đầu của phản ứng thay đổi không đáng kể.
Do vậy, ta có: PNO = PH2 =  2R 0t = 2.(1,73 kPa 2 .s 1 ).1s = 3,46 kPa
Vì thế, PNO  PH2  (100  3, 46) kPa = 96,5 kPa
Từ đó tính được PH2O  3, 46 kPa; PN2  1,73 kPa
Thực ra cách làm chính thống của bài này là phải lấy tích phân. Tuy vậy, nhiều bạn lớp 11 vẫn
chưa được học về tích phân nên có lẽ sẽ không thể giải theo cách chính thống được. Tuy vậy,
không phải BTC muốn đánh đố các bạn mà muốn xem các bạn có thể tìm một con đường khác để
xử lý vấn đề hay không. Nếu các bạn có từng xem qua đồ thị về sự phụ thuộc của nồng độ vào thời
gian thì sẽ thấy trong giai đoạn đầu của phản ứng, đường cong gần như là một đường thẳng. Nếu
biết phân tích chỗ đó thì vẫn có thể giải bài toán này như cách làm vừa nêu. BTC xin phép không
trình bày bài giải theo cách chính thống vì nó quá phổ biến.
Xét cho cùng, giải theo cách chính thống vẫn chính xác hơn: PNO tính được là 96,7 kPa. Nhưng
nếu so sánh thì sự khác biệt này có lẽ vẫn chấp nhận được.
dP dPH2
3. Dễ dàng rút ra được:  NO =  2
= 2k.PNO .PH 2
dt dt
* p0(NO) = 200 kPa >> p0(H2) = 5,00 kPa
Do đó mô ̣t cách gầ n đúng xem như áp suấ t riêng phầ n của NO không đổ i trong quá triǹ h khảo sát
phản ứng. Vì thế ta đă ̣t kapp = 2k. p0(NO)2 = 2.1,74.10-6 kPa-2s-1.(200 kPa)2 = 0,1392 s-1
Phản ứng trở thành bâ ̣c nhấ t theo H2.
Thời gian bán phản ứng của H2: t1/2 = ln2/kapp = 4,98 s.
* p0(H2) = 200 kPa >> p0(NO) = 5,00 kPa
Do đó mô ̣t cách gầ n đúng xem như áp suấ t riêng phầ n của H2 không đổ i trong quá triǹ h khảo sát
phản ứng. Vì thế ta đă ̣t kapp = 2k. p0(H2) = 2.1,74.10-6 kPa-2s-1.(200 kPa) = 6,96.10-4 kPa-1s-1
Phản ứng trở thành bâ ̣c hai theo NO.
Thời gian bán phản ứng của NO:
1 1
t1/2 = = 1 1
= 288 s
k app .PNO (6,94 kPa .s ).(5 kPa)
4. Trong cả 3 cơ chế , sự cô ̣ng gô ̣p của các giai đoa ̣n cơ bản cho ta phương triǹ h phản ứng ban
đầ u.
Xét cơ chế 1: Áp du ̣ng nguyên lý nồ ng đô ̣ ổ n đinh ̣ cho tiể u phân trung gian hoa ̣t đô ̣ng N, ta có:
dp N k
= k1.p NO .p H2  k 2 .p N .p NO  0 suy ra p N = 1 p H2
dt k2
dp N2 k
R= = k 2 .p N .p NO = k 2 . 1 .pH2 .p NO = k1.p NO .pH2
dt k2
Biể u thức tố c đô ̣ thu đươ ̣c không phù hơ ̣p với thực nghiê ̣m.
Xét cơ chế 2:
2
Biể u thức tố c đô ̣ thu đươ ̣c là R = k1 . p NO .p H2 do đó đây có thể là mô ̣t cơ chế tố t.
Xét cơ chế 3:
Biể u thức tố c đô ̣ thu đươ ̣c là R = k 2 .pN2O2 .pH2
k
Giả sử (1) đa ̣t cân bằ ng nhanh: k1.p 2NO = k 1.p N2O5  p N2O5 = 1 .p 2NO
k 1
k
Do đó R = k 2 . 1 .p 2NO .p H2
k 1
Đă ̣t k = k2.k1/k-1 thì ta thu đươ ̣c biể u thức tố c đô ̣ phù hơ ̣p với thực nghiê ̣m.
Như vâ ̣y, cơ chế 2 và cơ chế 3 đề u thu đươ ̣c biể u thức tố c đô ̣ phù hơ ̣p thực nghiê ̣m, tuy nhiên cơ
chế 3 có vẻ hơ ̣p lý hơn vì trong cơ chế 2, giai đoa ̣n châ ̣m là mô ̣t phản ứng tam phân tử.

Bài 4: 1,5 điểm (1. 0,5 điểm; 2. 0,5 điểm; 3. 0,5 điểm)
1. Biế n thiên entanpy của cả phản ứng có thể đươ ̣c tính từ năng lươ ̣ng phân ly liên kết thông
qua phương trình sau:
∆H = tổ ng năng lươ ̣ng phá vỡ liên kết + tổ ng năng lươ ̣ng hiǹ h thành liên kết
180,6 kJ/mol = 941 kJ/mol + 498 kJ/mol – E(NO)
Từ đó suy ra E(NO) = 629 kJ/mol
2. Tương tự:

3. Trước hế t, ta phải tiń h hiê ̣u ứng nhiê ̣t của phản ứng tiń h cho mô ̣t phân tử:

Sau đó tiń h bước sóng dựa vào giá tri ̣năng lươ ̣ng này:
Như vâ ̣y ánh sáng cầ n cung cấ p có bước sóng thuô ̣c vùng tử ngoa ̣i.

Bài 5: 4,0 điểm (1. 1,0 điểm; 2. 1,0 điểm; 3. 2,0 điểm)
1. Gọi số mol của Na2CO3, Na2HPO4 và CaCO3 lần lượt là x, y, z. Số mol cực đại của mẫu là n =
x + y + z < (1,049 g)/(100 g/mol) = 0,01049 mol.
Na2CO3 + 2HCl → 2NaCl + H2O + CO2
x 2x mol
Na2HPO4 + 2HCl → 2NaCl + H3PO4
y 2y mol
CaCO3 + 2HCl → CaCl2 + H2O + CO2
z 2z mol
Lượng HCl cần thiết để hoà tan mẫu là 2(x + y + z) = 2n < 0,02098 mol < 0,03 L . 1,0 M = 0,03
mol
Vậy mẫu A tan hết trong 30,0 mL dung dịch HCl 1,0 M.
2. Khi cho K2C2O4 vào dung dịch B: Ca2+ + C2O42-  CaC2O4
Vậy dung dịch C gồm Na+ (2x + 2y) mol, K+ (lượng dư), H+ (0,03 – 2x – 2y – 2z) mol, Cl- (0,03
mol), C2O42- (lượng dư).
Sở dĩ cần phải cho lượng dư K2C2O4 là để loại hết ion Ca2+ ra khỏi dung dịch, vì trong quá trình
chuẩn độ phía sau dung dịch Ca2+ có khả năng tạo kết tủa với ion photphat làm sai lệch kết quả
chuẩn độ.
3. Khi dùng chỉ thị bromocresol xanh:
OH- + H+ → H2O
OH- + H3PO4 → H2O + H2PO4-
Số mol H3PO4 = số mol Na2HPO4 = y mol
Số mol OH-: 0,03 – 2x – 2y – 2z + y = 0,05 M . 0,0104 L . 250/10 = 0,013 mol (1)
_ Khi dùng chỉ thị thymolphtalein:
OH- + H+ → H2O
2OH- + H3PO4 → 2H2O + HPO42-
Số mol OH- = 0,03 – 2x – 2x – 2z + 2y = 0,05 M . 0,012 L . 250/10 = 0,015 mol (2)
Mặt khác, khối lượng của hỗn hợp A là 106x + 142y + 100z = 1,049 g (3)
Từ (1), (2), (3) suy ra:
+ x = 0,0025 mol suy ra m(Na2CO3) = 0,265 g
+ y = 0,002 mol suy ra m(Na2HPO4) = 0,284 g
+ z = 0,005 mol suy ra m(CaCO3) = 0,5 g.
Từ đó suy ra thành phần % về khối lượng các chất có trong mẫu bột mài là:
Na2CO3: 25,3% Na2HPO4: 27,1% CaCO3: 47,6%

Bài 6: 3,0 điểm


Công thức các chất chưa biết
A – KI
B – HIO3
C – I2O5
D – KIO3
E – HI
F + G – CuI + I2
H – AgI

Phản ứng:
2KI + KNO3 + H2SO4  I2 + KNO2 + K2SO4 + H2O
3I2 + 10HNO3  6HIO3 + 10NO + 2H2O
3I2 + 6KOH  5 KI + KIO3 + 3H2O
HIO3 + KOH  KIO3 + H2O
2HIO3  I2O5 + H2O
5CO + I2O5  5CO2 + I2
2I2 + N2H4  4HI + N2
HI + KOH  KI + H2O
4KI + 2CuSO4  2CuI + I2 + 2K2SO4
KI + AgNO3  AgI + KNO3
Đề 2: Hữu cơ

Bài 1: 4,5 điểm (1. 1,75 điểm, 2. 1,25 điểm, 3. 1,5 điểm)

1. Thứ tự phản ứng tăng dần từ trái sang phải

Giải thích: RX---(chậm)-------> R+ ---(nhanh)-----> RY. Khả năng phản ứng càng tăng nếu R+
càng bền, nghĩa là cấu trúc cacbocation tạo thành càng phẳng và điện tích (+) được giải tỏa càng
nhiều (tác dụng +C > +I >> +H).

Mặt khác khả năng phản ứng của các dẫn xuất theo thứ tự: RI > RBr > RCl (do liên kết càng dễ
bị phân cực hóa thì càng kém bền, khả năng phản ứng càng tăng)

2. Gán nhiệt độ sôi (sai một cấu trúc không cho điểm)
p-nitrophenol: 201 (liên kết hydro liên phân tử bền do H phenol linh động)
m-nitrophenol: 194 (liên kết hydro liên phân tử nhưng H phenol kém linh động)
o-nitrophenol: 116 (có liên kết hydro nội phân tử)
Dạng E: tuy đối xứng cao hơn nhưng lại có liên kết hydro nội phân tử nên t0s thấp (95)

Dạng Z: chỉ có liên kết hydro liên phân tử, tos cao (156)

3. Cấu trúc của xiclobutan và p-quinon


Mỗi chất đều có 4 electron  trong hệ vòng
Tuy nhiên xiclobutan không bền và không thể điều chế được do có cấu trúc vòng phẳng,
liên hợp kín, có 4 electron  nên là hệ phản thơm Mobius (N = 4n). Hệ này có tính bền rất kém.
Trong khi đó p-quinon tuy cũng có 4 electron  nhưng bền vững do hệ này không mang
cấu trúc liên hợp phẳng mà tồn tại ở dạng thuyền. Cũng chính vì thế mà hai liên kết đôi trong
vòng biểu hiện các tính chất riêng rẽ, và nó là một dienophile quan trọng trong các phản ứng
Diels – Alder.

Bài 2: 4,5 điểm (1. 2,0 điểm. 2. a) 1,5 điểm, b) 1,0 điểm)

1. Đầu tiên phải xác định trong bazơ mạnh, carbanion nào sẽ được tạo ra. Có 3 khả năng
trong đó cấu trúc sau bền nhất (liên kết đôi có nhiều nhóm thế nên ổn định)

Thứ hai, ở nhiệt độ thấp –100C chưa thể tách nước ngay dù có dùng bazo mạnh. Từ đó ta
có cấu trúc A, B lần lượt là:

Cơ chế phản ứng diễn ra như sau:

2. Cơ chế phản ứng diễn ra như sau:


Do có đến hai cơ chế tạo sản phẩm chính nên chỉ cần viết được một cơ chế cũng cho
điểm tối đa.
Chỉ dẫn khoa học: Hiện sự tạo thành flavanon vẫn còn là một vấn đề gây tranh cãi
nhưng cơ chế tạo thành sản phẩm phụ auron đã được thống nhất. Tuy nhiên với những ai
viết cơ chế không qua trung gian epoxy mà kết luận không có sản phẩm phụ vẫn cho
nguyên điểm

Cấu trúc các chất trong dãy chuyển hóa


Bài 3: 4,0 điểm (1. a) 1,0 điểm; b) 0,5 điểm; c) 1,0 điểm 2. 1,5 điểm)

1.
a. A là đồng phân meso  DES có cấu hình trans
Tên IUPAC: 3,4-đi(4-hidroxiphenyl)hex-3-en
(0,25 cho xác định meso, 0,5 cho xác định đúng cấu hình, và 0,25 cho đọc tên đúng)

b. Tên gọi A: 3,4-đibrom-3,4-bis(4-hidroxiphenyl)hexan


Công thức Fischer:

c. Sơ đồ tổng hợp:

2. Tổng hợp hợp chất ong chúa


Bài 4: 3,0 điểm.

Xác định
đúng cấu trúc 6 chất đầu mỗi chất 0,25 điểm. Ba chất I K L mỗi chất 0,5 điểm

Bài 5: 4,0 điểm (1. 3,0 điểm; 2. 1,0 điểm)

1. Tổng hợp cacbohydrat có mang đồng vị:

a) Tổng hợp Kiliani – Fischer với Na14CN

b) Tổng hợp Kiliani – Fischer với NaCN, rồi khử bằng T2

c) Thoái phân Ruff, sau đó Kiliani – Fischer tetrozơ nhận được bằng Na14CN, rồi lặp lại
Kiliani – Fischer một lần nữa với NaCN

Trong tiến trình tổng hợp sẽ thu được các epime như các sản phẩm phụ khó tránh khỏi

2. Sử dụng giấy điện di có pH=7, pHI (A) < 7, pHi (B) > 7, pHI (C) gần với 7 do đó trên
giấy điện di các peptit sẽ phân bố như sau:
Đề 3: Đại cương. Vô cơ

Bài 1: 2,0 điểm (1. 1,0 điểm; 2. 0,5 điểm; 3. 0,5 điểm)
1. Theo định luật bảo toàn năng lượng thì:
Năng lượng chùm tia tới Ei = công thoát  + động năng của electron E.
h 6, 626.10 34 J.s
Ta có: p =  10
 1, 767.1024 kgms 1
λ 3, 75.10 m

p2 (1, 767.1024 kgms 1 ) 2


E= = = 1,714.1018 J
2me 2.(9,109.1031 kg)

 = (4,64 eV).(1,602.10-19 C/eV) = 0,743.10-18 J


Suy ra Ei = 0,743.10-18 J + 1,714.10-18 J = 2,457.10-18 J.
E 2, 457.10 18 J
Suy ra: ν =  34
 3, 708.1015 s
h 6, 626.10 J.s
2. Số electron cực đại = số photon = (3s . 15 J/s) /(2,457.10-18 J/photon) = 1,83.1019 hạt
3. Vì năng lượng nguồn sáng nhỏ hơn công thoát electron của đồng nên dù có chiếu tia
sáng với công suất là bao nhiêu, trong bao nhiêu giờ đi nữa thì số electron thoát ra
cũng bằng 0.

Bài 2: 3 điểm (1. 2,0 điểm; 2. 0,5 điểm; 3. 0,5 điểm)


1. i) V(mọ t giọ t nước) = 300 cm3/(110 giọ t) = 0,0273 cm3/giọ t
ii) m(H2O) = 0,0273 cm3/giọ t.1,00g/cm3 = 2,73.10-2 g/giọ t
iii) n(H2O) = (2,73.10-2 g/giọ t) : (18 g/mol) = 1,52.10-3 mol/giọ t
iv) N(H2O) = 1,52.10-3 mol/giọ t . 6,022.1023 phân tử/mol = 9,15.1020 phân tử/giọ t
2. c(H2O) = (1,52.10-3 mol/giọ t) : (0,0273 cm3/giọ t . 10-3 L/cm3) = 55,7 mol/L
o o
3. đọ dà i cà n tính = 9,15.1020 phân tử/giọ t . 1,50 A . 1 m/(1010 A ) . 1 km/(103 m) =
1,37.108 km xá p xỉ 1,5.108 km

Bài 3: 4 điểm (1. 0,5 điểm; 2. 0,5 điểm; 3. 1,0 điểm; 4. 1,0 điểm; 5. 1,0 điểm)
1. C7H8(l) + 9O2(k) → 7CO2(k) + 4H2O(l)
2. DcH(C7H8, l, 298K) = 7DfH(CO2, k, 298K) + 4DfH(H2O, l, 298K) – DfH(C7H8, l, 298K)
Suy ra DfH(C7H8, l, 298K) = 7DfH(CO2, k, 298K) + 4DfH(H2O, l, 298K) – DcH(C7H8, l,
298K) = 12,5 kJ/mol.
3. Xét phản ứng: PhCH2–H(k) → PhCH2·(k) + H.(k) D = 378,4 kJ.mol-1
D = DfH( PhCH2·,k,298 K) + DfH(H., k, 298 K) – DfH( PhCH3,k,298 K)
Suy ra: DfH(PhCH2·,k,298K) = D – (DfH(H., k, 298K) – DfH( PhCH3,k,298K))
Ta có: DfH(H., k, 298 K) = ½ DatH(H2, k, 298K) = +218,0 kJ mol–1.
DfH( PhCH2·,k,298 K)= DfH(C7H8, l, 298K) + DvapH(C7H8, l, 298K) = 50,5 kJ mol–1 =
210,9 kJ/mol.
4. DvapG= DvapH(C7H8, l, 298K) – T  vapS0 = 38,0 kJ/mol – 298 K.(99,0 J.K-1.mol-1.10-3

kJ/J) = 8,50 kJ/mol > 0


Vậy về mặt nguyên tắc, trạng thái tồn tại cơ bản của toluene ở 298 K là chất lỏng.
5. PhCH2–CH2Ph (k) → 2PhCH2·(k) D=?
D = 2DfH( PhCH2·,k,298 K) – DfH(PhCH2–CH2Ph,k,298 K) = 277,9 kJ/mol.

Bài 4: 4 điểm (1. 3,0 điểm; 2. 0,75 điểm; 3. 0,25 điểm)


1.
(CH2)2O → CH4 + CO
I: a b c mmHg
t: a–x b+x c+x mmHg
t=∞: 0 a+b a+c mmHg
Dễ dàng chứng minh được P((CH2)2O) = P∞ – Pt
Do vậy, ta có bảng số liệu sau:

t (min) 10 20 40 60 100 200 ∞


P((CH2)2O) 110,74 98,21 77,23 60,73 37,54 11,22 0
(mmHg)

Bằng phương pháp thử và sai, rút ra được R = k. P((CH2)2O)


Trong đó k = 0,021 min-1.
b) t1/2 = ln 2/k, suy ra t1/2 = 57 min 45 s
Mặt khác, vì ban đầu chỉ có mỗi etylen oxit nên P∞ = 2P0((CH2)2O)
Do đó P0((CH2)2O) = P∞/2 = 124,94 mmHg.
1 P0
c) k = ln suy ra k = 0,018 min-1
t P

Bài 5: 3,0 điểm (1. 1,0 điểm; 2.0,5 điểm; 3. 1,5 điểm)
1. Theo định luật Lambert – Beer thì bề dày càng tăng, khả năng hấp thụ ánh sáng càng tăng.
Do đó khi độ cao cột dung dịch tăng, ta thấy dung dịch sẫm màu hơn (khi nhìn từ trên
miệng ống nghiệm xuống).
2. c = c’h’/h
3. Nồng độ Fe(II) trong mẫu 1 = (0,00151 M).(70 mm)/(85 mm) = 1,24.10-3 M.
Suy ra nồng độ Fe(II) trong mẫu ban đầu = (1,24.10-3 M).(50 mL)/(10 mL) = 6,23.10-3 M.
Tổng nồng độ Fe trong mẫu 2 = (0,00151 M).(80 mm)/(129 mm) = 9,36.10-4 M.
Suy ra tổng nồng độ Fe trong mẫu 2 là (9,36.10-4 M).(50 mL)/(5 mL) = 9,36.10-3 M.
Suy ra nồng độ Fe(III) trong mẫu ban đầu = (9,36.10-3 – 6,23.10-3) M = 3,13.10-3 M

Bài 6: 4 điểm
1.
A B C D E
NaCl Cl2 F2 ClF3 ClF
F G H I J
HF HClO2 HCl HClO3 ClO2
2. Các phản ứng xảy ra:
2NaCl + MnO2 + H2SO4 = Na2SO4 + MnSO4 + Cl2 + 2 H2O
2F2 + Cl2 = ClF + ClF3
ClF3 + 2H2O = 3HF + HClO2
2HClO3 + SO2 = 2ClO2 + H2SO4
Đúng 4 phản ứng cho 0,5. Sai một phản ứng trừ 0,25
3. Cân bằng

4. HClO2 < HClO3 < HCl

Giải thích đúng 0,25, sắp xếp đúng 0,25.


Đề 3: Hữu cơ

Bài 1: 3,5 điểm (1. 1,0 điểm; 2. 1,0 điểm; 3. 1,5 điểm)
1.

A < B [B vòng cứng nhắc liên hợp khó]


v= 1 105

C < D [ D gần như không có sự liên hợp]


v= 1 103
2.
Ta thấy phản ứng 1 điều kiện khắc nghiệt hơn song hiệu suất lại thấp hơn nhiều sao
với phản ứng 2  chất đầu của 1 bền (vòng liên hợp), còn chất đầu của 2 kém bền
hơn nhiều. Thực tế nó có cấu trúc sau:

Vòng không phẳng nên không tồn tại liên hợp  hệ thống kém bền.
3. Xác định đúng cấu trúc mỗi chất được 0,25. Không lập luận trừ ½ số điểm đạt được
Bài 2: 3 điểm (1. 1,0 điểm; 2. 2,0 điểm)
4. Cơ chế phản ứng:

5. Cấu trúc các chất như sau:


Bài 3: 3,5 điểm (1. 1,5 điểm; 2. 2,0 điểm)
1. Đối với benzandehit việc tạo xetal vòng 6 là thuận lợi về mặt cấu dạng do lúc này
nhóm Ph chiếm vị trí e. Trong khi đó điều này hoàn toàn không thể chấp nhận được
đối với axeton do lúc này nhóm Me chiếm vị trí a. Mặt khác việc tạo xetal vòng 5
cạnh chỉ thuận lợi khi hai nhóm OH chiếm vị trí cis. Vì thế chỉ có OH của C2 và C3
mới có thể tham gia phản ứng này.
Cấu trúc các xetal như sau (mỗi cấu trúc 0,25)

2.
i) Từ trái qua phải pKa lần lượt là 10,5 9 6 4,3 2,3
ii)

iii) Theo sơ đồ trên ta có pHi = ½ (pKa3 + pKa4) = ½ (6,0 +9,0) = 7,5

Bài 4: 4 điểm
1. Đi từ các hợp chất hữu cơ không quá 3C, điều chế axit đimetyl axetic

2. Đi từ xeten, etanol và 1 hợp chất tự chọn hãy điều chế axetyl xyclopentan
Bài 5: 6,0 điểm (1. 3,0 điểm; 2.1,0 điểm; 3. 0,5 điểm; 4. 1,5 điểm)
Đề 4: Đại cương. Vô cơ

Câu 1: 4,0 điểm (1. 1,75 điểm; 2. 1,0 điểm; 3. 1,25 điểm)
1. Khi xảy ra phân rã , nguyên tử khối không thay đổi. Khi xảy ra 1 phân rã , nguyên
tử khối thay đổi 4u. Như thế, số khối của các đồng vị con cháu phải khác số khối của
đồng vị mẹ 4nu, với n là số nguyên. Chỉ 234U thoả mãn điều kiện này với n = 1. Trong 2
đồng vị 234U, 235U, chỉ 234U là đồng vị “con, cháu” của 238U. Sự chuyển hoá từ 238U thành
234U được biểu diễn bằng các phản ứng hạt nhân sau:
238
92 U 234
90Th + α ; Th
234
90
234
91 Pa + β ; 234
91 Pa 234
92 U + β
2. Trong nước thải chứa ion SO42-. Khi đưa Ba2+ vào dung dịch sẽ xảy ra phản ứng:
SO42-(aq) + Ba2+(aq)  BaSO4(r)
Kết tủa lượng lớn của BaSO4 sẽ kéo theo sự kết tủa của RaSO4. Nếu không có kết tủa
của BaSO4, thì RaSO4 không kết tủa được, vì nồng độ Ra2+ quá nhỏ, chưa đạt đến tích
số tan.
3. Sau n chu kì bán huỷ của rađi, lượng Ra chỉ còn lại 1/2n. Hoạt độ phóng xạ chỉ còn nhỏ hơn
1
khi: 2n > 103 hay n.log 2 > 3 n > 3/0,301  10. Thời gian cần lưu giữ để lượng rađi
1000
1
trong khối chất thải còn lại nhỏ hơn lượng ban đầu là: t  10 ×1600 năm = 16000 năm.
1000
Câu 2: 2,5 điểm (1. 1,5 điểm; 2. 1,0 điểm)
1.
HCHO CO2

H2O2 COCl2
(COCl)2 CH3CN

2. M(H2O) = (2.1,008 + 15,999) g/mol = 18,02 g/mol


M(D2O) = (2.2,014 + 15,999) g/mol = 20,03 g/mol
18, 02 g 1 cm 3
Vm(H2O) = .  18, 05 cm 3 / mol
1 mol 0,9982 g

20, 03 g 1 cm 3
Vm(D2O)= .  18,11 cm 3 / mol
1 mol 1,106 g

18, 05 cm 3 1 mol 1L
V(H2O) = . . 3  2,997.10 26 L/particle
1 mol 6, 022.10 particles 10 cm 3
23

18,11 cm 3 1 mol 1L
V(D2O) = . 23
. 3 3
 3, 007.10 26 L/particle
1 mol 6, 022.10 particles 10 cm
Câu 3: 4,5 điểm (1. 0,5 điểm; 2. 0,5 điểm; 3. 3,0 điểm; 4. 0,5 điểm)
1. Chúng ta sẽ sử dụng công thức
n1E1o +n 2 E o2
Eo =
n1 +n 2
Từ biểu thức này ta nhận được Eo = 1,35V đối với quá trình Cr(V) – Cr(IV), Eo = 1,33V
đối với quá trình Cr2O72- - Cr3+, Eo = -0,90V đối với quá trình Cr2+ - Cr
2. Để cho một quá trình tự phân ly có thể xảy ra thì Eo x < Ekh, với Eox là thế khử của
quá trình oxy hóa và Ekh là thế khử của quá trình khử. Một tiểu phân sẵn sàng bị dị
phân nếu như có sự chênh lệch lớn trên giản đồ Latimer giữa bên phải so với bên
trái. Điều này dẫn đến kết qủa là Cr(V) và Cr(IV) không bền đối với phản ứng dị phân
Đối với phản ứng 3Cr2+ = 2Cr3+ + Cr thì hằng số cân bằng có thể được tính từ các giá
trị thế khử chuẩn ∆rGo =-2f( -0,90V – (-0,42V)) = 93 kJ/mol. Từ giá trị ∆rGo ta sẽ thu
được giá trị hằng số cân bằng bằng cách dùng công thức:
K = exp(-∆rGo/RT) = 5,91.10-17
3. Mối liên hệ giữa Eo(Cr(OH)3/Cr) và Eo(Cr3+/Cr)
0.059
Eo(Cr(OH)3/Cr) = Eo(Cr3+/Cr) + log[𝐶𝑟 3+ ](khi [OH-] = 1M])
3
0.059 𝑇(𝐶𝑟(𝑂𝐻)3 )
⇒ Eo(Cr(OH)3/Cr) = Eo(Cr3+/Cr) + log
3 13
0.059 𝑇(𝐶𝑟(𝑂𝐻)3 )
⇒ -1,33 = -0,74 + log
3 13

⇒ 𝑇(𝐶𝑟(𝑂𝐻)3 ) = 10−30

Ta lại có
𝐶𝑟 3+ + 3𝑂𝐻 − ⇋ 𝐶𝑟(𝑂𝐻)3 1/𝑇(𝐶𝑟(𝑂𝐻)3 )
𝐶𝑟(𝑂𝐻)3 + 𝑂𝐻 − ⇋ [𝐶𝑟(𝑂𝐻)4 ]− 𝛽
Tổ hợp 2 cân bằng trên
𝐶𝑟 3+ + 4𝑂𝐻 − ⇋ [𝐶𝑟(𝑂𝐻)4 ]− 𝛽/𝑇(𝐶𝑟(𝑂𝐻)3 )

[𝐶𝑟(𝑂𝐻) ]
⇒ [𝐶𝑟 3+ ] = [𝑂𝐻−]4 𝛽/𝑇(𝐶𝑟(𝑂𝐻)
4
(*)
3)

Xét [𝐶𝑟(𝑂𝐻)4 ]− + 3𝑒 ⇋ 𝐶𝑟 + 4𝑂𝐻 −


0.059
Eo([𝐶𝑟(𝑂𝐻)4 ]− /Cr) = Eo(Cr3+/Cr) + log[𝐶𝑟 3+ ]
3

Từ nguyên tắc thế chuẩn ta có [𝐶𝑟(𝑂𝐻)4 ] = [𝑂𝐻 − ] = 1
Từ (*) ta có
1
[𝐶𝑟 3+ ] = 𝛽/𝑇(𝐶𝑟(𝑂𝐻)
3)

Vậy
0.059 1
-1,33 = -0,74 + log 𝛽/𝑇(𝐶𝑟(𝑂𝐻) ) ⇒ 𝛽 = 1
3 3

*Độ tan :
Xét quả trình tan của Cr(OH)3
𝐶𝑟(𝑂𝐻)3 ⇋ 𝐶𝑟 3+ + 3𝑂𝐻 − 𝑇(𝐶𝑟(𝑂𝐻)3 )
𝐶𝑟(𝑂𝐻)3 + 𝑂𝐻 − ⇋ [𝐶𝑟(𝑂𝐻)4 ]− 𝛽

Vậy S = [𝐶𝑟 3+ ] + [𝐶𝑟(𝑂𝐻)4 ]
𝑇(𝐶𝑟(𝑂𝐻)3 )
= [𝑂𝐻 − ]3
+ [𝑂𝐻 − ]𝛽

𝟏𝟎−𝟑𝟎 𝟏𝟎−𝟏𝟒
Ta có hàm S = (𝟏𝟎−𝟏𝟒 )𝟑 [𝑯+ ]𝟑 + [𝑯+] = 𝒇([𝑯+ ])

*Thế :
-Ở pH=0, [𝐶𝑟 3+ ] = 1𝑀, E= Eo(Cr3+/Cr) =-0.74V
- Cr3+ bắt đầu kết tủa khi :
3𝟏𝟎 −𝟑𝟎
[𝑂𝐻 − ] = √ = 10-10 (M) => pH = 4
1

-Tủa Cr(OH)3 tan ra hết thành [𝐶𝑟(𝑂𝐻)4 ]− khi [𝐶𝑟(𝑂𝐻)4 ] = 1𝑀
1
[𝑂𝐻 − ] = =1(M)=> pH = 14
𝛽
𝛽
-Sau đó cân bằng chủ yếu là 𝐶𝑟 3+ + 4𝑂𝐻 − ⇋ [𝐶𝑟(𝑂𝐻)4 ]− 𝐾 = 𝑇(𝐶𝑟(𝑂𝐻) ) = 𝟏𝟎𝟑𝟎
3

Vậy theo chiều tăng pH ta có :


 pH từ 0 – 4 :
E= Eo(Cr3+/Cr) =-0.74V
 pH từ 4 – 14 :
0.059 𝟏𝟎−𝟑𝟎
E= -0,74 + log [𝑂𝐻 −]3
3

 pH lớn hơn 14 :
0.059 𝟏𝟎𝟑𝟎
 E= -0,74 + log [𝑂𝐻 −]4
3

Do tại pH = 14 ([OH-] = 1M), dạng tồn tại chủ yếu trong dung dịch là Cr(OH)4- nên thế
chuẩn của Cr(OH)3/Cr cũng chính là thế chuẩn của Cr(OH)4-/Cr.
1. Sản phẩm có màu vàng được xác định là natri cromat. Nó được tạo thành qua phản
ứng:
4Cl- + Cr2O72- + 6H+ = 2CrO2Cl2 + 3H2O
CrO2Cl2 + 4NaOH = Na2CrO4 + 2NaCl + 2H2O
Câu 4: 3,0 điểm (1. 1,0 điểm; 2. 2,0 điểm)
Ta có: Nồng độ H3PO4 và CaCl2 sau khi trộn lẫn là 0,5M.
1. Trước khi có kết tủa, tính pH theo nấc 1 của H3PO4  pH = 1,26. Từ đó tính được :
[Ca2+] = 0,50M ; [HPO42-] = 6,42.10-8M ; [PO43-] = 5,60.10-18
Do đó không có kết tủa CaHPO4 cũng như Ca3(PO4)2 xuất hiện.
2. Trước hết cần xét xem pH nào kết tủa trước (xét độc lập, không tính đến ảnh
hưởng lẫn nhau). Ta có:
- Để xuất hiện kết tủa CaHPO4 thì:
2 K S(CaHPO4 ) K1.K 2 .h
[HPO 4 ]  = 10-6,30  0,5. > 10-6,30
[Ca ] 2+
h + K1.h + K1.K 2 .h  K1.K 2 .K 3
3 2
 pH bắt đầu xuất hiện kết tủa CaHPO4 là 1,75
- Để xuất hiện kết tủa Ca3(PO4)2 thì :

3 KS(Ca3 (PO4 )2 )
[PO4 ]  = 10-12,55
[Ca 2+ ]3
K1.K 2 .K 3
 0,5. > 10-12,55
h + K1.h + K1.K 2 .h  K1.K 2 .K 3
3 2

 pH bắt đầu xuất hiện kết tủa Ca3(PO4)2 là 3,65


Vậy CaHPO4 kết tủa trước.
Khi Ca3(PO4)2 vừa xuất hiện thì CaHPO4 cũng vừa đạt trạng thái cân bằng, tức là ta có:
KS(Ca3 (PO4 )2 ) = [Ca2+]3.[PO43-]2

KS(CaHPO4 ) = [Ca2+].[HPO42-]
3
[PO 4 ]2 K S(Ca 3 (PO4 )2 )
 2  = 10-6,2 
[HPO 4 ] 3
K 3S(CaHPO )
4

(0,5.K1.K 2 .K 3 )2 .(h 3 + K1.h 2 + K1.K 2 .h  K1.K 2 .K 3 )


3
 106,2
(0,5.K1.K 2 .h)

 pH bắt đầu xuất hiện kết tủa Ca3(PO4)2 là 9,07.

Câu 5: 3,0 điểm (1. 1,0 điểm; 2.0,5 điểm; 3. 1,5 điểm)
1. Trong cả 2 thí nghiệm, vì [Y]0 >> [CH3X], do đó trong suốt quá trình phản ứng, nồng
độ Y thay đổi không đáng kể. Trong biểu thức R = k.[Y]m.[CH3X]n ta có quyền đặt kapp =
k.[Y]m = const.
Đối với thí nghiệm thứ nhất, ta xác định được n = 1 và kapp(1) = 0,933 h-1
Đối với thí nghiệm thứ hai, ta xác định được n = 1 và kapp(2) = 1,42 h-1
Ta có: kapp = k.[Y]m  ln kapp = ln k + mln [Y]
Từ đó ta thiết lập được hệ phương trình bậc nhất 2 ẩn là ln k và m, giải ra ta được m =
1; k = 0,300 M-1.h-1.
Do đó biểu thức tốc độ là R = (0,300 M-1.h-1)[CH3X].[Y]
2. Theo phương trình Arrhenius, ta có:
k1
R.ln
k2
Ea   103 kJ/mol
1 1

T2 T1
3. Vì Ea < D(C–X) do đó không thể tồn tại giai đoạn chậm là giai đoạn cắt đứt liên kết C–
X.
Mặt khác, dựa vào biểu thức tốc độ phản ứng ta có thể dự đoán giai đoạn chậm có thể
là giai đoạn bao gồm cả CH3X và Y.
Có 2 cơ chế có thể được đề nghị:
Cơ chế 1: chỉ 1 giai đoạn CH3X + Y  CH3Y + X.
Cơ chế 2: gồm 2 giai đoạn: CH3X + Y ƒ [CH3X,Y]  CH3Y + X.

Câu 6: (1. 1,5 điểm; 2. 0,5 điểm; 3. 1,0 điểm)


Chiều hôm đó Lestrade đến:
Anh đã tìm được thủ phạm rồi sao Holmes? Hắn là ai vậy?
Holmes từ tốn bảo: “Ngồi xuống cùng ăn cơm tối với chúng tôi đã nào. Thủ phạm đâu
thể thoát khỏi tay anh được?”
Nhìn thái độ của Lestrade tôi biết tỏng anh ta đang vô cùng nôn nóng nhưng vẫn cố
nhịn. Lúc này tự nhiên tôi nhớ lại một câu ngạn ngữ cũ: “Dục tốc bất đạt”. Tuy nhiên có
vẻ như Lestrade, và nhiều người khác nữa cố tình lờ đi điều đó
Rồi bữa ăn cũng kết thúc, Holmes dẫn Lestrade đến bộ dụng cụ thí nghiệm đã bày sẵn,
và một lần nữa anh biểu diễn thí nghiệm trở lại.
Anh nghĩ chất bột này là gì hở Lestrade, Watson?
Holmes cười mỉm trước sự im lặng của chúng tôi: “Asen đấy”
Cái gì? Không thể nào asen lại xuất hiện ngay trong ống được. Theo như tôi biết thì
phải đến 500oC arsin mới phân hủy thành các nguyên tố cơ mà.
Anh bạn Lestrade của tôi ơi, anh đúng rồi. Nhưng đấy chỉ là những kiến thức anh thu
lượm được từ sách vở mà thôi. Chẳng lẽ anh không nhận thấy rằng ở trong ống áp suất
khá thấp sao? Và với một áp suất đủ thấp cùng với một luồng khí đủ nóng thì arsin đã
phân hủy thành các nguyên tố rồi. Holmes nhẹ nhàng đáp trả với một giọng đầy mỉa
mai.
Dĩ nhiên Lestrade vẫn chưa chịu thua: Tuy nhiên các hiđrua dễ phân hủy đâu phải chỉ
có arsin, còn cả stibin nữa mà. Vậy có thể chất bột đó là antimon lắm chứ.
Một điểm cộng cho Lestrade. Holmes nói với một giọng vui vẻ. Đúng là chỉ có hai hiđrua
là arsin và stibin tồn tại ở dạng khí và dễ phân hủy thành các chất rắn mà thôi. Và chỉ từ
thí nghiệm này ta không thể rõ được chất bột màu đen là gì. Vì thế ta cần phải làm một
thí nghiệm kiểm chứng với chai nước Javel này.
Trước sự ngạc nhiên của Lestrade, chất bột màu đen tan hoàn toàn.
Holmes nói tiếp: Để phân biệt asen và antimon cách đơn giản nhất là oxy hóa chúng
bằng một hóa chất dễ kiếm chính là nước Javel. Lúc này chỉ asen tan được mà thôi,
antimon thì không.
Như vậy mọi sự đã rõ ràng: Loại thuốc bổ màu trắng chính là thạch tín nên nạn nhân tử
vong vì đã bị đầu độc bằng asen trong thời gian dài. Và kẻ duy nhất có thể thực hiện
được điều này chính là người con cả. Holmes nói như đang muốn hạ màn một vở kịch
nhiều chương hồi.
Tôi còn phải học hỏi anh nhiều, Holmes ạ. Lestrade nói
Cảm ơn sự đề cao của anh. Tôi biết anh sẽ khá hơn nếu biết tư duy một cách linh hoạt
hơn, không dựa quá nhiều vào những kiến thức sách vở và những tư duy kinh viện.
Nếu anh nghĩ kỹ hơn anh sẽ thấy rằng vết cắt trên cổ nạn nhân hoàn toàn không hề tự
nhiên, vì làm gì có kẻ sát nhân nào có thể cắt đúng vào vị trí động mạch cổ của nạn
nhân được kia chứ, ngay cả khi ông ta đang ngủ cũng rất khó xác định chính xác vị trí
của nó kia mà.
Cảm ơn anh đã cho tôi một bài học về tư duy. Lestrade lầm lũi bước ra cửa.
Các phản ứng hóa học đã xảy ra trong quá trình phân tích :
As2O3 + 6Zn + 12HCl = 2AsH3 + 6ZnCl2 + 3H2O
4AsH3 = As4 + 6H2
As4 + 10NaOCl + 6H2O = 4H3AsO4 + 10NaCl
Đề 4: Hữu cơ

Hướng dẫn chấm


Câu 1: 4,0 điểm (1. 1,25 điểm; 2. 1,25 điểm; 3. 1,50 điểm)
i) A, B, C lần lượt là:

Giải thích: sản phẩm tạo thành đều là các hệ thơm, đo đó bền vững [thỏa mãn quy tắc [4n+2]: A
có 2eπ, B có 6eπ, C có 6eπ]
ii) Duy nhất hợp chất đầu [azulen] là hợp chất thơm, nó có cấu tạo lưỡng cực và thỏa mãn
các tính chất của hệ thơm

Những chất còn lại đều có cấu trúc không phẳng do có tương tác đẩy giữa các nguyên tử hiđro
xuyên vòng.
i) Chất [a] có tính axit lớn hơn do trên thực tế nó tồn tại chủ yếu ở dạng enol [85%] nhờ có
liên kết H nội phân tử
ii) Chất [d] có tính axit mạnh hơn do hiệu ứng –I của Br sẽ manh hơn so với [c], do e ở [d]
sẽ bị dịch chuyển về Br theo 3 ngả, còn ở [c] chỉ có 1 ngả hơn nữa mạch lại dài hơn làm giảm –I
nhanh chóng.
 Điểm thưởng nếu bài làm có thêm nhận xét: Ở đây nếu nhìn toàn diện hơn sẽ thấy [d]
cứng nhắc, còn [c] có thể quay tự do trong không gian quanh các liên kết đơn là cho phân tử này
có thể tồn tại ở cấu dạng như dưới đây, rõ ràng Br có tích điện âm nên tạo ra xung quang nó một
điện trường làm cho proton khó bị tách ra, tính axit giảm [hiệu ứng trường]

Câu 2: 3,0 điểm (1. 1,5 điểm; 2. 1,5 điểm)


1. Cơ chế như sau:

2. Cơ chế như sau:

Câu 3: 5,0 điểm (1. Tổng hợp mỗi chất 1,0 điểm; 2. 2,0 điểm)

Câu 4: 5,0 điểm


4.1: Cấu trúc các chất như sau:
4.2:
1. Cơ chế:

2.Giải thích: Sản phẩm thế nguyên tử halogen vào vị trí m. Điều này hoàn toàn bình thường khi
HSbF6 là một siêu axit và nó hoàn toàn có khả năng nhường proton cho OH tạo OH2+, lúc này trở
thành nhóm hút e và phản hoạt hóa vòng benzen.
Câu 5: 3,0 điểm (1. 1,5 điểm; 2. 1,5 điểm)
1. Trong phản ứng có sự ngưng tụ tạo bazơ Schiff. Chính vì thế amoniac và amin bậc nhất
có thể tham gia phản ứng này bình thường.
Phản ứng phát huỳnh quang chỉ xảy ra được với hợp chất mang nhóm NH2 (dựa trên cấu
trúc sản phẩm thấy không còn nguyên tử H nào). Vì thế các aminoaxit chứa nhóm NH
như prolin không thể cho phản ứng này
2. Cơ chế như sau:
Đề 5: Đại cương. Vô cơ

Câu 1: 2,0 điểm (1. 1,0 điểm; 2. 1,0 điểm)


1. Năng lượng của electron ở trạng thái 1s = E1 = -13,6 eV
Năng lượng của electron ở trạng thái 3s = E3 = -13,6/9 eV = -1,5 eV
2. Sự chuyển mức năng lượng electron chỉ được cho phép khi Δl = ± 1. Do đó trước hết cần
cung cấp một năng lượng 10,2 eV để electron chuyển lên mức 2p, sau đó cung cấp tiếp 1,9 eV
để tiếp tục chuyển electron lên mức 3s.

Câu 2: 3,0 điểm (1. 2,0 điểm; 2. 0,5 điểm; 3. 0,5 điểm)
1. Các phản ứng mô tả quá trình:
P4S3 + 8O2 → P4O10 + 3SO2
2KClO3 → 2KCl + 3O2
Phản ứng tổng cộng:
3P4S3 + 16KClO3 → 16KCl + 3P4O10 + 9SO2
2. Tỉ số khối lượng giữa P4S3 và KClO3 = 3.220,06 : 16.122,55 ≈ 1 : 3
3. ΔHo = -11677,2 kJ

Câu 3: 6,0 điểm (1. 3,0 điểm; 2. 3,0 điểm)


1.
i. Các giá trị pKa của H3PO4 là 2,15; 7,20 và 12,38. Hệ đệm có pH = 7,25 cho biết đây là hỗn hợp
của KH2PO4 và K2HPO4. Dựa vào phương trình gần đúng, ta có:
[H+] = Ka2. n(KH2PO4)/n(K2HPO4)
(do trong cùng dung dịch, tỉ lệ nồng độ cũng chính là tỉ lệ số mol)
Suy ra n(KH2PO4)/n(K2HPO4) = [H+]/Ka2 = 1,122
Ta lại có n(KH2PO4) + n(K2HPO4) = 0,085 mol/L . 1,5 L = 0,1275 mol
Do đó ta tính được n(KH2PO4) = 0,06008 mol, n(K2HPO4) = 0,06742 mol. Nói cách khác, ta cần
phải sử dụng 8,18 g KH2PO4 và 11,7 g K2HPO4.
ii. Để giảm pH xuống còn 7,1 ta phải cần một acid mạnh, lúc này công thức gần đúng của chúng
ta sẽ là:
[H+] = Ka2.[n(KH2PO4) + n(H+)]/[n(K2HPO4) – n(H+)]
Giải ra, ta được n(H+) = 0,0110 mol.
Để pH tăng lên 7,4 ta phải cần một base mạnh, lúc này phương trình hệ đệm sẽ là:
[H+] = Ka2.[n(KH2PO4) – n(OH-)]/[n(K2HPO4) + n(OH-)]
Giải ra, ta được n(OH-) = 0,0108 mol.
2.
i. Cation không màu, không có kết tủa trong môi trường NaOH dư trong số các cation đã cho chỉ
có thể là Zn2+. Zn2+ còn được kiểm chứng bởi sự tạo thành phức màu tím với dithizone:

ii. Chất kết tủa vàng với Pb(NO3)2 sau khi tác dụng với H2O2 trong số các ion ở trên chỉ có thể là
CrO42- đi từ Cr3+. Vậy đây là phản ứng nhận biết Cr3+.
iii. Dung dịch này cũng tạo màu đỏ đặc trưng khi thêm dung dịch KSCN vào dung dịch, acid hoá
dung dịch màu đỏ này, ta thấy dung dịch mất màu đỏ đặc trưng cho dung dịch Fe3+.
iv. Dung tịch T tạo được kết tủa màu đỏ với dimethylglyoxime trong môi trường NH3 nhẹ là dung
dịch Ni2+.
v. Dung dịch chứa cation M tạo phức chất với NO2- ở pH acid nhẹ là dung dịch Co2+.
vi. Dung dịch tạo kết tủa với NaBiO3 là dung dịch Mn2+.

Câu 4: 3,0 điểm (1. 0,75 điểm; 2. 0,75 điểm, 3. 1,5 điểm)
Phương trình động học trong điều kiện đã cho có dạng:
d[O-2 ]
v=- =k[O-2 ]n
2dt
⇒ lg 𝑣𝑜 = 𝑛 lg 𝐶𝑜 (𝑂2− ) + lg 𝑘
1. 2. Thay dữ liệu vào phương trình trên tính được các giá trị trung bình n = 1 và k ≈ 501 s-1. Vậy
phương trình động học có dạng:
v=k[O-2 ]
với k = 501 s-1
3.
Giai đoạn (2) diễn ra nhanh vì E- là tiểu phân trung gian hoạt động. Giai đoạn (3) cũng diễn ra
nhanh vì O22- là bazơ liên hợp của H2O2 - một axit rất yếu. Do đó giai đoạn (1) quyết định tốc độ
phản ứng. Vì vậy:
d[O2 ]
v= =k1 [E][O-2 ]
dt
Mặt khác, E- là tiểu phân hoạt động nên nồng độ gần như không đổi. Do đó nồng độ E cũng gần
như không đổi. Ta gộp k1 và [E] thành k và thu được:
v=k[O-2 ]
Như vậy cơ chế đã cho phù hợp với động học phản ứng.

Câu 5: 3,0 điểm (1. 1,5 điểm; 2.1,5 điểm)


1. Các cấu trúc có thể có của phức:

2.
i. Số mol phức = 0,32 g/(266,5 g/mol) = 1,2.10-3 mol
Số mol điện tích dương của phức hay số mol H+ = 28,8 mL . 10-3 L/mL . 0,125 M =
3,6.10-3 gấp ba lần số mol phức
Vậy phức đó là [Cr(H2O)6]Cl3: Hexaquơ Crom (III) Clorua
ii. Phức hexaquơ của Cr3+ có 3 electron độc thân, của Fe3+ có 1 electron độc thân nên đều
thuận từ. Phức hexaquơ của Co3+ không có electron độc thân nên nghịch từ.
Câu 6: (1. 1,0 điểm; 2. 0,75 điểm; 3. 1,25 điểm)
1. Dựa trên thành phần phần trăm có thể suy ra được công thức của khoáng sẽ là:
Pu6Ta10Xe22Pm17(Al2)20Hg2H58 (Không cho điểm với bất kỳ ai ghi Al40)
Khoáng này không thể tồn tại trong tự nhiên do thiếu anion (chẳng hạn oxy)
Do thiếu sót về khối lượng nguyên tử nên cho nguyên điểm câu 1 cho mọi người
2. Flo sẽ thay thế các ion OH trong tinh thể. Một điển hình của kiểu thay thế này chính là quặng
floapatit.
3. V = abc x sin = 608,566 Å3 = 6,0857.10-22 cm3
Như vậy:
M 219, 46
mo    3, 6455.1022 g
N A 6, 022.1023
m 4mo
d   2,396 g.cm3
V V
Đề 5: Hữu cơ

Câu 1: 4,0 điểm (1. 2,5 điểm; 2. 1,5 điểm)


i. Chỉ dẫn khoa học: trong dung môi không hoặc ít phân cực, hằng số điện môi ε nhỏ hơn 3,
phản ứng cộng halogen vào olefin chạy theo cơ chế cộng electrophin và quy tắc cộng
trans. Tuy nhiên khi dùng dung môi phân cực với ε > 35, cơ chế sẽ có 1 vài biến đổi để
phù hợp về mặt năng lượng
Etilenglicol có ε = 37,7 do đó khi sử dụng làm dung môi, phản ứng cộng Br2 của Z và E –
stiben sẽ đi theo con đường dưới đây:

Trong dung môi phân cực, cation I chuyển thánh II vì sự solvat hóa II có lợi hơn về mặt
năng lượng. Do có sự tự quay của liên kết đơn nên cation II chuyển thánh III bền hơn [các nhóm
Ph ở xa sẽ đẩy nhau ít hơn], sau đó chuyển thánh cation IV và cùng tạo ra 1 sản phẩm
ii. Hiệu ứng không gian sẽ làm giảm sự solvat hóa của NH3+, do đó mà o – CH3C6H4NH2 có
pKa nhỏ nhất. Trong khi đó ở p–CH3C6H4NH2 do có hiệu ứng +I của CH3- sẽ làm giảm
sự liên hợp của NH2 dẫn tới pKa lớn nhất.
Ở dãy sau nhận thấy pKa tăng dần, nghĩa là tính bazơ tăng dần do hiệu ứng không gian
ngày càng tăng [vì kích thước nhóm thế ngày càng lớn] dẫn tới sự liên hợp vào vòng của
cặp e trên nguyên tử N ngày càng giảm, tức ảnh hưởng -C của vòng sẽ ngày càng kém đi.
iii. Hợp chất màu xanh tím có bản chất là hợp chất bọc, các phân tử amilozơ (polime) ở dạng
vòng xoắn như lò xo sẽ bọc lấy các phân tử iot. Liên kết trong hợp chất bọc kém bền, do
đó khi đun nóng, màu xanh tím biến mất vì hợp chất bọc bị phá hủy tái tạo amilozơ (phân
tử chủ) và iot (phân tử khách). Để nguội, màu đó lại xuất hiện vì lại sinh ra hợp chất bọc
Một điều thú vị là sử dụng hợp chất bọc, người ta có thể tách các ankan mạch thẳng ra
khỏi hỗn hợp với các ankan mạch nhánh
i. Công thức cấu tạo của 1-clobutan:

Ta phải xét cấu dạng ở cả 2 liên kết a và b. Trong đó cấu dạng bền khi nhìn theo liên kết
a là cấu dạng ở đó hai nhóm Cl và Et ở vị trí anti để tránh sự đẩy nhau giữa hai nhóm thế. Còn
cấu dạng bền khi nhìn theo liên kết b là cấu dạng ở đó hai nhóm Me và CH2Cl ở vị trí anti.

ii) Các cấu trúc có thể có: về nguyên tắc có thể có 4 đồng phân quang học như dưới đây

2 cấu trúc sau tuy vòng 6 cạnh ở dạng ghế nhưng 2 vòng 5 cạnh đều ở dạng nửa ghế nên
sức căng quá lớn và thực tế không tồn tại. Thực tế chỉ tồn tại 2 cấu trúc đầu.
Câu 2: 3,5 điểm (1. 2,0 điểm; 2. 1,5 điểm)
3. Cơ chế các phản ứng

2. Cấu trúc các chất.

Câu 3: 3,5 điểm


Câu 4: 4,5 điểm
1. Sau khi tính toán thu được công thức đơn giản của B là C10H15Br. Vậy công thức thực
nghiệm của B là (C10H15Br)n
Do tỉ lệ đồng vị trong thiên nhiên nên B không thể chứa đồng vị của H, của C mà chỉ có
thể chứa đồng vị Br79 và Br81(có hàm lượng tự nhiên là 1 : 1)
Nếu n = 2 thì trong B có 3 loại phân tử bao gồm 79 - 79, 79 - 81, 81 - 81 và không thể
thỏa mãn đề bài. Do đó n = 1
Vậy A là C10H16
Công thức cấu tạo A, B lần lượt là

Công thức lập thể của A, B là

Do Br có tính chọn lọc cao nên sản phẩm bậc III có tỷ lệ rất lớn, hơn nữa gốc tự do trung
gian có cấu trúc tứ diện càng làm thuận lợi cho việc thế Br vào C bậc III. Coi như không có sản
phẩm thế vào C bậc II trong trường hợp này vì tốc độ thế quá nhỏ.
Tính tan: cả 2 chất đều không tan trong dung môi phân cực mà chỉ tan trong các dung
môi không phân cực như CHCl3, CCl4…
Nhiệt độ nóng chảy: yếu tố quan trọng nhất là đối xứng phân tử. Về mặt này A hoàn toàn
lấn lướt so với B, do đó dĩ nhiên nhiệt độ nóng chảy của A > B.
2. A, B, C, D lần lượt là

Sở dĩ B chuyển thành C như trên là do cả tác nhân nucleophin lẫn B đều cồng kềnh gây
bất lợi lớn về mặt không gian cho tác nhân tấn công. Cơ chế phản ứng này như sau:
H không phải hợp chất bền do dễ có phản ứng đóng vòng như sau:

Thêm EtOH và HCl dễ xảy ra phản ứng sau

Cuối cùng nghiên cứu sinh không thể thu được đúng hóa chất mong muốn
Câu 5: 4,5 điểm

1. i. ngly = nala = nPhe = 2.10-3 mol, MX = 293  trong X có 1 gly, 1 ala, 1 Phe
Vậy X là ala – gly – Phe, tên gọi: Alanyl Glyxyl Phenylalanin
ii.
Đề 6: Đại cương. Vô cơ

Câu 1: 3,5 điểm (1. 0,5 điểm; 2. 0,5 điểm; 3. 1,25 điểm; 4. 0,5 điểm; 5. 0,75 điểm)
1. Cần 2 ion Fe3+.
2. n(Fe3+) = a
n(Fe2+) = b
Do đó 1 – x = a + b và 3a + 2b = 2
Từ hai phương trình này ta thu được a/b = 2x(1 – 3x)
3. Một ô mạng cơ sở của tinh thể FeO lý tưởng có chứa 4 nguyên tử Fe và 4 nguyên tử O.
Tuy nhiên, trong ô mạng cơ sở của Fe0.925O thì số nguyên tử Fe và O là 3,7 và 4.
Do đó khối lượng phân tử sẽ là: (3,7.55,85 + 4.16)/6,02.1023 = 4,49.10-22 g.
Thể tích ô mạng là: 4,49.10-22/6,02 = 7,468.10-23 cm3
Hằng số mạng a = (7,468.10-23 cm3)1/3 = 4,21.10-8 cm
Khoảng cách giữa 2 nguyên tử Fe gần nhất là: 21/2.4,21.10-8 cm/2 = 2,98.10-8 cm.
4. Khoảng cách giữa 2 nguyên tử oxy trong mạng wustite bằng với khoảng cách giữa 2 nguyên
tử Fe và bằng 2,98.10-8 cm.

Câu 2: 3,5 điểm (1. 1,0 điểm; 2. 1,0 điểm; 3. 1,5 điểm)
1. Ta có: 0 = 2 = 1, 1  2

   22
S12  S2  S1  R.ln  0 1    R.ln
 2  1.1

Ta lại có:  RT ln[ K 2 / K1 ]  T .S12    


12   0

Do đó K1/K2 = 4
2. G12   RT ln( K1 / K 2 )  12  T S12

   22
Mà S12  S2  S1  R.ln  0 1    R.ln
 2  1.1

Do đó ta rút ra được 12   RT ln( K1 / 4 K 2 )


Mà 12  ( N Ae2 ) / (4 0 r )
Do vậy dễ dàng tính được r = 0,469.10-8 cm.
3. Cấu trúc của bazơ liên hợp của C4H2O4 là:

K1/K2 < 1 chứng tỏ K2 > K1.


Điều này là do sau khi phân ly nấc thứ nhất, cấu trúc của monoanion không cho phép sự
cộng hưởng của liên kết C=C bởi vì hầu như mật độ electron của vòng đều tập trung ở gần nhóm
OH, và cấu trúc của hợp chất bị “bóp méo” (distorted). Sự phân ly của ion H+ thứ hai đã giúp cho
phân tử càng có thêm tính đối xứng và làm cho electron phân đố đồng đều trên khắp vòng, dẫn
đến dianion bền hơn monoanion rất nhiều.
Câu 3: 2,5 điểm (1. 1,0 điểm; 2. 1,5 điểm)
1. (i) nCO(g) + (2n+1)H2(g)  CnH2n+2 (g) + nH2O (g)
 r  = -160n – 42 kJ/mol

(ii) 2nCO(g) + (n+1)H2(g)  CnH2n+2(g) + nCO2(g)


 r  = -196n – 42 kJ/mol
2. a) ntot = 0,3 mol.
Đối với hệ phản ứng ban đầu: Tại cân bằng có chứa 10% H2O  n(H2O) = 0,03 mol.
Mà do n(CO) = n(H2O) nên n(CO) = 0,03 mol.
Từ đó suy ra n(CO2) = 0,2 – 0,03 = 0,17 mol
n(H2) = 0,1 – 0,03 = 0,07 mol.
ptot p
n(CO). .n(H 2O). tot
p(CO).p(H 2O) ntot ntot n(CO).n(H 2O)
Ka = = =  0,0756
p(CO2 ).p(H 2 ) ptot ptot n(CO ).n(H )
n(CO2 ). .n(H 2 ). 2 2
ntot ntot
Đối với hệ phản ứng sau: Tại cân bằng có chứa 30% H2O  n’(H2O) = 0,09 mol
 n’(H2) = 0,1 – 0,09 = 0,01 mol.
 Kb = p’(H2O)/p’(H2) = n’(H2O)/n’(H2) = 0,09/0,01 = 9
Ta lại có:
ptot p
n(CO). .n(H 2O). tot
p(CO).p(H 2O) ntot ntot n(CO).n(H 2O)
Ka = = =  0,0756
p(CO2 ).p(H 2 ) p p n(CO2 ).n(H 2 )
n(CO2 ). tot .n(H 2 ). tot
ntot ntot
Từ đó suy ra n(CO)/n(CO2) = 0,0756/9
Suy ra Kc = n(CO2)/n(CO) = 9/0,0756 = 119
b)  r H  43,3 kJ/mol
Câu 4: 4,5 điểm (1. 0,5 điểm, 2. 0,5 điểm, 3. 0,5 điểm, 4. 1,5 điểm, 5. 1,5 điểm)
1. R = k.[NO]x.[O2]y
Từ các giá trị trong bảng, xác định được x = 2, y = 1.
2. k = 12,5 (dm3)2.mol-2.s-1.

3. Ta có:

Cơ chế phản ứng này giải thích tốt cả dữ kiện nhiệt động và dữ kiện động học của phản
ứng.
Ở đây ta có thể thấy, để phản ứng xảy ra được thì NO phải kết hợp với O2 để tạo thành
NO3 rồi NO thứ hai mới tác dụng với NO3 để tạo thành sản phẩm cuối. Như vậy, nếu nhiệt độ
phản ứng tăng thì sẽ làm cho quá trình phân ly NO3 thành NO và O2 xảy ra nhanh hơn, dẫn đến
tốc độ phản ứng giảm. Do đó năng lượng hoạt hoá có giá trị âm.
Câu 5: 2,5 điểm (1. 1,25 điểm; 2. 1,25 điểm)
1. Dung dịch chỉ chứa NaHCO3 có pH = 8,34.
pH = 7< 8,34 nghĩa là trong hệ lúc này có tiểu phân H2CO3 và HCO3
Ta có:
[H  ]
α H2CO3   0,183
K a1  [H  ]
Nghĩa là Na2CO3 ban đầu đã bị chuẩn độ hoàn toàn thành NaHCO3, rồi NaHCO3 bị chuẩn
độ 18,3% để tạo thành H2CO3.
Do đó ta có: VHCl.CHCl = VA.CA + 18,3%.VA.CA = 11,83 mL.
2. Phenolphthalein chuyển màu ở pH = 9 > 8,34
Tại pH = 9 dung dịch chủ yếu gồm có ion cacbonat và bicacbonat.
Do đó:
[H  ]
α HCO = = 0,955
3
[H  ]  K a 2
Nghĩa là chỉ chuẩn độ 95,5% lượng Na2CO3 ban đầu.
Do đó sai số chuẩn độ ở đây là -4,5%.
Câu 6: 2,5 điểm (1. 2,25 điểm; 2. 0,25 điểm)
Ngoan ngoãn chịu trói đi, nếu không muốn bị bắn.
Bị bắn ư, ngài đội trưởng. Ngài dám bắn tôi à? Với loại súng chống tăng kia ? Nếu chỉ một
viên đạn đi lạc vào ba cái thùng thép kia thì cả tôi với ngài đều toi cả. Liệu ngài có sẵn sàng cho
điều đó ? 007 nói với một giọng đầy tự tin
Ngươi biết trong ba chiếc thùng kia là gì rồi sao ?
Có gì khó đoán đâu ? Chỉ cần mở được lớp mật mã thứ hai tôi cũng biết được các ngài
đang tổng hợp các hợp chất có chứa liên kết halogen – halogen và halogen – oxy. Chúng dễ nổ
vì trong đó tồn tại liên kết halogen – oxy vốn có năng lượng liên kết rất thấp. Cụ thể thì ba chất
các ngài đang tổng hợp là clo peclorat ClClO4 ; flo peclorat FClO4 và flo clorat FClO3 .
Sắc mặt của viên đội trưởng càng lúc càng xám lại, nhưng ngay lập tức ông ta lấy lại bình
tĩnh. Đội trưởng không được phép thể hiện thái độ hoảng sơ trước mặt thuộc cấp của mình. « Là
quân nhân, chết chỉ là chuyện nhỏ, miễn bí mật quốc gia không lọt được ra ngoài. »
Ồ đúng, tôi quên ngài là một quân nhân dũng cảm, thế nhưng nếu chúng ta cùng chết thì
chẳng phải phân xưởng này cũng « an giấc ngàn thu » sao ?
Ngươi, ngươi…
Ngài thấy không, cho dù ngài có uy hiếp tôi đi chăng nữa, tôi mới là người nắm giữ lợi thế.
007 nở một nụ cười mỉa mai
Ngừng lại một chút, anh lại tiếp
Rõ ràng ngài không thể bắt sống tôi, và tôi cũng chẳng thể làm gì được ngài. Vậy thì tốt
nhất là chúng ta đình chiến ở đây nhỉ ?
Viên đội trưởng gào lên : « Xông lên, bắt sống hắn đi »
Ngài vẫn tin là có thể bắt sống được tôi ? Thế thì ăn mìn đi.
007 móc trong túi ra một quả mìn, châm ngòi và quăng vào đám lính. Một đám khói dày
đặc bốc lên. Và khi khói tan đi thì điều đầu tiên đập vào mắt của đám lính là một mẩu giấy nhỏ
dán ngay trên màn hình máy chủ, với một thông điệp rất rõ ràng, dễ hiểu
Chào tạm biệt.
Đề 6: Hữu cơ

Câu 1: 5,0 điểm (1. 3,0 điểm; 2. 0,75 điểm; 3. 1,25 điểm)
1.
i. Như đã biết, momen lưỡng cực  = e.d. Do độ âm điện của F lớn hơn nên eF > eCl. Tuy nhiên
độ dài liên kết C – Cl (1,78 Å) lại lớn hơn C – F (1,38Å) nên lúc này momen lưỡng cực của
CH3Cl lại lớn hơn CH3F.
ii. Do cuban có cấu trúc đối xứng cao nên sẽ sắp xếp vào mạng tinh thể khít hơn nhiều so với n-
octan có cấu trúc zig-zag. Vì vậy nhiệt độ sôi của cuban cao hơn.
iii. Do DMSO chỉ là dung môi có khả năng nhận liên kết hiđro và ít phân cực hơn so với nước
nên khả năng solvat hóa cặp ion sẽ thấp hơn nhiều so với nước. Chính vì vậy tính axit của
phenol trong dung môi dimetyl sunfoxit sẽ thấp hơn so với dung môi nước.
2. Dựa trên các thông tin đã cho có thể kết luận được đó là xiclo-1,2,4,5-tetraol. (Không cho
điểm nếu không lập luận)
3.

Chất đầu tiên là xitozin, chất thứ hai là uraxin. Vai trò quan trọng của sự tautomer hóa này
chính là sự tham gia của các liên kết hydro của các mạch axit nucleic. Cấu trúc thơm sẽ làm
giảm số liên kết hydro đối với các cặp ADN.
Câu 2: 3,0 điểm (1. 1,5 điểm; 2. 1,5 điểm)
1. Rõ ràng trong hệ thống vòng 6 cứng nhắc rất khó để xảy ra sự tách syn, mà cũng không thể
cho rằng đồng phân trans chỉ chủ yếu tồn tại ở dạng ee làm chậm khả năng tách được. Như
vậy chỉ có thể cho rằng đồng phân cis phải đi qua một hợp chất trung gian trong đó các nhóm
thế thích hợp ở vị trí axial để tham gia phản ứng tách, và sự tạo thành trung gian này đã làm
chậm đi tốc độ của phản ứng tách loại đồng phân cis. Phản ứng xảy ra theo như sơ đồ dưới:
2. Cơ chế như sau:

Giai đoạn chuyển Y thành X là sự cộng HBr trái Markovnikov. Ở đây do nguyên tử Br gây –I
mạnh sẽ làm bất lợi cho sự hình thành cacbocation bậc cao. Mặt khác sản phẩm có tính đối
xứng cao sẽ có tính bền nhiệt động cao hơn sản phẩm ít đối xứng hơn.
Câu 3: 2,25 điểm (Đúng mỗi chất 0,25)
Cấu trúc các chất như sau:
Câu 4: 3,5 điểm (1. 2,0 điểm, 2. 1,5 điểm)
1. Cấu trúc các chất như sau

2. Sơ đồ tổng hợp như sau:

Câu 5: 2,75 điểm


Quy trình của giáo sư Slughorn

Quy trình của Hoàng tử Lai

Chỉ dẫn khoa học: Thực ra cả hai đều là quy trình sản xuất công nghiệp của hai hãng dược
khác nhau là Boost Company và BHC. Quy trình đầu tiên do Boost Company đưa ra vào đầu
những năm 50 nhưng do hiệu suất thấp và nhiều sản phẩm phụ nên đã được thay thế bằng
quy trình dưới của hãng BHC đưa ra. Quy trình này đã được giải thưởng Presidental Green
Chemistry Challenge Award năm 1997 và Kirpatrick Chemical Engineering Achievement
Award năm 2003. Hàng năm có đến hơn 35 triệu tấn Ibuprofen được sản xuất theo quy trình
xanh này.
Câu 6: 3,5 điểm (1. 1,5 điểm, 2. 0,75 điểm, 3. 1,25 điểm)
1. Sơ đồ tổng hợp đường L-glucozơ

2. Do enzym cacboxipepdaza chỉ thủy phân nhóm aminoaxit cuối mạch nên thứ tự các aminoaxit
sẽ là Leu – Phe – Gly – Trp.
3. Cấu trúc các chất trong chuỗi tổng hợp alanin như sau:
NĂM 2012
Đề 1: Đại cương. Vô cơ

Câu 1: 2,25 điểm


1. Do cùng chịu sự co lantanoid và có kiểu cấu trúc mạng tinh thể như nhau nên bán kính của
chúng cũng xấp xỉ nhau.
Do mạng tinh thể như nhau, bán kính cũng bằng nhau nên khối lượng riêng giờ đây chỉ còn phụ
thuộc vào khối lượng phân tử. Chính vì vậy (Au) = 10,5.196,97 / 107,87 = 19,17 g/cm3
2. a) Từ 238g U sinh ra 206g Pb. Vậy số g U cần để tạo 1,21 g Pb sẽ là 1,21.238/206 = 1,4g. Tức
số gam 138U ban đầu sẽ là 5,8g.
Như vậy ln(4,4/5,8) = -t.ln2/ = 1,8.109 năm.
b) 5,8g 238U sẽ có NA/M = 1,468.1022 nguyên tử.
Mà ta có v = -dN/dt = kN  v = N.ln2/ = 4,3.105 phân rã/phút.
Câu2: 5,0 điểm
1.
[Fe(CN)6]3-, ∆o lớn, trường mạnh [Fe(H2O)6]3+, ∆o nhỏ,
trường yếu

2. Với ]2-, vuông phẳng


n = 4 4thì:
[Ni(CN) [NiCl4]2-tứ diện

- Không tồn tại nguyên tố l = 0, ml = 0 và ms = 1/2


- Có 4 nguyên tố có l = 1, ml = 1 và ms = 1/2
- Có 4 nguyên tố có l = 2, ml = 3 và ms = 1/2
- Có 8 nguyên tố có l = 3, ml = 3, 5 và ms = 1/2
- Có 8 nguyên tố có l = 4, ml = 5, 7 và ms = 1/2
Vậy tổng cộng có tất cả 24 nguyên tố.
Câu3: 6,0 điểm
1. a) Để phản ứng xảy ra tự phát thì ∆G < 0. Tức là:
aSiO2
G  G o  RT ln <0
 PO 
aSi  o2 
P 
Có thể cho rằng hoạt độ của các chất rắn NGUYÊN CHẤT bằng 1. Như vậy biểu thức ∆G sẽ trở
thành:

Po
G  G o  RT ln
PO2

Theo định nghĩa áp suất riêng phần thì P(O2) = 0,2 atm. Từ đó suy ra ∆G = -846 kJ/mol. Tức phản
ứng xảy ra tự phát.
b) Phản ứng oxy hóa chỉ ngừng lại khi ∆G của phản ứng không âm. Vậy nếu giả sử phản ứng đạt
cân bằng (∆G = 0) ở 500K tính được P(O2) = 7,22.10-86 atm. Tức áp suất bảo vệ quá lớn so với
điều kiện cân bằng, có nghĩa là phản ứng vẫn xảy ra.
Một cách khác là thay tất cả các giá trị đã biết vào biểu thức tính ∆G suy ra ∆G = -731 kJ/mol.
Tức sự bảo vệ trong trường hợp này không đạt hiệu quả.
điện phân dung dịch
2.Phương trình điện phân:H2O (l)  H2 (k) + ½ O2 (k)
Ca(NO3)2

Khối lượng giảm 41,9 gam do sự điện phân và do sự kết tinh của canxi tetrahidrat quá bão hòa.
Sau khi điện phân ta có dung dịch bão hòa.
1 2
mH 2    5  12  3600  2, 24 g;
96485 2
1 32
mO2    5 12  3600  17,91 g.
96485 4
Khối lượng giảm do nước bị điện phân: 2,24 + 17,91 = 20,15 gam
Khối lượng giảm do canxi tetrahidrat kết tinh: 41,9 – 20,15 = 21,75 g Ca(NO3)24H2O, ứng với
21, 75
số mol là  0, 092 mol.
236,18

Sau khi điện phân có dung dịch bão hòa gồm 160 – 20,15 – 0,092  4  18,02 = 133,22 g H2O hòa
tan tối đa được 100 – 0,092  164,1 = 84,9 g hay 0,517 mol Ca(NO3)2.
Dung dịch bão hòa như thế được điều chế từ 0,517 mol Ca(NO3)24H2O (hay 122,105 g) trong
133,22 – 0,517  4  18,02 = 95,95 g H2O.
100 122,105
Vậy 100 g nước có thể hòa tan tối đa  127,5 gam Ca(NO3)24H2O.
95,95

3. a) Phương trình tốc độ tăng dân số có dạng dN/dt = ksN – ktN. Đây không có gì xa lạ chính là
phương trình động học bậc nhất. Tức N(t) = Noekt.
b) Lấy No = 0,5.109 để tính các giá trị k trung bình sẽ thấy mô hình Malthus hầu như khớp với thực
nghiệm
Vui một tí: Năm 2011, dân số thế giới đạt 7 tỉ. Vậy hỏi xem mô hình Malthus còn đúng cho trường
hợp này hay không. Nếu vẫn đúng hãy thử dự đoán xem năm nào Trái đất có 8 tỉ người.
Câu4: 4,0 điểm
1.
a) Với bán phản ứng: Mg2+ + 2e = Mg thì phương trình Nernst sẽ có dạng:
0, 059 0, 059
EMg 2 Mg  E o  lg CMg 2  E o  2, 43  lg 0, 02  2,38V
2 2
b) Dựa trên giản đồ Pourbaix thì cặp Mg(OH)2/Mg tồn tại ở pH = 9,37. Tức ở pH thấp hơn sẽ xảy
ra sự hòa tan hydroxit này. Như vậy ta có:
2
2  K 
K s ( Mg (OH)2 )=  Mg 2  OH     Mg 2   w   1,1.1011
 H  
  .
c) Mg(OH)2 + 2H+ + 2e = Mg + 2H2O.
Phương trình Nernst cho bán phản ứng này có dạng E = EoMg(OH)2/Mg + 0,059lg[H+] tức
E=EoMg(OH)2/Mg – 0,059pH.
Khi pH tăng một đơn vị thì thế giảm 0,059V. Vậy ở pH = 12 ta có:
E = -2,43 – 0,059(12 – 9,37) = -2,59V.
d) Ở điều kiện đã cho thì kim loại đã hoàn toàn bị chuyển thành hydroxit. Lớp hydroxit không tan
bao phủ bên ngoài sẽ ngăn kim loại không bị ăn mòn.
2. Có thể xảy ra ba phản ứng sau:
(1) H3PO4 + NaOH = NaH2PO4 + H2O
(2) NaH2PO4 + NaOH = Na2HPO4 + H2O
(3) Na2HPO4 + NaOH = Na3PO4 + H2O
Trước khi thêm NaOH thì Epin = 0,2000 + 0,059lg[H+] = 0,2000 + 0,059 lg(K1C)1/2 = 0,1078 V.
Sau khi thêm NaOH thì Epin = 0,1078 – 0,3000 = - 0,1922V.
Từ đó ta có Epin = 0,2000 + 0,059lg[H+] = - 0,1922  pH = 6,647.
Điều đó có nghĩa là sau khi thêm NaOH chỉ xảy ra hai phản ứng (1) và (2). Lúc này hệ trở thành
hệ đệm NaH2PO4/Na2HPO4.

lg  H    lg K 2  lg
 NaH 2 PO4    NaH 2 PO4   3, 64
 Na2 HPO4   Na2 HPO4 
Từ phương trình (1) suy ra nNaOH(1) = nNaH2PO4 = 0,01 mol.

Sau khi phản ứng 2 xảy ra thì nNaH2PO4(còn lại) + nNa2HPO4 = 0,01 mol (I).

nNaH2PO4(còn lại) = 3,64 nNa2HPO4 (II).

Giải hệ (I) và (II) thu được nNaH2PO4(còn lại) = 0,007845 mol.

và nNa2HPO4 = 0,02155 mol.


Như vậy tính được CNaOH = 0,12155 M.
Câu5:2,75 điểm
A – Ca3P2, B – PH3, C – Ca(OH)2, D – P4O10 (cũng cho điểm nếu ghi P2O5), E – AgNO3, F – Ag,
H – AgBr, I – Ca3N2, X – P, Y – N, Z - NH3.
Yêu cầu phải có lập luận rõ ràng và chuẩn xác, nếu chỉ đưa ra kết quả sẽ trừ nửa số điểm.
Đề 1: Hữu cơ

Câu 1: 3,0 điểm


1. a) b)

2. Xúc tác Pd/C vừa có khả năng hoạt hóa cho phản ứng dehydro hóa, vừa cho phản ứng hydro
hóa, do đó bên cạnh sản phẩm khử nối đôi C=C còn có khả năng dehydro hóa tạo thành thymol:

Phản ứng được thực hiện ở nhiệt độ phòng.


Một điều thú vị là thymol chỉ được tạo thành khi có hydro sục liên tục vào hỗn hợp Pulegon và
Pd/C. Nếu không có hydro sục vào hỗn hợp phản ứng thì kết quả vẫn chỉ là hỗn hợp giữa Pulegon
và Pd/C.
3. Etilen có hoạt tính rất thấp trong phản ứng Diels – Alder với buta-1,3-dien. Do đó, buta-1,3-
dien sẽ nhị hợp (cũng theo phản ứng Diels – Alder) để tạo thành sản phẩm sau:

Câu2: 6,0 điểm


1. a) Bước 1:

Ở bước thứ hai, phản ứng có thể là sự tấn công của cacbanion vào chính nhóm metyl ở cùng một
phân tử, cũng có thể là sự tấn công giữa hai phân tử với nhau:
Hướng 1:

Hướng 2:

b) Hỗn hợp phản ứng:

Nếu phản ứng xảy ra theo hướng 1: vì là phản ứng nội phân tử, nên hỗn hợp sản phẩm sẽ là:

Nếu phản ứng xảy ra theo hướng 2, có thể 2 phân tử A sẽ phản ứng với nhau, cũng có thể 2 phân
tử B sẽ phản ứng với nhau, hoặc cũng có thể A và B sẽ phản ứng với nhau với xác suất như nhau,
do đó hỗn hợp sản phẩm sẽ là:
Và tỉ lệ sản phẩm sẽ là 1 : 1 : 1 : 1.
c) Từ sự phân tích ở câu b, ta sẽ thấy phản ứng sẽ đi theo hướng thứ 2, nghĩa là hướng lưỡng phân
tử. Ta thấy sự tấn công của cacbanion vào nhóm metyl đi theo mô hình của phản ứng SN2 như sau:

Như vậy, để phản ứng xảy ra hiệu quả, trong trạng thái chuyển tiếp nhóm tấn công, nhóm đi ra và
trung tâm phản ứng phải cùng nằm trên một đường thẳng chứa liên kết C–L ban đầu. Cấu trúc của
chất nền ban đầu không cho phép thiết lập trạng thái chuyển tiếp như vậy do tâm phản ứng quá xa
so với cacbanion, do đó phản ứng không thể diễn ra theo hướng đơn phân tử (unimolecular) được
mà phải xảy ra theo hướng lưỡng phân tử (bimolecular).
2.
Câu hỏi được đặt ra là tại sao trong 1-etoxynaphtalen, electron lại có xu hướng tấn công vào vị trí
số 3 và không phải là vị trí số 2, 4 và nhân thơm bên cạnh?
Trước hết, nếu electron tấn công vào nhân thơm bên cạnh, sản phẩm trung gian sẽ là:

Cả anion-gốc 1 và 7 đều có thể được bền hóa bằng việc cộng hưởng với vòng benzen bên cạnh,
tuy nhiên sự cộng hưởng trong 7 không hiệu quả vì vòng benzene bên cạnh có mật độ electron cao
do ảnh hưởng của nhóm OEt.
Thứ hai, nếu electron tấn công vào vị trí số 2 và số 4 thì một công thức cộng hưởng của chất
trung gian lần lượt là:

xuất hiện sự tương tác giữa nhóm OEt và điện tích âm, không thuận lợi về mặt năng lượng.
Học sinh chỉ cần vẽ 3 bước trong cơ chế trên.

Câu3: 4,0 điểm


1.
Để sản phẩm tạo thành từ dẫn xuất trên trở thành dẫn xuất của pyridin, ta chỉ cần cho chất đó tác
dụng với một chất oxi hóa nhẹ, ví dụ như các peroxit/peraxit.
Trong Đề thi IChO 42nd, người ta sử dụng ure-hydroperoxit (UHP) để làm tác nhân oxi hóa cho
este ở trên với hiệu suất rất cao và không cần phải trải qua giai đoạn tinh chế sản phẩm. Mặt
khác, UHP là một trong những tác nhân hóa học thân thiện với môi trường, và dễ dàng điều chế
vì ure và hydroperoxit là những hóa chất thương mại rẻ tiền, do đó các nhà hóa học đang cố gắng
tận dụng tính năng của UHP để thay thế cho những peroxit/peraxit thường dùng trước đây như
mCPBA.
2.

Các đồng phân có thể có của Z:


Câu4: 3,0 điểm
1.

2.

Câu5: 3,0 điểm


1.

Do có sự chuyển đổi sang cấu dạng ít bền hơn (để đóng vòng 1,6-anhydro) nên hiệu suất phản ứng
thủy phân disaccarit ban đầu rất thấp (18%).

2. Dựa trên cấu trúc sản phẩm Edman suy ra aminoaxit đầu N là Phe.

Trypsin không cho sản phẩm cắt mạch nên Lys phải đứng cuối mạch.
BrCN cắt sau Met, mà trong hỗn hợp sản phẩm có một aminoaxit tự do nên Met sẽ phải nằm ở vị
trí áp cuối.

Chymotrypsin cắt sau Phe và Tyr.

Từ đó suy ra trật tự sắp xếp phải là: Phe – Asp – Tyr – Met – Leu – Met – Lys.
Đề 2: Đại cương. Vô cơ

Câu 1: 4,0 điểm


1. a) Các cấu trúc Lewis và điện tích hình thức:

b) Hai cấu trúc A và B


c) Góc H – N – N nằm giữa 109o và 120o còn góc NNN vào khoảng 180o
Cấu trúc thực sự của hai phân tử hydro azotua và xiclotriazen như sau:

Góc nhị diện giữa liên kết N-H và mặt phẳng NNN đo được là 74o
d) Trạng thái lai hóa của nguyên tử N:

e) 50,4 kJ/mol

2. a) n = 3417
b) Là polyme nhiệt dẻo do liên kết van der Waals giữa các mạch polyme rất yếu.
Câu 2: 3,0 điểm
1. Trong điều kiện đang xét: 2FeO(r) = 2Fe(r) + O2(k)
KP1 = PO2/Pchuẩn = 4,08.10-21

 PO   4,13.1016 
G o   RT ln  2   8,314.103 ln  5 
 390300 J = 390,3 kJ
 Pchuan   1, 013.10 
2. FeO(r) + CO(k) = Fe(r) + CO2(k)
PCO2 xCO2 0, 43
KP2     0, 754
PCO xCO 0,57

3. Phương trình phân ly CO2 ở 1000K: 2CO2(k) = 2CO(k) + O2(k)


K P1
Từ hai câu 1 và 2 ta có thể suy ra G3o  G1o  2G2o  K P 3  2
 7,16.1021
K P2

3
Nếu gọi  là độ phân ly của CO2 thì ta sẽ thu được biểu thức: K P 3  .Ptong .
 
2(1   ) 1  
2

 2

2K P3
Giả thiết  << 1 thì   3  2, 43.107
Ptong

4. Chỉ tính được nếu giả sử các đại lượng nhiệt động ∆H, ∆S không thay đổi theo nhiệt độ

Câu 3: 4,0 điểm


1. a) c = cA + cB + cC + cD = 46,1 mmol.dm-3
b) k = 0,0983 s-1
c) t1/2 = 7,05s
d) Ta có:
cB c c
v1   k1c A , v2  C  k2 c A , v3  D  k3c A
t t t
v  v1  v2  v3  kc A  k1  k2  k3  k  0,0983(1)
cB cB  0 cB k2
    0,940(2)
cC cC  0 cC k1
cB cB  0 cB k2
    4,33(3)
cD cD  0 cD k3

Từ hệ ba phương trình thu được k1 = 0,0456 s-1 , k2 = 0,0428 s-1 , k3 = 0,00989 s-1
e) Ở thời điểm t = t1/2 = 7,05s thì cA = cB + cC + cD = co/2 = 23,05 mmol.dm-3 (4)
Giải hệ phương trình (2), (3), (4) thu được cB = 10,0 mmol.dm-3 , cC = 10,7 mmol.dm-3 và cD =
2,32 mmol.dm-3
2. a) Sự phỏng định trạng thái dừng dẫn đến:
d SO3 .2 H 2 O
 k1 SO3 H 2 O  k 1 SO3 .2 H 2 O  k 2 SO3 .2 H 2 O  0
2

dt

Như vậy: k 1 SO3 .2 H 2 O  k 2 SO3 .2 H 2 O  k1 SO3 H 2 O


2

k SO3 H 2 O
2

Và do đó: SO3 .2 H 2 O  1
k 1  k 2

d H 2 SO4 
Cũng có:  k 2 SO3 .2 H 2 O và thay thế từ các kết qủa trên ta được:
dt

d H 2 SO4  k k SO3 H 2 O


2

 k 2 SO3 .2 H 2 O  1 2
dt k 1  k 2

Tuy nhiên vì k2 << k-1 nên phương trình trên sẽ được đơn giản hoá:

d H 2 SO4  k k SO3 H 2 O


2

 k 2 SO3 .2 H 2 O  1 2 = Keq.k2[SO3][H2O]2.
dt k 1  k 2

Phản ứng có bậc 3.


b) + Với phương án 1, chỉ có giai đoạn chậm là quan trọng để quyết định sự phụ thuộc của tốc độ
phản ứng theo nhiệt độ. Giai đoạn tạo phức rất bền là nguyên nhân chính khiến năng lượng hoạt
hóa âm.
Với phương án 1: k = Ae-E(B)/RT = Ae-20000/RT giảm khi nhiệt độ tăng
+ Với phương án 2: k = Ae-E(A)/RT = Ae-83600/RT tăng khi nhiệt độ tăng
+ Cho rằng các giá trị thừa số trước mũ trong phương trình Arrhenius ở cả hai phương án xấp xỉ
nhau nhưng phản ứng xảy ra nhanh ở nhiệt độ thấp trong thượng tầng khí quyển chủ yếu do sự
phụ thuộc nhiệt độ đã dẫn ra ở trên, và vì thế cơ chế 1 là hợp lý. Việc giải thích cơ chế 1 hợp lý
dựa trên các giá trị hằng số tốc độ hoàn toàn vô nghĩa. Cũng có hướng giải thích cho rằng chính
sự va chạm của các tiểu phân trong phương án 2 là nguyên nhân, nhưng thực sự vai trò chính của
năng lượng hoạt hóa âm là không thể chối cãi.
Câu 4: 4,0 điểm
1. a) Phản ứng xảy ra trong pin: Ni + 2H+ = Ni2+ + H2
RT 2
Với cặp 2H+/H2 ta có: E1  E2oH  / H  ln  H    0, 296V
2
2F
RT
Với cặp Ni2+/Ni ta có: E2  ENio 2 / Ni  ln  Ni 2   0,339V
2F
Vậy thế của pin E = E1 – E2 = 0,043V
Trong tiến trình phản ứng lượng H+ giảm còn Ni2+ tăng dẫn đến sự giảm mạnh thế
b) Catot: 2H2O – 2e = 2OH- + H2 Anot: Ni(OH)2 + 2e = Ni + 2OH-
Phản ứng xảy ra lúc này: Ni + 2H2O = Ni(OH)2 + H2
RT 2
Lúc này E1 = E1  E2oH  / H  ln  H    0,531V
2
2F
Ta lại có: Ni + 2OH = Ni(OH)2
2+ -
K = 1/Ks
Từ đẳng thức –nFE (Ni(OH)2/Ni) = RTlnK (n = 2) tính được Eo = 0,443V
o

Như vậy thế cho quá trình xảy ra ở anot sẽ là:


RT 2
E2   ENi (OH )2 / Ni  ENio 2 / Ni  ln OH    0,397V
2F
Và như vậy thế của pin lúc này Epin = E1 – E2 = -0,134V
2. Ta có:
pK = 3.52
pH = pKa + log ([A-]/[HA])
[A-]/[HA] = 10(pH-pKa)
Trong máu, pH =7.40, [A-]/[HA] = 10(7.40-3.52) = 7586
Tổng lượng ASA = 7586 +1 = 7587
Trong dạ dày, pH = 2.00, [A-]/[HA] = 10(2.00-3.52) = 3.02x10-2
Tổng lượng ASA = 1+ 3.02x10-2 = 1.03
Tỉ lệ giữa tổng lượng ASA trong máu và trong dạ dày sẽ là = 7587/1.03 = 7400
Câu 5: 3,0 điểm

A2+, B2+, C2+, D2+, E2+

Cl -

CCl2
A2+, B2+, D2+, E2+

OH -

A(OH)2
B2+, D2+, E2+
X-

BX2 D2+, E2+


Z-

EZ2 D2+
Y-
DY2
X-, Y-, Z-, Cl-, OH-

A2+

A(OH)2 X-, Y-, Z-, Cl-

D2+

DY2 X-, Z-, Cl-


B2+

BX2 Z-, Cl-


E2+

EZ2 Cl-
C2+
CCl2

Câu 6: 2,0 điểm


Các phản ứng xảy ra
ATP4- + H2O = ADP3- + HPO42- + H+
2HPO42- + 2OH- + 3CaCl2 = Ca3(PO4)2 + 6Cl- + 2H2O
Ca3(PO4)2 + CoCl2 = Co3(PO4)2 + CaCl2
3(NH4)2S + Co3(PO4)2 = 3CoS + 2(NH4)3PO4
Sau khi làm đúng như hướng dẫn trên the cabbage magazine thì một phần cơ bắp của ông già
Noel có màu sáng, phần khác lại có màu đậm. Sau nhiều kết quả phân tích thì lượng cơ màu nhạt
chiếm ưu thế hơn. Điều đó có nghĩa là lượng ATP trong các cơ ấy có nhiều do sự phân hủy quá
nhanh của ATPaza để sản sinh một lượng lớn năng lượng. Do có quá nhiều năng lượng sản sinh
trong một thời gian ngắn nên khả năng duy trì năng lượng để hoàn tất quãng đường giao quà của
ông già Noel hầu như không thể, vì vậy ”giấc mơ vẫn chỉ là giấc mơ” thôi
Đề 2: Hữu cơ

Câu 1: 4,5 điểm


1. Phản ứng tạo thành gem diol:

Tùy thuộc vào hợp chất cacbonyl khác nhau mà chiều thuận hay chiều nghịch trên cân bằng trên
sẽ chiếm ưu thế. Trong các hợp chất cacbonyl đã cho, chỉ có chất 3 và chất 5 có xu hướng tạo
thành gem diol.
Đối với chất 3, 3 nguyên tử cacbon (của nhóm cacbonyl) có mật độ điện tích dương cao, tương
tác đẩy dẫn đến kém bền, do đó nhóm C=O ở giữa sẽ có xu hướng tạo thành gem diol:

Đối với chất 5, sự tạo thành gem diol chuyển nguyên tử carbon sp2 trong vòng 3 thành nguyên tử
carbon sp3 làm giảm sức căng cho vòng 3:

2. a) Cơ chế:

Giai đoạn tốc định là giai đoạn tạo thành cacbocation. Trong giai đoạn này, không có sự tham gia
của ion bromua mà chỉ có sự đóng góp của ancol tert-butylic ban đầu, do vậy tốc độ phản ứng
không phụ thuộc vào nồng độ của anion bromua trong dung dịch.
b) Trong giai đoạn chậm, trạng thái trung gian được làm bền bởi sự solvat hoá của dung môi phân
cực. Dung môi càng phân cực thì sự solvat hoá càng tốt. Nước có độ phân cực cao hơn etanol, do
đó khi thêm etanol vào thì độ phân cực của môi trường phản ứng giảm, dẫn đến tốc độ phản ứng
giảm.
2. Đối với cis-2-bromxiclohexanol:
Đối với trans-2-bromxiclohexanol:

Câu 2: 3,5 điểm


1.

2. Hiệu suất phản ứng phụ thuộc rất nhiều vào ΔG0 của phản ứng. Trong cả 2 phản ứng đã cho, sự cắt đứt
và tạo liên kết giống nhau hoàn toàn, do đó ΔH0 của phản ứng rất gần nhau. Trong khi đó, phản ứng (1)
bao gồm 2 chất tác dụng với nhau để tạo thành 1 chất, trong khi phản ứng (2) là phản ứng nội phân tử để
tạo thành một sản phẩm duy nhất, do đó ΔS0 của phản ứng (1) nhỏ hơn nhiều so với ΔS0 của phản ứng
(2), làm ΔG02 âm hơn ΔG01, nên hiệu suất của phản ứng (2) cao hơn so với phản ứng (1).

Câu 3: 4,0 điểm


1.
b) Giữa i và ii có cân bằng chuyển hóa sau:

Dạng ii (đồng phân exo) bền hơn dạng i (đồng phân endo), do đó đồng phân ii chiếm ưu thế hơn
i tại cân bằng.
c)

Khử một nối đôi C=O thứ nhất:


Hướng phản ứng của nối đôi C=O thứ hai cũng tương tự như vậy.

d) Trong i có hệ thống carbonyl-α,β chưa no, trong khi iii có hai liên kết đôi cô lập, do vậy i có
khả năng hấp thụ kích thích trong vùng UV-Vis, còn iii chỉ có khả năng hấp thụ kích thích trong
vùng tử ngoại xa. Vì vậy, theo dữ kiện đề bài, ta dự đoán i có màu vàng nhạt, còn iii thì có màu
trắng.

Câu4: 4,0 điểm


1.

2.
Câu 5: 4,0 điểm
1.

2.
Đề 3: Đại cương. Vô cơ

Câu 1: 4,5 điểm


N2O ≠0 XeF2 =0

Cl2O ≠0 XeF4 =0

P4 =0 SF4 ≠0

B2H6 =0 Fe(C5H5)2 =0

ClF3 ≠0 CH2Cl2 ≠0

P4O10 =0 Fe(CO)5 =0

SO2 ≠0 (CN)2 =0

BF3 =0 CCl2 ≠0

NF3 ≠0 O3 ≠0

Câu 2: 4,0 điểm


1. a) Ở áp suất khí quyển thay giá trị 1 atm = 101325 Pa thu được T = 453,9 K
Vẫn cho nguyên điểm nếu HS sử dụng giá trị 105 Pa thu được T = 451,45 K
b) Nếu không xảy ra phản ứng hóa học thì độ nhớt sẽ giảm nếu tăng nhiệt độ. Theo đồ thị độ nhớt
của hệ tăng lên khi xuất hiện gốc tự do chính do mạch polyme vẫn tiếp tục phát triển. Nếu tiếp tục
gia tăng nhiệt độ thì phân tử polyme sẽ bị phá hủy làm độ nhớt giảm. Vậy đồ thị thỏa mãn sẽ có
dạng

Tm Tb
T
2. a) C2H5OH(k) ⇄ CH3CHO(k) + H2(k)
b) ΔHo298 = 69.0 kJ
ΔSo298 = 112 J/K
ΔGo298 = 35.6 kJ
KP = exp(-ΔG/RT) = 5.69.10-7
c) Cân bằng dịch chuyển hoàn toàn về phía phải (không tạo sản phẩm)
d) Không! (do V>0 nên nếu p tăng thì hiệu suất giảm)

Câu 3: 4,5 điểm


1. A
2. C
3.
d [ A]
k  [A] = [Ao] – kt
dt
Công thức tính chu kỳ bán hủy động học bậc 0 t1/2 = [Ao]/2k
4. a) Phản ứng: 2 NO2 → 2 NO + O2
b) Tính toán:

v = k.[NO2]x ⇒ log v = log k + x‧log [NO2]

log 5.4‧10-5 = log k + x‧log 0.010 và log 1.38‧10-4 = log k + x‧log 0.016
Giải phương trình trên cho 3 cặp dữ liệu thu được x = 2. Nếu thí sinh không tính đủ các cặp
cũng bị trừ 0,5

⇒ 0.4075 = x‧0.0204

x ≈ 2 tức k = v/[NO2]2
k1 = 5.40‧10-5/0.012 = 5.40‧10-1
k2 = 7.78‧10-5/0.0122 = 5.35‧10-1
k3 = 1.06‧10-4/0.0142 = 5.41‧10-1
k4 = 1.38‧10-4/0.0162 = 5.39‧10-1
k = 0,54 L/mol.s
(Nếu có ai sử dụng phương pháp giả thiết để giải thì phải giả thiết tuần tự, giả thiết bậc 1 trước,
2, rồi 3… Nếu trực tiếp giả sử bậc 2 ngay trừ ½ số điểm tối đa câu 3.4)

Câu 4: 4,0 điểm


1. CO2(k) ⇌ CO2(aq) KH
CO2(aq) + H2O ⇌ HCO3-(aq) + H+(aq) K1
Ta có : pH = -lg[H+] = -lg(K1KH.PCO2)1/2
Với pH = 5,6 thì PCO2 = 4,30.10-4 atm
Với pH = 5,4 thì PCO2 = 1,08.10-3 atm
Vậy tỉ số n = 2,5
2. 1.00 g graphit = 0.0833 mol cacbon
6 mol cacbon đối với 1 mol liti; tức 1 g graphit có thể chứa 0.0139 mol liti
Để đưa vào 1 mol liti, cần 96485 C.
Như vậy 1 g graphit có thể nạp 96485 × 0.0139 = 1340 C.
1340 C / g = 1340 A s / g = 1340 x 1000 mA × (1 / 3600) h = 372 mA h / g

Câu 5: 3,0 điểm

250.10 6 105
n  0, 010085mol  MA = 131,28 g/mol  A = Xe
8,314  298,15

B: 0,656 g Xe ứng với 5  10-3 mol Xe; 0,380 g F2 ứng với 10  10-3 mol F2, do đó tỉ lệ mol
Xe:F = 1:4  B = XeF4 (M=207 g/mol) và Xe + 2F2  XeF4.
C: 1,036 g XeF4 ứng với 5  10-3 mol XeF4 và có 5  10-3 mol O2. Vậy C là XeF6, và XeF4 +
O2F2  XeF6 + O2.
D: 1 mol D sau khi hidro hóa, hòa tan vào nước tác dụng được với 2 mol NaOH nên đã có 2 mol
HF, vậy D là XeF2, và XeF2 + H2  2HF + Xe (HF + NaOH  NaF + H2O)
Đề 3: Hữu cơ

Câu 1: 3,0 điểm


1. Ta có:

(1) + (2)  (3) do đó Ka(E) = K.Ka(Z)


Ta có: ∆G = 1,4 pK do đó K = 10∆G/1,4 = 769.
Ka(E) = K.Ka(Z) = 769.10-3,77 = 1,31.10-1  pKa(E) = 0,883
Do pKa(E) < pKa(Z) nên cặp e ở vị trí E có tính bazơ kém hơn cặp e chiếm vị trí Z
2.

H và nhóm HgOAc nằm ở vị trí đối song, do đó sự tương tác giữa MO sigma liên kết của C-H và
MO sigma* phản liên kết của N-HgOAc diễn ra thuận lợi.

H và nhóm HgOAc không đối song với nhau, do đó sự tương tác giữa MO sigma liên kết của C-
H và MO sigma* phản liên kết của N-HgOAc diễn ra không thuận lợi. Chính vì vậy mà có sự
chênh lệch lớn về tốc độ giữa 2 phản ứng trên.

Câu 2: 4,0 điểm


1. a)

b) Cấu trúc sản


phẩm phụ:

2.

Câu 3: 4,5 điểm


1. a)

b)

c)

d)

2. a)
Trừ nửa số điểm nếu phương pháp của thí sinh có thể dẫn đến sự tự aldol hóa trong các giai
đoạn trên.
b)

c)

Câu4: 4,0 điểm


1. Sự metyl hóa cho ra sản phẩm đa metyl hóa ở cả dạng enol (3 nhóm metyl) và xeto (6 nhóm
metyl).
Dẫn xuất bisunfit dễ dàng mất nước để tạo thành axit 3,5-dihydroxybenzenesunfonic

2. Sự tạo thành sản phẩm có màu sau phản ứng:

Sự tách nước sản phẩm kiểu andol cho sản phẩm có cấu trúc quinoit, vốn là hệ thống trợ màu
mạnh nên tạo sản phẩm có màu

Câu 5: 4,5 điểm


1. Cấu trúc các đường tương ứng
Không lập luận không được điểm
2. Cấu trúc các chất

Cơ chế phản ứng cuối


Đề 4: Đại cương. Vô cơ

Câu 1: 4,0 điểm


1. a) Phản ứng tổng hợp hạt nhân nguyên tố thứ 118 dẫn đến việc tạo thành 3 nơtron:
249
98 Cf  20
48
Ca  294
118118  3 0 n .
1

Sự phân rã  của hạt nhân nhận được cho hạt nhân của nguyên tố thứ 116:

118  116
294
118
290
116  24 He
b) Nguyên tố thứ 118 này hoàn tất chu kỳ 7. Nó thuộc về nhóm khí trơ (nhóm 18) và có cấu hình
electron [Rn] 5f14 6d10 7s2 7p6.
c) Năng lượng ion hóa:
Z IE, eV
18 15.8
36 14.0
54 12.1
86 10.7
Năng lượng ion hóa không phụ thuộc tuyến tính vào số hiệu nguyên tử. Tuyến tính hóa bằng cách
chuyển sang hàm logarit Z – IE cho kết quả Z = 118 có năng lượng ion hóa IE = 9.7 eV.

2. a) CO2 (M=44.0): lập phương tâm diện, 4 phân tử trong một ô mạng cơ sở:
44.0 g
4
44.0  4
  6.02  10  9 3 
23 176
( 23 27)
 4
 1.67  10 6 g / m 3
(0.56  10 m) 6.02  (0.56)  10
3
1.057  10

ρ=1.67106 g・m-3

b)
(0.20  0.10  0.050 )   dry ice
N  NA
44.0
(0.20  0.10  0.050 )  1.67  10 6
  6.02  10 23
44.0
 2.28  10 25

N=2.31025 phân tử

Câu 2: 4,0 điểm


a) Dễ dàng rút ra được x = (k2/k3)[monome].

Với các dữ kiện đề cho tính được x = 400

b) M = x.Mmonome = 52000

c) i) Axit không ảnh hưởng đến tốc độ phản ứng nên sẽ không gây ảnh hưởng đáng kể nào

ii) Thêm bazơ thì tốc độ phản ứng giảm do nó sẽ phản ứng với chất xúc tác (ức chế khơi mào)

HO- + HBF4-BF4+ H2O

iii) Thêm chất điện ly sẽ giúp tăng khả năng tạo thành cation do đó tốc độ phản ứng tăng

Câu 3: 3,5 điểm


a) ∆Go = 158031 J  Kp = 1,48.10-25

pBr  K p  3,85.1013 bar

b) Cho rằng các khí là lý tưởng thì nBr = 1,39.10-14 mol  NBr = 8,37.109 nguyên tử
Tương tự tính được NBr2 = 2,18.1022 phân tử
Vậy độ phân ly  = 1,92.10-11 %
1, 0 H  1 1 
   H  31710 J/mol
R  282, 45 332,35 

c) ln
0,132

Câu 4: 4,5 điểm


1. a) Eo1 = 1.67V
b) E = 1,43V
c) Fe(OH)2(r) + 2e = Fe(r) + 2OH-(aq) E3 = -0,88V
Fe2+(aq) + 2e = Fe(r) E1 = -0,44V
Vậy Fe(OH)2(r) = Fe2+(aq) + 2OH-(aq) có E = -0,44V
Ta có ∆Go = -nFE = -RTlnKsp Ksp = 1,3.10-15
d) Khi phủ thiếc lên lớp sắt thì thiếc sẽ bị oxy hóa tạo thành lớp oxit bền bảo vệ bề mặt. Tuy nhiên
nếu lớp thiếc bị xây xát thì sẽ xảy ra phản ứng tự phát sau: Fe + Sn 2+ = Fe2+ + Sn. Trong đó Fe
đóng vai trò cực âm sẽ bị ăn mòn nhanh chóng
2. Từ khoảng chuyển màu chỉ thị cho thấy phản ứng chuẩn độ sẽ kết thúc với pH = 8 – 10
(phenolphtalein) và pH = 3 – 4 (metyl da cam). Như vậy sự chuẩn độ với chỉ thị phenolphtalein sẽ
cho sản phẩm hydrocacbonat từ cacbonat (phản ứng 1) và với metyl da cam cả cacbonat và
hydrocacbonat đều chuyển hết thành cacbon dioxit (xảy ra cả phản ứng 1 và 2)
CO32- + H+ = HCO3- (1)
HCO3- + H+ = H2O + CO2 (2)
Gọi x, y lần lượt là số mol của CO32- và HCO3-. Để chuyển hết cacbonat thành CO2 cần 2x mol
axit trong khi chuyển thành hydrocacbonat chỉ cần x mol axit. Lưu ý để chuyển cacbonat thành
hydrocacbonat cần lượng axit ít gấp 3 lần so với việc chuyển toàn bộ cả hai ion thành CO2. Vậy ta
có:
3x = 2x + y tức y = x,
Có nghĩa dung dịch đầu chứa lượng bằng nhau của cả hai ion

Câu 5: 4,0 điểm


Sau khi nghe Holmes chơi hết bản nhạc, bác sĩ Watson lại một lần nữa nhìn đăm đăm vào bức thư
và bảng mã, nhưng vẫn chẳng sáng hơn được chút nào. Mọi thứ vẫn cứ mờ mờ ảo ảo cũng như
chính sự tồn tại của Nicholas Flamel trong lịch sử. Ông là nhân vật có thật, nhưng việc ông chế
tạo ra được hòn đá phù thủy có đúng hay không lại phụ thuộc vào bảng mật mã trong bức thư.

“Holmes này, lại một lần nữa tôi phải nhờ anh vén giúp tôi màn sương mù đang che phủ dày đặc
trong tôi nữa thôi”

“Vậy thì Watson, lấy hộ tôi hai cái khẩu trang và đôi găng tay, chúng ta sẽ làm một số phản ứng
hóa học đơn giản” Holmes nói, và quày quà đi vào góc tường, nơi chất đống các chai lọ để Holmes
tiến hành các phân tích Hóa học của mình

Giờ tôi sẽ trộn các chất vào nhau xem điều gì sẽ xảy ra. Watson nhớ mang khẩu trang vào nhé, sẽ
có vài chất có mùi khó chịu đấy. Cậu cũng chuẩn bị sẵn giấy bút để ghi lại hiện tượng nhé
Đầu tiên hòa tan các chất này vào nước, sau đó đổ cái này vào cái này…

Sau khi Holmes thực hiện xong các thí nghiệm, ông nhìn các ghi chép của bác sĩ Watson: “Cậu
vẫn luôn ghi chép mọi thứ thật tỉ mỉ, Watson ạ. Giờ hẳn cậu đã nhìn ra rồi chứ”

NaOH HNO3 K2S AlCl3 CuSO4 Hiện tượng

NaOH X - -   2- 2

HNO3 - X  - - 3- 1

K2S -  X   1- 1 1 1

AlCl3 * -  X - 1* 2- 1

CuSO4   - X 2- 2

Giờ hãy so sánh kết quả thu được với bảng mật mã chúng ta đang có, trong đó chú ý đến hai ký
hiệu ● và ☼ hoàn toàn đối nghịch nhau, chắc hẳn một ký hiệu ám chỉ “xuất hiện kết tủa” và một
ký hiệu ám chỉ “không có hiện tượng”. Tức dòng thứ 3 và 5 ám chỉ 2 chất NaOH và CuSO4, vì chỉ
có nó cho phép xuất hiện hai kết tủa. Còn dòng nào cho chất nào thì cũng chẳng quan trọng lắm
đâu.

Vậy cuối cùng các ký hiệu của chúng ta sẽ là ☼: Không hiện tượng ; ● : Kết tủa ; ►: khí thoát ra
, ╫: vừa cho khí vừa cho kết tủa và ◙: cho kết tủa rồi sau đó tan đi

Thế hóa ra đây không phải là cách thức điều chế hòn đá phù thủy sao? Bác sĩ Watson thốt lên đầy
chán nản

Hẳn nhiên rồi Watson ạ. Người bạn của chúng ta có lẽ đã quá mê muội với mong ước được
trường sinh mất rồi. Mặt khác tôi cũng không nghĩ đây là tác phẩm của Nicholas Flamel, kiểu
chữ tượng hình cổ này thịnh hành hơn ở Arab thế kỷ thứ VI – VII nên phần nhiều nó là tác phẩm
của một nhà giả kim thuật Arab được một dân mê sưu tầm mua lại mà thôi. Cuối cùng đây vẫn
chỉ là một bảng nhận biết các hóa chất chứ không phải là một điều gì đó bí ẩn hay sâu xa cả. Hy
vọng người bạn của chúng ta sẽ không quá thất vọng sau khi nhận được kết quả giải mã.
Đề 4: Hữu cơ

Câu 1: 4,0 điểm


a) Ở cặp thứ nhất, do phân tử 3,5-dinitropyridin có hai nhóm NO2 hút e mạnh làm giảm tính
bazơ nên 3,4-dimetylpyridin có tính bazơ mạnh hơn
Ở cặp thứ hai do tương tác không gian với hai nhóm metyl hai bên thì nhóm NO2 trong
3,5-dimetyl-4-nitroanilin đã bị lệch phần nào ra khỏi mặt phẳng vòng thơm, do đó nó
không còn gây ảnh hưởng -C mạnh như trong trường hợp 4-nitroanilin nên nó sẽ là phân
tử vào vòng sau
Ở cặp thứ ba nguyên tử N trong phtalimit không còn thể hiện tính bazơ do chịu ảnh
hưởng –C mạnh của hai nhóm cacbonyl hai bên nên sẽ có tính bazơ yếu hơn (Hoặc có thể
giải thích dựa trên công thức hỗ biến tạo hệ thống thơm isoindole-1,3-diol)
Ở cặp thứ tư thì rõ ràng amin có tính bazơ mạnh hơn nitrin.
b) Các cặp phân tử ở vòng ½:

Ở cặp thứ nhất phân tử 3,5-dimetyl-4-nitroanilin có khả năng thế electrophin tốt hơn do
lúc này ảnh hưởng –C của nhóm NO2 giảm, mặt khác vòng có nhóm thế amin +C mạnh
làm khả năng tham gia phản ứng thế SE tốt hơn so với 3,4-dimetylpyridin
Ở cặp thứ hai rõ ràng nhóm CF3 gây –H làm giảm mạnh mật độ electron trên vòng nên
khả năng thế electrophin kém
c) Trong phân tử 3,5-dimetyl-4-nitroanilin vẫn tồn tại hiệu ứng cộng hưởng cho dù đã bị
giảm đi phần nào bởi hai nhóm metyl. Tuy nhiên khi so với phân tử còn lại có các vectơ
momen lưỡng cực định hướng không cùng phương thì rõ ràng 3,5-dimetyl-4-nitroanilin
có giá trị momen lưỡng cực lớn hơn.

Câu 2: 4,0 điểm


1. Cơ chế:

2. Phản ứng cộng BH3 vào anken thuận nghịch tỏa nhiệt. Nếu phản ứng được tiến hành trong
thời gian dài thì boran sẽ “bước” đến cuối mạch sau một chuỗi các phản ứng cộng – tách –
cộng… để tạo sản phẩm bền nhất chính là dẫn xuất ankylboran đầu mạch, và chính sản phẩm này
sẽ chuyển thành ancol tương ứng. Toàn quá trình có thể được biểu diễn bằng sơ đồ như sau:
Câu 3: 6,0 điểm

Câu 4: 3,0 điểm


1. a) Có hai đơn vị kiểu C(sp) – C(sp) khả dĩ gồm: =C=C= và -CC- và có thể gắn thêm các đơn
vị này vào vòng benzen theo hai cách. Một là gắn vào giữa hai nguyên tử cacbon, và khả năng
thứ hai là gắn vào giữa liên kết C – H. Như vậy có hai cấu trúc thỏa mãn:
b) Cacbo – benzen này có 6 nguyên tử C lai hóa sp2 và 12 nguyên tử C lai hóa sp. Mỗi nguyên tử
C sp2đóng góp 1 electron , còn C sp đóng góp 2 electron . Như vậy phân tử có tổng số 30
electron  . Một cách tính khác dựa trên độ bất bão hòa của phân tử là 16, mà cacbo – benzen có
một vòng, tức còn lại 15 độ bất bão hòa ứng với 30 electron 
2. Sự tách N-amin oxit (tách Cope) là phản ứng tách đồng bộ theo cơ chế tách syn, tức nó chỉ có
thể lấy được H ở vị trí syn đối với nó. Tức với đồng phân syn chỉ có nguyên tử H gắn trên cacbon
xa nhóm phenyl thỏa mãn yêu cầu. Trong khi đó với đồng phân anti thì cả hai nguyên tử H kế
cận với nhóm amin oxit đều có thể lấy được nên thu được hỗn hợp hai sản phẩm, và sản phẩm
chính là anken bền hơn.

Phản ứng Diels – Alder trong môi trường nước được tăng tốc chủ yếu do sức căng bề mặt khiến
các phân tử hữu cơ vốn ky nước tiến đến gần nhau hơn khi hình thành trạng thái chuyển tiếp

Câu 5: 3,0 điểm


1.a) Đun sôi lâu trong môi trường axit chính để thủy phân liên kết peptit thành các đoạn
aminoaxit riêng phần. Việc thêm natri axetat vào hệ chính tạo môi trường đệm với pH cần thiết
để cystin kết tinh dưới dạng ion lưỡng cực
b) Trong quá trình này không thể thu được cystein vì hai lý do: Trong tóc cystein tồn tại chủ yếu
ở dạng cystin, mà liên kết disunfua S-S lại bền trong môi trường axit. Lý do thứ hai là nếu cho
rằng có sự hình thành cystein thì ngay khi tiếp xúc với oxy không khí nó đã bị oxy hóa thành
cystin.
2. Vai trò chính trong sự loại bỏ độc tính của ba chất trên trong glutathiol (G-SH) do nhóm thiol
SH. Nhóm này sẽ phản ứng với cả ba chất để chuyển thành dẫn xuất không độc.
3. Sơ đồ tổng hợp tripeptit Ala – Val – Phe
Đề 5: Đại cương. Vô cơ

Câu 1: 4,0 điểm


Vị trí 1: Liên kết hydro (chuỗi xoắn kép protein). Có thể phá vỡ bằng cách điều chỉnh pH, sự
thêm H+ sẽ bẻ gãy liên kết này
Vị trí 2: Tương tác Van der waals giữa các gốc kị nước. Phá vỡ liên kết bằng cách thêm vào các
chất tẩy rửa, trong đó đầu ưa nước sẽ hướng ra bên ngoài, còn đầu kị nước sẽ tương tác với các
gốc kị nước ở vị trí 2.
Vị trí 3: Liên kết cầu disunfua. Có thể bị phá vỡ dưới ảnh hưởng của chất oxy hóa (chẳng hạn
RCO3H)
Vị trí 4: Liên kết tĩnh điện. Dễ dàng bị phá vỡ bằng cách thêm muối có khả năng điện ly

Câu 2: 5,0 điểm


1. N2(k) + 3H2(k) → 2NH3(k)
2. ∆Ho = -91,8kJ.mol-1
∆So = -198,1J.mol-1.K-1.
∆Go = -32,7kJ.mol-1.
3. Amoniac sẽ hình thành ngay lập tức nhưng do năng lượng hoạt hóa cao nên hai chất khí
không thể phản ứng được với nhau. Tốc độ của phản ứng này rất thấp.
4. Sử dụng phương trình Kirchoff để tính entanpy và entropy ở 800K và 1300K ta thu được các
kết qủa sau:
N2(k) H2(k) NH3(k)
∆fH(800K) 15,1kJ.mol-1 14,7kJ.mol-1 -24,1kJ.mol-1
∆fH(1300K) 31,5kJ.mol-1 29,9kJ.mol-1 4,4kJ.mol-1
S(800) 220,6J(mol.K)-1 252,9J(mol.K)-1 236,4J(mol.K)-1
S(1300K) 236,9J(mol.K)-1 174,5J(mol.K)-1 266,2J(mol.K)-1

∆fG(800K) = 72,9kJ.mol-1.
∆fG(1300K) = 184,0kJ.mol-1.
5. Hằng số cân bằng có thể tính được từ biến thiên năng lượng Gibbs bởi biểu thức: Ks(T) =
exp(-∆G(RT)-1).
Ta có các kết qủa tính hằng số cân bằng sau:
Ks(298,15K) = 5,36.105.
Ks(800K) = 1,74.10-5.
Ks(1300K) = 4,04.10-8.
2
x NH
Sử dụng biểu thức: K X  3
; x H 2  3 x N 2 ; x N 2  x H 2  x NH 3  1
x H3 2 .x N 2

(1  4 x N 2 ) 2 4 1
KX  4
 x N2 2  x N2  0
27 x N2 27 K X 27 K x
Chúng ta nhận được biểu thức:
2 4 1
 x N2    
27 K x 27 K x 27 K x

Các kết qủa thu được cho ở bảng sau:


T(K) x(N2) x(H2) X(NH3)
298,15 0,01570 0,04710 0,03720
800 0,24966 0,74898 0,00136
1300 0,24998 0,74994 0,00008
6. Chất xúc tác làm giảm năng lượng hoạt hoá của phản ứng và làm tăng tốc độ phản ứng. Các
giá trị nhiệt động không hề thay đổi.
7. Áp suất cao dẫn đến kết qủa là phần mol của NH3 tăng lên do Kx=Kp.p2 tăng lên. Sự tăng áp
suất làm cân bằng chuyển dịch về phía tạo thành sản phẩm nhưng không làm thay đổi tốc độ
phản ứng.
8. Điều kiện tốt nhất của phản ứng là: áp suất cao, nhiệt độ càng thấp càng tốt và có mặt chất xúc
tác. Nhiệt độ cũng phải tối ưu để chuyển hóa nhanh và hiệu suất có thể chấp nhận được

Câu 3: 4,0 điểm


a) Phương trình động học trong hệ kín:
d [P]
= k[A][P]
dt
Lưu ý đến sự bảo toàn khối lượng [A] + [P] = [A]0 + [P]0, ta có:
d [P]
= k [A]0  [P]0  [P][P]
dt
Ở giai đoạn đầu thì tốc độ tạo thành P tăng lên, nhưng sau vài giai đoạn tích tụ sản phẩm thì tốc
độ tạo thành sản phẩm sẽ chậm dần và cuối cùng đạt mức zero.

[P]

b) Phương trình động học trong hệ mở.


d [P]
= k[A]0 [P]
dt
Do A luôn không đổi nên phản ứng có động học giả bậc nhất. Lúc này cả vận tốc phản ứng và
nồng độ [P] tăng theo thời gian:
[P] = [P]0 exp  k[A]0 t 

[P]

t
Câu 4: 4,0 điểm
a) 0,03803M
b) 49,95%
RT m m
c) x   ln  H    0, 414
nF n
 
K1 K1 K 2 
d) CH 2 A   H 2 A   HA    A2    H 2 A 1  
 H   H  2 
     
2
CH 2 A  H  
  H 2 A  2
 H     H   K1  K1K 2

 X   E o '  0, 062V
e) E  E o 
RT
nF  2

ln  H     H   K1  K1K 2 
RT
ln
nF  H 2 A

Câu 5: 3,0 điểm


Theo đề bài A hẳn phải là một hydrua, tức phản ứng giữa A và nước sẽ sinh ra hydro (khí B). Gọi
M là nguyên tố chưa biết thì ta có 1,0 / (1,0 + M) > 0,1, tức M < 9,0.
Do hydrua là chất rắn nên chỉ có hai khả năng là Li và Be
Phản ứng tạo thành sản phẩm duy nhất C có thể được viết dưới dạng: MHx + aCO2 = MHx.aCO2
32a
Theo đề bài ta có:  0, 616  M  7,94a  x
x  M  44a
Nghiệm duy nhất có thể chấp nhận là a = x = 1, tức M = 6,94 (Li)
Vậy các chất lần lượt là: A. LiH, B. H2, C. HCOOLi (LiH.CO2) , D. HCOOH và E. CO
Các phản ứng xảy ra
2LiH + 2H2O = 2LiOH + H2
LiH + CO2 = HCOOLi
2HCOOLi + H2SO4 = Li2SO4 + 2HCOOH
HCOOLi + H2SO4 = LiHSO4 + CO + H2O
Đề 5: Hữu cơ

Câu 1: 4,0 điểm


1.

Phản ứng
xảy ra theo
sơ đồ như sau:

Khi X = OR: giai đoạn trung gian carbanion không được bền hoá, ngược lại đôi điện tử trên
nguyên tử carbon tham gia tương tác đẩy với các đôi điện tử tự do trên nguyên tử oxygen làm
mất tính bền của carbanion.
Khi X = SR: giai đoạn trung gian carbanion được bền hoá bởi sự xen phủ giữa AO-p chứa đôi
điện tử tự do của nguyên tử carbon với các AO-d trống của nguyên tử lưu huỳnh.
Do vậy chỉ có phản ứng 2 xảy ra theo sơ đồ như trên.
2. a)

b) Công thức chiếu Newmann:

Công thức chiếu Fischer:


c)

d) Xét cấu trúc của methcanthinone:

Phản ứng khử xảy ra theo quy tắc Cram-chelate, cho ra sản phẩm là diastereomer của PSE.
3. Xét cấu trúc của sản phẩm:

Trong sản phẩm 1 có tương tác đẩy giữa các nhóm thế, còn sản phẩm 2 không có, như vậy G
của phản ứng tạo thành sản phẩm 2 sẽ âm hơn sản phẩm 1.
Xét trạng thái trung gian của phản ứng 1 và 2:
- Phản ứng 1:

- Phản ứng 2:
Trong giai đoạn trung gian của phản ứng 1, cacbocation được bền hoá bởi hệ cộng hưởng allyl.
Tuy trong phản ứng 2, cacbocation trung gian cũng có hệ cộng hưởng allyl, nhưng bù lại tính
thơm của vòng kế cận bị phá vỡ. Do đó trạng thái trung gian của phản ứng 1 bền hơn phản ứng 2,
chính vì vậy mà năng lượng hoạt hoá của phản ứng 1 thấp hơn phản ứng 2.
Hay nói cách khác, phản ứng 1 thuận lợi về mặt động học, phản ứng 2 thuận lợi về mặt nhiệt
động. Do vậy nên ở nhiệt độ thấp sự hình thành sản phẩm 1 chiếm ưu thế, nhưng hiệu suất không
cao, còn ở nhiệt độ cao sự hình thành sản phẩm 2 chiếm ưu thế hơn, hiệu suất sẽ cao hơn. (Hiệu
suất phản ứng liên quan đến hằng số cân bằng K của phản ứng).

Câu 2: 4,0 điểm


a) Cấu trúc của các chất trung gian E, D, C, B:

Suy ra cấu trúc của A có thể có:


Suy ra quy tắc isoprene tương ứng với A:

b) Thuỷ phân vòng lactone của A trong môi trường kiềm, rồi sau đó acid hoá.
Suy ra chỉ có cấu trúc dạng a1 thoả các điền kiện cho F là nhiều dạng hỗ biến.

Câu 3: 4,0 điểm


1. a)

b)

c)

d)
2.

Câu 4: 4,0 điểm


a)
b) Trong phản ứng Alder – ene, ta dự đoán trạng thái trung gian sẽ là một vòng sáu cấu dạng ghế.
Ta có thể có 4 trạng thái chuyển tiếp như sau:

Thuật ngữ endo/exo mô tả sự định hướng của vòng xuất phát từ enophine và kết thúc ở ene. Nếu
vòng ngoài hướng về phía ene thì được gọi là endo, còn vòng ngoài ngược hướng với ene thì
được gọi là exo. E và Z cho biết cấu hình của nối đôi, còn anti và syn chỉ vị trí tương đối của hai
hiđro vinylic.
Trạng thái chuyển tiếp endo-Z-anti hoàn toàn kém bền bởi vì nó có chứa 2 liên kết “tựa” trục
(pseaudoaxial), mà nếu từ đó tạo thành sản phẩm thì sản phẩm sẽ có sức căng 1,3-diaxial. Hai
trạng thái exo-Z-syn và endo-E-syn gần như tương đương nhau về mặt năng lượng vì cùng chứa
1 liên kết “tựa” trục, tuy nhiên hai trạng thái này vẫn kém bền hơn exo-E-anti vì tất cả liên kết
trong cấu trúc này đều ở vị trí “tựa” xích đạo.
Tuy vậy, exo-E-anti lại có hai cấu trạng tương ứng:

Trong đó, trạng thái TS1 bền hơn TS2 vì có số liên kết “ngụy” xích đạo nhiều hơn. Do vậy sản
phẩm B1 được tạo ra từ trạng thái TS1 bền hơn sản phẩm B2 được tạo ra từ trạng thái chuyển tiếp
TS2

Câu 5: 4,0 điểm


1.
a) Cấu trúc của B1 và B2

b) Độ polyme hóa chính là số đơn vị lặp lại trong quá trình phản ứng, với Ao là 3, A1 là 6, còn A2
là 12…
Như vậy với A7 sẽ là n = 3 + 3.2 + 3.22 + 3.23 + 3.24 + 3.25 + 3.26 + 3.27 = 765
c) Số nhóm NH2 trong A8 sẽ là 3.28 = 768 nhóm, trong đó số nhóm thực chất sẽ phản ứng tiếp là
768(1 – 0,0417) = 736 nhóm, tức còn lại 32 nhóm không tham gia polyme hóa tiếp. Ở bước
polyme hóa tiếp theo số nhóm amin phản ứng sẽ tăng gấp 2 (736 x 2 = 1472 nhóm) cùng với 32
nhóm NH2 không phản ứng được sẽ có tổng cộng trong A9 là 1504 nhóm. Tức để phản ứng được
với 1 molA9 cần 1504 mol HCl, có nghĩa số mol HCl cần = 1504.2.10-4 = 0,3008 mol  VHCl =
501,3 mL
2.
H2C

H2C H2C - + + - - +
Na Na Na
Na

X1 X2

+ - +
Na - Na O
n n

X3

O O
+ - - +
Na O O Na
m n n m

X4
O O O

m
X6

- -
O O
O O

m X7
NĂM 2014
Đề 1: Đại cương. Vô cơ
Bài 1:
1. Do có sự thay đổi số nguyên tử trong một ô mạng nên thể tích phải được xét trên
một đơn vị khối lượng hay nguyên tử.
Vbcc = a3 / 2 = 0,01238 nm3 / nguyên tử
Vfcc = a3 / 4 = 0,01196 nm3 / nguyên tử
V fcc  Vbcc
Vậy %V  100%  4,94%
Vbcc

2.
a) Vạch α ứng với bước sóng của photon phát ra khi electron của nguyên tử hydro
chuyển từ n = 3 về n = 2.
2 2
hc Z  Z 
E   E3  E2   R  H   R  H 
  3   2 
6, 626.1034.2,998.108 1 1
  R  
 4 9
Với  = 6562,8.10-10m tính được R = 2,179.10-18
b) Năng lượng ion hóa
H(n = 1)(k) → H+(k) + e(k) I1
Ve = 0 m/s VH+(k) = 0 m/s
 1 
I1 = EH+(k) + E2 – EH(n=1) = 0  0    R  2   2,179.1010 J / nguyên tử H
 n 

c) H(n=x) 
photon
 H+(k) + e(k)
Theo bảo toàn năng lượng thì Ephoton = E(H+(k)) + Eelectron – E(H(n=x))
hc  1 
Tức  We    R  2   x  2
  x 
Vậy ban đầu H ở trạng thái n = 2.
3.
a) Phức Cu2+ bền hơn phức Zn2+ . Lý do phức của đồng chịu ảnh hưởng của hiệu ứng
Jahn – Teller làm cho liên kết trục của phức bát diện dài hơn liên kết xích đạo, dẫn
đến sự tương tác giữa các ligand trong phức bát diện của Cu2+ được giảm thiểu. Đây
chính là cơ sở của dãy độ bền Irving – William,
b) Sự tạo thành phức có phối tử đa càng được thuận lợi về mặt entropy, do đó phức có
ligand nhiều càng bền hơn phức có ligand ít càng. Vì thế [Ru(bipy)3]3+ bền hơn
[Ru(py)6]3+.
c) Cu+ là tâm axit mềm, nên thích hợp tạo liên kết với ligand là bazơ mềm. Lưu huỳnh
có tính bazơ mềm hơn nitơ.

Bài 2:
1. Có phản ứng
N2 + 3H2 ⇌ 2NH3
Ban đầu 1 mol 3 mol
Cân bằng 1-x 3 – 3x 2x
Với Pi = xi.Po (Po: áp suất hệ lúc cân bằng)
2 2 2
PNH xNH 1 nNH3 no2
Kp  3
 3
   (no là tổng số mol khí lúc cân bằng)
PN2 .PH32 xN2 .xH3 2 Po2 nN2 .n 3H 2 Po2

Từ đây ta có:
4x2 (4  2 x) 2
  KP
(1  x)(3  3 x)3 Po2
4 x 2 (4  2 x) 2
 K P .Po
27(1  x) 4
2 x(4  2 x)
   K P .Po
3 (1  x) 2

Với Po = 500 atm  x = 0,055 (mol), tức phần trăm chuyển hóa là 5,5%
Với Po = 1000 atm  x = 0,0764 (mol), tức phần trăm chuyển hóa là 7,64%
Kết quả thu được hoàn toàn phù hợp với nguyên lý Le Chartelier.
2. H2O(l) ⇌ H2O(k) Kp = Pnước = 1 atm
Như vậy đế nước sôi ở 100 oC thì áp suất của hơi nước phải là 1 atm.
Ở 25oC ta có: H2O(l) ⇌ H2O(k) Kp = Pnước = ?
P  H  1 1 
ln  25     298,15  373,15   P25 = 0,0369 atm
 P100  R  
Nghĩa là để quá trình chuyển pha xảy ra được ở 25oC thì áp suất khí quyển (hệ) phải bằng
0,0369 atm.
Gọi cạnh bình lập phương là a (dm). Như vậy để tất cả nước bốc hơi thì số mol hơi nước
phải tương ứng với số mol nước ban đầu. Tức là
3, 785.103  0,998
PV  nRT  0, 0369  a 3   0, 082  298,15
18, 02

 a = 51,78 dm

Bài 3:
1. a) Biến đổi phương trình Eyring thành dạng:
𝑘ℎ
∆𝐺 ≠ = −𝑅𝑇𝑙𝑛 ( )
𝑘𝐵 𝑇
Từ đó ta có bảng sau:
k (M-1s-1) T (K) ∆𝑮≠
(kcal/mol)
2.6×10-3 293 20.6
3.5×10-3 298 20.8
3.9×10-3 303 21.1
4.3×10-3 308 21.4
b) Thế ∆G≠ = ∆H≠ - T∆S≠ vào phương trình Eyring:
𝑘𝐵 𝑇 −∆𝐻 ≠ + 𝑇∆𝑆 ≠
𝑘= 𝑒𝑥𝑝 ( )
ℎ 𝑅𝑇
Lấy log hai vế, ta có:
𝑘 ∆𝐻 ≠ ∆𝑆 ≠ 𝑘𝐵
ln =− + + 𝑙𝑛
𝑇 𝑅𝑇 𝑅 ℎ
Do đó ln(k/T) quan hệ tuyến tính với 1/T, nên ta có đồ thị sau:

-11,1
0,0032 0,00325 0,0033 0,00335 0,0034 0,00345
-11,2
-11,3

ln(k/T)
-11,4
-11,5
-11,6
-11,7
1/T

Hệ số góc chính là -∆H≠/R = -2633, suy ra ∆H≠ = 5.23 kcal/mol.


Giao điểm với trục tung là ∆S≠/R + ln(kB/h) = -2.59, do đó ∆S≠ = -0.0518 kcal/mol.
Ta có bảng sau:
T (K) ∆𝑮≠ = ∆𝑯≠ − 𝑻∆𝑺≠
(kcal/mol)
293 20.4
298 20.6
303 20.9
308 21.2

2. a) Nếu xem các nồng độ khác lớn hơn rất nhiều so với [Tl3+], ta có thể coi quy luật
động học trở thành giả bậc nhất (pseudo-first order) đối với Tl3+. Do đó phương
trình động học trở thành:
𝑑[𝑇𝑙 3+ ]
− = 𝑘𝑜𝑏𝑠 [𝑇𝑙 3+ ]
𝑑𝑡
Nên phương trình động học tích phân, hay mối quan hệ giữa [Tl3+] với thời gian là:
[𝑇𝑙 3+ ] = [𝑇𝑙 3+ ]0 𝑒𝑥𝑝(−𝑘𝑜𝑏𝑠 𝑡)
b) Thành phần nguyên tử của trạng thái chuyển tiếp bằng tổng nguyên tử ở phần tử số trừ
đi tổng nguyên tử ở phần mẫu số trong phương trình động học vi phân. Do đó thành phần
nguyên tử của trạng thái chuyển tiếp là: [TlN2H4]≠.
c) Vì thành phần nguyên tử của trạng thái chuyển tiếp là [TlN2H4]≠, bước tốc định phải là sự
tương tác giữa Tl3+ và N2H4. Trong môi trường acid mạnh, phần lớn N2H4 bị proton hóa
thành N2H5+. Do đó cơ chế sẽ bao gồm sự deproton hóa N2H5+ để tái tạo N2H4, rồi N2H4 tác
dụng với Tl3+ theo tỉ lệ 1:1 trong giai đoạn quyết định tốc độ.
Cần nhớ rằng nếu N2H4 bị oxi hóa hoàn toàn thành N2, tỉ lệ phản ứng giữa Tl3+ và N2H4
không còn là 1:1 nữa, do đó sự oxi hóa N2H4 thành N2 (sản phẩm cuối cùng) không hoàn
toàn nằm trong bước quyết định tốc độ. Cơ chế được đề nghị như sau:
(1)𝑁2 𝐻5+ ⇌ 𝑁2 𝐻4 + 𝐻 +
(2)𝑁2 𝐻4 + 𝑇𝑙 3+ ⇌ 𝑇𝑙 + + 𝑁2 𝐻2 + 2𝐻 +
(3)𝑇𝑙 3+ + 𝑁2 𝐻2 ⇌ 𝑇𝑙 + + 𝑁2 + 2𝐻 +
Trong đó bước (2) là bước chậm.

Bài 4:

1. (NH2)2CO + H2O 
 2 NH 3  CO2
2. [Ca2+] = 6,85.10-3M
3. [Ca2+] = [C2O42-] + [HC2O4-] + [H2C2O4]
= [C2O42-](1 + [H+]/K1 + [H+]2/K1K2)
Vậy [C2O42-] = [Ca2+]/(1 + [H+]/K1 + [H+]2/K1K2) (*)
Thay (*) vào biểu thức tích số tan: T = [Ca2+][C2O42-] ta tính được [C2O42-] = 1,92.10-4M
4. Ta có:

CCa  Ca 2   CaC2O4( aq )   Ca  C2O4 2 


2
 
 1 
T   K  K K  C O
f1 f 2  2 4 
2
 
 C2O42  f1

  
dCCa 1
 T  TK f 1 K f 2  0
d C2O4 
2 2
C2O42 

[C2O42-] = 1,0.10-2M  [Ca2+] = 1,3.10-6M


5. Cân bằng điện tích: 2[Ca2+] + [H+] = 2[C2O42-] + [HC2O4-] + [OH-] (1)
Cân bằng khối lượng: [Ca2+] = [C2O42-] + [HC2O4-] + [H2C2O4] (2)
Vì Kb2 rất nhỏ nên nồng độ của H2C2O4 có thể bỏ qua.
Kết hợp (1) và (2) ta có: [HC2O4-] = Kw/[H+] - [H+] (3)
[C2O42-] = (K2Kw)/[H+]2 – K2 (4)
[Ca2+] = T/[C2O42-] = T[H+]2/(K2Kw – K2[H+]2) (5)
Thay (3), (4), (5) vào (2) và giải phương trình sinh ra ta được: [H+] = 5,5.10-8M
[Ca2+] = 1,04.10-4M

Bài 5:
1. A là NF3. Phản ứng xảy ra như sau: 4NH3 + 3F2 = 3NH4F + NF3
2. Do trong NF3 cặp electron của N bị giữ chặt hơn nên sự proton hóa khó khăn hơn so
với NH3, có nghĩa là nhiệt phản ứng proton hóa NF3 nhỏ hơn.
3. 2NF3 + 2Hg = N2F2 + HgF2
Cấu trúc hai đồng phân của N2F2

4. 2N2F2+SnF4=[N2F+]2[SnF6]2-
[N2F+]2[SnF6]2-=[N2F]+[SnF5]-+N2F2
Cấu trúc các ion:

D: E: F:
5. NF3+F2+BF3=NF4+BF4-
Khi thủy phân cation NF4+ có sự tạo thành HOF và NF3
NF4+ + H2O = NF3 + HOF + H+
HOF là chất oxy hóa rất mạnh và kém bền, có thể phân hủy tạo HF và O2
2HOF = 2HF + O2
Hay oxy hóa nước để tạo H2O2 và HF
HOF + H2O = HF + H2O2
Đề 1: Hữu cơ
Bài 1:
1. a) D.E.P là tên viết tắt của Dietyl Phtalat vốn là một dieste của axit phtalic. Từ đó có
thể dễ dàng suy ra cấu trúc các chất như sau:

b) X thường được dùng để bảo vệ nhóm chức amin

c) Đây là phương pháp Gabriel thường được sử dụng để tổng hợp các aminoaxit từ
dẫn xuất halogen

2. D > E > A > B > C. Đầu bài hỏi rằng khả năng phân ly của gốc -COOH, ta có D và E có
gốc bazơ trong phân tử nên sẽ giúp gốc -COOH phân ly mạnh hơn (do có phản ứng
axit – bazơ) nên khả năng phân ly của gốc -COOH của D và E là mạnh nhất. Do tính
bazơ của CH3-NH- mạnh hơn NH2- nên sự phân ly gốc -COOH của D > E. Còn A, B, C
thì so sánh theo hiệu ứng -I .
3. Chất D là một amin no bậc hai nên sẽ có tính bazơ cao nhất so với các amin thơm còn
lại.
Các chất A, C, E do có mặt nhóm rút nên tính bazơ sẽ thấp hơn so với B và F. Trong
đó B có tính bazơ cao hơn F do có mặt nhóm CH3 làm tăng tính bazơ.
E có tính bazơ cao hơn hai chất A, C do số nhóm rút ít hơn.
A có tính bazơ cao hơn C do nhóm NO2 para chỉ gây hiệu ứng –I (do tương tác đẩy
giữa các nhóm nitro khiến nhóm nitro para bị lệch phẳng nên không thể gây –C).
Trái lại nhóm CN para gây –C do không có tương tác đẩy với hai nhóm nitro bên
cạnh (nhóm CN định hướng thẳng). Do –C luôn mạnh hơn –I nên chất C phải có tính
bazơ thấp hơn.
Như vậy thứ tự tính bazơ sẽ như sau: (D) > (B) > (F) > (E) > (A) > (C)

Bài 2:
1. a)

b)

2. Phản ứng xảy ra theo cơ chế E1cB do ion enolat sinh ra từ cơ photpho trung gian thể
hiện tính bazơ thay vì nucleophin. Lý do có thể cho rằng trung gian sinh ra cồng
kềnh sẽ khó tương tác với nhóm CHO andehit vốn dĩ đã bị chắn bởi nhóm OTBS
khiến không thể tạo thành sản phẩm như ý muốn. Cơ chế cụ thể của phản ứng như
sau:

Bài 3:
1. Các sản phẩm phản ứng

Hướng tấn công phía trước bị cản trở bởi nhóm benzyl.

Hướng tấn công phía trên bị cản trở bởi methyl.

Tấn công axial, do chỉ có carbanion sinh ra khi tách H axial mới có thể tạo được sự
xen phủ bền với nhóm C=O như hình vẽ:

2. Quy trình tổng hợp đề nghị như sau:


Bài 4:

Ở bước phản ứng chuyển hóa E thành F bước đầu tiên là sự tách E1cB nhóm OCH3
equatorial, sản phẩm xiclohexen sinh ra sẽ cộng syn để thu được chất F như hình vẽ:
Hướng tấn công của phản ứng cộng H2 sẽ tránh nhóm OCH3 equatorial gắn với C4
dẫn đến nguyên tử H C5 nằm ở vị trí axial, do phản ứng là cộng syn nên nguyên tử H của C6
phải là equatorial dẫn đến sản phẩm F.

Bài 5:
1. Từ kết quả cắt mạch bằng enzym cacboxipeptidaza cho thấy đoạn cuối mạch có thứ
tự Asn – Met – Asp – Gly – COOH.
Aminoaxit đầu N là Glu.
BrCN cắt sau Met, trypsin cắt sau các aminoaxit có tính bazơ, ở đây trypsin cắt sau
Lys và Chymotrypsin cắt sau các aminoaxit có tính ky nước (ở đây chymotrypsin cắt sau
Phe) nên từ những kết quả trên có thể suy ra trật tự sắp xếp các aminoaxit trong peptit là:
Glu – Met – Pro – Pro – Phe – Lys – Glu – Asn – Met – Asp – Gly.
2. Nếu cho rằng trạng thái chuyển tiếp đi qua trung gian carbocation thì chất nào tạo
thành carbocation dễ dàng nhất thì khả năng phản ứng tăng lên tương ứng. Trong
chất D thì có sự giải toả electron mạnh nhất:

Giải thích tương tự ta thấy carbocation tạo thành từ B bền hơn C, còn ở A sẽ kém bền
nhất. Như vậy thứ tự sẽ là: D > B > C > A.
Vẫn có thể có một số đáp án khác phù hợp với đề bài. Những trường hợp như
vậy vẫn được chấp nhận.
Đề 2: Đại cương. Vô cơ

Bài 1:
1.
a) Giản đồ MO của CO

b) HOMO của CO là orbital σ3 , mang đóng góp lớn hơn từ O. Tương tự đối với LUMO là
π* với đóng góp lớn hơn từ C. Tức CO nối với Fe3+ tại đầu C để liên kết σ cho và π nhận
có hiệu quả (xem hình)
c) Phức trên sẽ có liên kết π-cho nhận với kim loại cho – CO nhận. Chiều dài liên kết
giữa C và O sẽ tăng, do electron “cho vào” orbital π* mang tính phản liên kết của CO,
làm giảm bậc liên kết giữa của phân tử CO.
2. Ligand có thể tạo phức π-cho có mức năng lượng t2g thấp hơn mức năng lượng t2g
của kim loại (vì đó là những MO bão hòa). Trong khi ligand có thể tạo phức π-nhận
có mức t2g cao hơn mức t2g của kim loại (vì đó là những MO trống). Do đó, ta có sự tổ
hợp MO như sau:

Ta có thể thấy ∆O1 < ∆O < ∆O2, do đó phức π-cho có sự tách mức năng lượng nhỏ hơn phức
chỉ có σ-cho. Còn phức π-nhận có độ tách tinh trường lớn nhất. Tùy thuộc vào khả năng cho
hay nhận π tốt hay kém của ligand mà khả năng tách mức năng lượng biến đổi lớn hay nhỏ.
Đó là cơ sở của dãy phổ hóa học của ligand.

Bài 2:
1)
a) Uml = -618 - (161 + 520 + 158/2 – 328) = -1050 kJ/mol
b) Các ion dương và ion âm có kích thước tương đối khác nhau sẽ có cách sắp xếp khác
nhau trong mạng tinh thể.
Phối trí 4 (hốc tứ Phối trí 6 (hốc bát Phối trí 8 ( lập
diện) diện) phương)
r(+)/r(-) 3 𝑟(+) √2 − 1 <
𝑟(+)
< √3 − 1 √3 − 1 <
𝑟(+)
<1
√ −1< < √2 − 1 𝑟(−) 𝑟(−)
2 𝑟(−)

Đối với mạng tinh thể NaCl, Na+ có số phối trí 6, phân bố trong các hốc bát diện của Cl-.
Vậy cần phải chứng minh:
𝑟(+)
√2 − 1 < < √3 − 1
𝑟(−)
Sử dụng các hình sau để chứng minh hình học:

Giới hạn dưới - Để mạng tinh thể bền vững:


AC >= AD*√2
2r+ + 2r- >= 2√2r-
𝑟(+)
√2 − 1 < 𝑟(−)

Giới hạn trên - Để chứng minh vế sau, ta phải xét điểm chuyển tiếp giữa
phối trí 6 và phối trí 8.
Để cấu trúc phối trí 8 bền vững:
AC >= AB√3
2r+ + 2r- >= 2√2r
𝑟(+)
> √3 − 1
𝑟(−)
𝑟(+)
Vậy giới hạn trên của phối trí 6 sẽ là: < √3 − 1
𝑟(−)

c) Số cặp 2 ion Cl- và Li+ = 3.97 ~ 4. Kết hợp với tỉ lệ r(+)/r(-) = 0.539 => Kiểu NaCl.
2)
a) Sử dụng A = Aoe-kt với k = ln 2/ t1/2. Tính ra được tC = 100000 năm và tU= 200000 năm.
Phương pháp 14C kém hiệu quả vì thời gian lớn hơn rất nhiều so với chu kỳ bán hủy. Phương
pháp 238U cũng không hiệu quả lắm do thời gian quá sớm so với chu kỳ bán hủy, ngoài ra loại
bỏ ảnh hưởng của các đồng vị cân bằng thế kỉ. Tuy nhiên phương pháp 238U hiệu quả hơn do
có ít thất thoát và sai số nhỏ.
(Có thể chứng minh sai số phương pháp 238U nhỏ hơn bằng lý luận: t = ln(x) x 1/k với x =
Ao/A => giá trị x càng lớn, đạo hàm (f’) càng nhỏ, hay sai số dt tại dx càng nhỏ.)
b) Thử nghiệm hạt nhân trên mặt đất sản sinh ra 14C cũng như vô số các đồng vị khác.
Tuy nhiên, lượng 238U, 206Pb (hay các đồng vị khác của chuỗi) trong mẫu vật không
đổi, do các đồng vị này nặng và không phân tán rộng trong không khí.
Việc nồng độ 14C tăng lên trong không khí làm cho mẫu vật có vẻ già đi. Tức là tuổi thực tế
của mẫu vật nhỏ hơn so với tính toán. Nếu tuổi thực tế nhỏ hơn hẳn so với 100000 năm,
phương pháp 14C hợp lý hơn. Nếu tuổi thực tế nhỏ hơn không đáng kể, phương pháp 238U vẫn
hợp lý hơn.
Nếu nồng độ 14C thực tế lớn hơn tự nhiên n lần thì tuổi phải giảm đi một lượng:
ln (n) x 1/k = 8266.6 ln (n)

Bài 3:
1.
H 2 Ol   H 2 Oh  (*)

Kp
G298K *   RTLnK  K   3,04.10 2  K p  3080 Pa  Pcb, 298K *  Pbh, 298K H 2 O 
0
nk
P 0

 Pkk , 298K H 2 O  0,67 Pbh, 298 H 2 O  2064 Pa

bh: bão hòa; cb: cân bằng; kk: không khí.


Tính Pcb' , 298K H 2 O  khi có cân bằng: Na2 SO4 .10 H 2 Or   Na2 SO4r   10 H 2 Ok  (**) ở 25oC.

Pcb' ,298 K  H 2O   2523Pa  Pkk ,298 K  H 2O 

Như vậy phản ứng (**) chuyển dịch theo chiều thuận và Na 2 SO 4 .10 H 2 O mất nước .

K 2 H 0  1 1 
2. Dùng Ln     tính lại các hằng số cân bằng của (*) và (**) ở 0oC và
K1 R  T1 T2 

suy ra:
Pcb' ,298 K  H 2O   366 Pa  Pkk ,298 K  H 2O   404 Pa .

3. Phản ứng (**) chuyển theo chiều nghịch và Na2SO4 chảy rữa.
Có điều này khi: Pkk , 298K H 2 O   Pcb' , 298K H 2 O   2523 Pa nghĩa là độ ẩm tương đối của không

khí bằng 81,9%.


4. Không.
Bài 4:
1.
a) A là chất khử, B là chất oxi hóa.
b) Chất trung gian: [A,B]
𝑑[𝐴, 𝐵]
= 𝑘𝑑 [𝐴][𝐵] − 𝑘𝑟 [𝐴, 𝐵] − 𝑘𝑒 [𝐴, 𝐵] = 0
𝑑𝑡
Do đó:
𝑘𝑑
[𝐴, 𝐵] = [𝐴][𝐵]
𝑘𝑟 + 𝑘𝑒
Vì bước (2) là bước chậm: R = ke[A,B] nên kết hợp hai phương trình trên ta có:
𝑘𝑒 𝑘𝑑
𝑅= [𝐴][𝐵]
𝑘𝑟 + 𝑘𝑒
Vì bước (2) là bước chậm, ta có thể giả thiết ke << kr. Do đó:
𝑘𝑒 𝑘𝑑
𝑅= [𝐴][𝐵]
𝑘𝑟
c) Với giả thiết kd ~ kr, ta có kd/kr ~ 1. Do đó:
R = ke[A][B]
d) Nếu xem quá trình khuếch tán để mang hai phân tử lại gần nhau là một phản ứng
hóa học thì đây là một phản ứng rất nhanh (kd ~ 1013 M-1s-1, d = diffusion = khuếch
tán). Ngược lại, sau khi hai phân tử va chạm nhau do sự khuếch tán rồi tách rời
nhau, vì trước và sau phản ứng đều không có sự thay đổi đáng kể nên hằng số tốc độ
của quá trình này, kr, cũng xấp xỉ bằng kd, nghĩa là phản ứng cũng rất nhanh. Do đó
câu hỏi đặt ra ở đây là tại sao ke lại nhỏ hơn rất nhiều so với kd và kr, làm cho bước
(2) trở thành bước quyết định tốc độ?
Lấy ví dụ như phản ứng oxi hóa – khử ngoại cầu giữa [Fe(H2O)6]3+ và [Fe(H2O)6]2+:
[Fe*(H2O)6]3+ + [Fe(H2O)6]2+ → [Fe(H2O)6]3+ + [Fe*(H2O)6]2+
(dấu * để đánh dấu [Fe*(H2O)6]3+ → [Fe*(H2O)6]2+, còn [Fe(H2O)6]2+ → [Fe(H2O)6]3+)
Ta có thể thấy rằng liên kết Fe3+-OH2 ngắn hơn so với liên kết Fe2+-OH2. Do đó, khi hai phân
tử [Fe(H2O)6]3+ và [Fe(H2O)6]2+ rồi sự chuyển electron xảy ra tức thời thì trong hậu phức
hợp, Fe3+-OH2 dài hơn bình thường, còn Fe2+-OH2 sẽ ngắn hơn bình thường, dẫn đến năng
lượng của hậu phức hợp sẽ rất cao. Dĩ nhiên, điều này là trái với định luật bảo toàn năng
lượng vì về mặt nguyên tắc ta phải cung cấp năng lượng để bù đắp cho khoảng cách năng
lượng giữa tiền phức hợp và hậu phức hợp. Chính vì thế, mỗi phức chất trong tiền phức hợp
phải được sắp xếp lại (reorganization) bằng sự dao động liên kết. Điều này đã khiến cho
năng lượng hoạt hóa của quá trình cho-nhận electron rất cao, làm cho bước (2) trở thành
bước chậm nhất.
2. Chỉ có cặp [Co(NH3)5Cl]2+ và [Cr(H2O)6]2+ có thể thực hiện phản ứng oxi hóa – khử
nội cầu, bởi vì Cl- có nhiều cặp electron tự do, có thể đóng vai trò là ligand cầu nối.

Bài 5:
1. Ta có:

[Pb2+] = 1.00×10-2 mol· L-1

Với K sp, [Pb2+] [OH-]2 = 8.0×10-16 [OH-] = 2.8×10-7 mol· L-1

Có K w = 1.00×10-14 = [H3O+] [OH-] [H3O+] = 3.5×10-8 mol·L-1 pH = 7.45

2. Ở pH tương đối cao thì độ tan do ion Pb(OH)3- quyết định,


[Pb(OH) 3 ] = 1,00102 mol.L1

Áp dụng biểu thức Ka : [Pb(OH) 3 ] [H3O+] = 1.0×10-15 , [H3O+] = 1.0×10-13 mol·L-1


pH = 13,00
Dùng Ksp thu được [Pb2+] = 8.00×10-14 mol·L-1 quá nhỏ so với 1.00×10-2 mol·L-1
3. S = [Pb2+] + [Pb(OH)3-]
4. [H3O+] = 10-9.40 = 4.0×10-10 mol·L-1,
Áp dụng Kw = 1.00×10-14, [OH-] = 2.5×10-5 mol·L-1

Từ Ksp, [Pb2+] = 1.3×10-6 mol·L-1 , Từ Ka, [Pb(OH)3-] = 2.5×10-6 mol·L-1

Ta có: S = [Pb 2+] + [Pb(OH)3-], S = 1.3×10-6 + 2.5×10-6 = 3.8×10-6 mol·L-1


5. Ở pH tương đối thấp Pb2+ sẽ chiếm ưu thế. [Pb2+] = 1.0×10-3 mol·L-1

Ksp= [Pb2+ ] ∙ [OH − ]2 =8.0×10-16


16
[OH − ] = 8,0  10 = 8,94107 [H 3O+ ] = 1,12108 pH = 7,95
1,0  103

Ka 1,0  1015
[Pb(OH) 3 ] = = 8
= 8,94108 mol.L1 ,
 H O  1,12  10
 3 

Nên trong môi trường axit nồng độ của nó có thể bỏ qua được và Pb2+ chiếm ưu thế.
Trong môi trường tương đối kiếm thì Pb(OH)3- sẽ chiếm ưu thế.

Từ Ka tính được [Pb(OH) 3 ] = 1,00103 [H 3O+ ] = 1,001012 mol·L-1

pH = 12.00 and [OH-] = 1.00 × 10−2


Từ Ksp ta có: [Pb 2+]= 8.00 × 1012 mol·L-1 và [Pb2+] << [Pb(OH)3-]
Vùng pH sẽ là 7.95-12.00

Bài 6:
Hợp chá t chưa bié t tên: A – NO2, B – NO, C – NOHSO4, D – H2NSO5.
Phản ứng:
1) SO2 + NO2 + H2O = H2SO4 + NO 2) 2NO2 + H2O (lạ nh) = HNO3 + HNO2
3) 3NO2 + H2O (nó ng) = 2HNO3 + NO 4) 4NO2 + 2H2O + O2 = 4HNO3
5) 3NO2 + 2SO2 + H2O = 2NOHSO4 + NO 6) NO2 + SO2 + H2O = H2NSO5
7) 2H2NSO5 + NO2 = 2NOHSO4 + NO + H2O
8) 2NOHSO4 + H2O = 2H2SO4 + NO + NO2 (3NOHSO4 +2H2O = 3H2SO4 + HNO3 + 2NO)
Công thức cấu tạo của axit sunfonitrơ D:
Đề 2: Hữu cơ

Bài 1:
1. E có tính bazơ lớn nhất (do chỉ có nó là amin). C có tính bazơ bé nhất do cặp e trên
nguyên tử Nitơ chịu ảnh hưởng –C của hai nhóm C=O và vòng benzen. Trong khi đó
D chỉ chịu ảnh hưởng của nhóm C=O, còn B do cấu trúc cứng nhắc của vòng
quinuclidin nên khả năng liên hợp của cặp e trên N về phía các nhóm rút trở nên rất
hạn chế. Như vậy tính bazơ sẽ theo thứ tự: E > A > B > D > C
2. Một chất chỉ thể hiện tính thơm khi nó thỏa mãn các điều kiện sau: Hệ liên hợp đồng
phẳng trên toàn hệ vòng, cũng như có số electron pi thỏa mãn quy tắc Huckel N = 4n
+ 2 (với N là số electron pi, còn n là một số nguyên). Trường hợp hệ phản thơm thì N
= 4n. Nếu không thỏa mãn những điều kiện này hệ thống được xem là không thơm.
Với các hệ thống đa vòng thì chỉ tính dựa trên tổng số electron pi theo chu vi (tức với
chất T sẽ tính số electron pi trên vòng lớn bao bên ngoài, chứ không xét những vòng
nhỏ).
Từ đó rút ra kết luận:
Hệ thơm: N, S, T
Hệ phản thơm: M, R
Hệ không thơm: O, P (số electron pi tuy thỏa mãn N = 4n nhưng hệ thống này không
phẳng, xiclooctatetraen và quinon đều mang cấu dạng thuyền).
3. Do trong X tồn tại liên kết hydro nội phân tử nên độ tan của X rất nhỏ hơn Y. Tuy
nhiên điều này khiến nhiệt độ nóng chảy của X lớn hơn (Liên kết hydro nội phân tử
khiến phân tử có dạng gần với hình cầu nên xếp khít hơn vào mạng tinh thể).

Bài 2:
1. Thông thường mỗi carbon trong một liên kết ba có trạng thái lai hóa sp, nên ưu tiên
góc liên kết 180o. Trong khi đó, nối ba trong benzyne bị khống chế bởi cấu trúc lục
giác phẳng của vòng sáu nên rất căng. Do đó, benzyne là một chất trung gian rất hoạt
động. Một khi được tạo thành, chúng lập tức phản ứng với nucleophile hay diene
(Diels-Alder) hay hệ allyl (Alder-ene).
2.

3.

Phản ứng tạo thành benzyne theo phương pháp này được gọi là phản ứng
Hexadehydro-Diels-Alder (HDDA)

Bài 3:
c) Nếu không trung hòa dung dịch trước bằng NaHCO3 thì khi cho Me2S vào để khử nhóm
peroxyl ancol, nhóm andehit vừa tạo thành sẽ phản ứng ngay với metanol dẫn đến sự tạo
thành sản phẩm diaxetal:

Bài 4:
1. Sơ đồ tổng hợp đề nghị như sau:

2. Cấu trúc các chất chưa biết

Bài 5:
1.
a) Cơ chế phản ứng như sau:

b) Có thể giải thích theo một trong ba cách sau:


Cách 1: Obitan p của carbocation phải nằm thẳng góc với cặp electron n của oxy để
tạo được sự liên hợp bền.
Cách 2: Sản phẩm α-glycozit không xảy ra sự đẩy lưỡng cực giữa cặp electron n của
oxy và liên kết C-O như sản phẩm β-glycozit.

Cách 3: Tồn tại tương tác n - *C-O giữa cặp e của O1 với liên kết C2 – O(CH3).

c) Sơ đồ tổng hợp.
2. Các phản ứng xảy ra:
a.

1-flo-2,4- đinitrobenzen Leu-Gly-Ser DNP-Leu-Gly-Ser


b.

Ile-Glu-Phe Glu-Phe

c.

Asn-Ser-Ala Benzyloxycacbonyl clorua Z-Asn-Ser-Ala


d.

Z-Asn-Ser-Ala p-nitrophenol
Este Z-Asn-Ser-Ala p-nitrophenyl
e.

Este Z-Asn-Ser-Ala p-nitrophenyl este etyl Valin

Este etyl Z-Asn-Ser-Ala-Val


f.

Este etyl Z-Asn-Ser-Ala-Val Este etyl Asn-Ser-Ala-Val

Vẫn có thể có một số đáp án khác phù hợp với đề bài. Những trường hợp như
vậy vẫn được chấp nhận.
Đề 3: Đại cương. Vô cơ

Bài 1:
1. Phân bố hình học của các chất:
Chất Công thức Trạng thái lai hoá Hình học phân tử
VSEPR
(C6H5)IF5- MX6E1 Sp3d3 Tháp đáy ngũ giác
(C6H5)2Xe MX2E3 Sp3d Gần như đường thẳng
I(C6H5)2+ MX2E2 Sp3 Góc
POCl3 MX4E0 Sp3 Tháp đáy tam giác
SBrF5 MX6E0 Sp3d2 Tứ diện lệch
SBr2F4 MX6E0 Sp3d2 Tứ diện lệch (Phải vẽ rõ 2
dạng, cis và trans do sự
phân bố Br2
H2O2 MX2E2 Sp3

N2F2 MX2E1 Sp2 Phân tử phẳng, 2 dạng cis và


trans do sự phân bố F

I2O5 MX3E1 Sp3 Tứ diện (cho mỗi nguyên tử


Oxy)
Cl2O7 MX4E0 Sp3 Tứ diện ( Cho mỗi nguyên tử
Clo)
SF4 MX4E1 Sp3d Bập bênh
NH2- MX2E2 Sp3 góc

2. H2  H2+ + e E1 = 15,42 eV (1)


H2  H(1s) + H(1s) E2 = 435,5 kJ/mol = 4,52 eV (2)
H2+  H+ + H(1s) E3 = ?
E3 = EH+ + EH – EH2+ = EH – EH2+
1
EH = -13,6 x 1 = -13,6 eV

EH2 = 2EH – E2 = -13,6 x 2 -4,52 = -31,72 eV (từ 2)


EH2+ = EH2 + E1 = -31,72 + 15,42 = -16,3 eV
E3 = EH – EH2+ = -13,6 – (-16,3) = 2,7eV
Năng lượng liên kết H2+ nhỏ hơn năng lượng liên kết H2 phù hợp với việc giảm bậc liên kết
H2 theo thuyết MO.
3.
a) Từ tỉ số bán kính:
rSi4+ 41pm rAl3+ 50 pm
  0.29   0.36
rO2- 140 pm rO2- 140 pm

Cả hai đều nằm trong khoảng 0,225 – 0,414 nên cả hai ion đều chiếm hốc tứ diện
b) Độ dài liên kết giữa Si4+ và O2- là 181 pm. Tuy nhiên giá trị thực nghiệm lại là 160 pm
nhỏ hơn nhiều. Lý do là ion Si4+ có bán kính nhỏ đã khiến cho liên kết Si – O mang
tính cộng hóa trị nhiều hơn nên trị số bán kính phải giảm.
c) Ta có:
pV 1.01325 105 Pa 798.6 103 m3
n   10.00 mol
RT 8.3144 JK -1mol-1 973.15 K
(2207+2590+2697+6144+18.02x) 13638+18,016x
= (g)
6,022 1023 6,022 10 23
13638+18.016x
811.5:  6.022  1024:x = 216
6.022  1023
13638+18.016 x 13638  18.016  216
D=  10 3
 1.946(gcm3 )
6.022 10 a
23 3
6.022 10  (2464 10 )
23

Bài 2:
1.
a) ∆H, ∆S, ∆G là hàm trạng thái, không phụ thuộc vào cách thức diễn biến quá trình do
đó không có sự khác biệt giữa tiến hành thuận nghịch hay bất thuận nghịch.
Phản ứng : 2H2(k) + O2 (k) => 2 H2O(l)
Tính toán như bình thường thu được:
∆H = -571.66 kJ/mol
∆S = -327.32 J/molK
∆G = -474.1 kJ/mol
b) Bất thuận nghịch:
Trong phản ứng bất thuận nghịch, công có ích bằng 0 do phản ứng xảy ra nhanh, không thu
hồi được năng lượng.
0.082.298
Công thể tích = Wv = -P∆V = -101325 x (0-1-2) x x 10-3 = 7428J
1

Nhiệt: Q = ∆H - Wcó ích = ∆H = -571.66 kJ/mol


Thuận nghịch:
Trong phản ứng thuận nghịch, công có ích bằng ∆G (Tương tự trong pin, năng lượng có ích
của phản ứng trong pin tạo ra dòng điện nếu xét hiệu suất 100% chính là ∆G)
Wcó ích = ∆G = -474.1 kJ/mol
Công thể tích không có sự khác biệt.
Wv = 7428 J
Nhiệt: Q = ∆H - Wcó ích = -571.66 –(-474.1) = -97.56 kJ/mol
−𝑄
c) ∆Smôi trường = 𝑇

∆Svũ trụ = ∆Smôi trường + ∆Sphản ứng


Bất thuận nghịch:
571.66
∆Smôi trường = . 1000 = 1918.3 J/mol.K
298

∆Svũ trụ = 1918.3 -327.32 = 1591 J/mol.K


Thuận nghịch:
97.56
∆Smôi trường = . 1000= 327.38 J/mol.K
298

∆Svũ trụ = 327.38-327.32 = 0.06 J/mol.K


2.

a) 2HgO(r) ⇌ 2Hg(k) + O2(k)


Pcb: 2a a (atm)
Pcb = 3a = 8 => a = 4/3 (atm)
Kp = 4a3 = 9,48 atm3
b) Lượng HgO tối thiểu để đạt cân bằng phải đủ để giải phóng lượng O2 và Hg có áp suất
như trên. Như vậy:
4.0.5 2
mmin = n x MHgO = 0.082.773 . 3 . 216=4,53 gam

c) Khi m = 5 g
Do m = 5g > mo = 4.53g, lớn hơn lượng HgO cần thiết để đạt cân bằng nên cân bằng sẽ
không chuyển dịch, lượng khí trong bình không đổi. Do đó áp suất giữ nguyên P1 = 4 atm.
Khi m = 3g < mo = 4.53 gam  hệ không đạt được cân bằng. HgO rắn phản ứng hoàn
3 3
. .0.082.773
toàn tạo thành Hg và O2 với P2 = 3PO2 = 216 2 = 2.64 atm.
0.5

Bài 3:
1. Cho L = PPh3. Ta có:
𝑑[𝑅ℎ𝐶𝑙𝐿3 ]
− = 𝑘1 [𝑅ℎ𝐶𝑙𝐿3 ][𝐻2 ] + 𝑘2 [𝑅ℎ𝐶𝑙𝐿3 ] − 𝑘−2 [𝑅ℎ𝐶𝑙𝐿2 ][𝐿] (∗)
𝑑𝑡
Ta lại có:
𝑑[𝑅ℎ𝐶𝑙𝐿2 ]
= 𝑘2 [𝑅ℎ𝐶𝑙𝐿3 ] − 𝑘−2 [𝑅ℎ𝐶𝑙𝐿2 ][𝐿] − 𝑘3 [𝑅ℎ𝐶𝑙𝐿2 ][𝐻2 ] = 0
𝑑𝑡
𝑘2 [𝑅ℎ𝐶𝑙𝐿3 ]
⟹ [𝑅ℎ𝐶𝑙𝐿2 ] = (∗∗)
𝑘−2 [𝐿] + 𝑘3 [𝐻2 ]
Thế (**) vào (*), thu gọn, ta được:
𝑑[𝑅ℎ𝐶𝑙𝐿3 ] 𝑘2 𝑘3
− = (𝑘1 + ) [𝑅ℎ𝐶𝑙𝐿3 ][𝐻2 ]
𝑑𝑡 𝑘−2 [𝐿] + 𝑘3 [𝐻2 ]
2.
a)
𝑑[𝐴]
− = 𝑘𝑓 [𝐴] − 𝑘𝑟 [𝐵]
𝑑𝑡
b) Tại cân bằng: kf[A]eq = kr[B]eq, do đó:
[𝐴]𝑒𝑞 [𝐵]𝑒𝑞
=
𝑘𝑟 𝑘𝑓
Ta lại có: [A]eq + [B]eq = [A]0. Theo tính chất dãy tỷ số bằng nhau, ta có:
[𝐴]𝑒𝑞 [𝐵]𝑒𝑞 [𝐴]𝑒𝑞 + [𝐵]𝑒𝑞 [𝐴]0
= = =
𝑘𝑟 𝑘𝑓 𝑘𝑟 + 𝑘𝑓 𝑘𝑟 + 𝑘𝑓

Do đó:
𝑘𝑟 [𝐴]0 𝑘𝑓 [𝐴]0
[𝐴]𝑒𝑞 = ; [𝐵]𝑒𝑞 =
𝑘𝑟 + 𝑘𝑓 𝑘𝑟 + 𝑘𝑓

c) Ta có: [B] = [A]0 – [A]. Thế vào biểu thức ở câu a, ta thu được:
𝑑[𝐴]
− = 𝑘𝑓 [𝐴] − 𝑘𝑟 ([𝐴]0 − [𝐴]) = (𝑘𝑟 + 𝑘𝑓 )[𝐴] + 𝑘𝑟 [𝐴]0
𝑑𝑡
𝑑[𝐴]
⇒ = −𝑑𝑡
(𝑘𝑟 + 𝑘𝑓 )[𝐴] + 𝑘𝑟 [𝐴]0

Lấy tích phân hai vế, ta được:


1
ln[(𝑘𝑟 + 𝑘𝑓 )[𝐴] + 𝑘𝑟 [𝐴]0 ] = −𝑡 + 𝐶
𝑘𝑟 + 𝑘𝑓

Biến đổi và lấy hàm mũ hai vế, ta được:

𝑃𝑒 −(𝑘𝑟 +𝑘𝑓 )𝑡 − 𝑘𝑟 [𝐴]0


[𝐴] =
𝑘𝑟 + 𝑘𝑓

Khi t = 0, [A] = [A]0, do đó:

(𝑘𝑟 + 𝑘𝑓 )[𝐴]0 + 𝑘𝑟 [𝐴]0 = 𝑃

và:
1
𝐶= ln[(2𝑘𝑟 + 𝑘𝑓 )[𝐴]0 ]
𝑘𝑟 + 𝑘𝑓
Từ đó suy ra:

(2𝑘𝑟 + 𝑘𝑓 )𝑒 −(𝑘𝑟 +𝑘𝑓 )𝑡 − 𝑘𝑟 [𝐴]0


[𝐴] =
𝑘𝑟 + 𝑘𝑓

d) Thời gian bán phản ứng trong phản ứng thuận nghịch được tính bằng thời gian từ
đầu cho đến một nửa chặng đường hệ đạt đến trạng thái cân bằng, do đó:
𝑘 [𝐴]
[𝐴]0 + 𝑟 0
[𝐴]0 + [𝐴]𝑒𝑞 𝑘𝑟 + 𝑘𝑓 (2𝑘𝑟 + 𝑘𝑓 )[𝐴]0
[𝐴]1/2 = = =
2 2 2(𝑘𝑟 + 𝑘𝑓 )

Thế vào biểu thức thu được ở câu c, ta có:


−(𝑘𝑟 +𝑘𝑓 )𝑡1
(2𝑘𝑟 + 𝑘𝑓 )[𝐴]0 (2𝑘𝑟 + 𝑘𝑓 )𝑒 2 − 𝑘𝑟 [𝐴]0
=
2(𝑘𝑟 + 𝑘𝑓 ) 𝑘𝑟 + 𝑘𝑓

(4𝑘𝑟 + 𝑘𝑓 )[𝐴]0 −(𝑘𝑟 +𝑘𝑓 )𝑡1


⇒ = (2𝑘𝑟 + 𝑘𝑓 )𝑒 2
2
1 (4𝑘𝑟 + 𝑘𝑓 )
⇒ 𝑡1/2 = − 𝑙𝑛 [ [𝐴]0 ]
𝑘𝑟 + 𝑘𝑓 2(2𝑘𝑟 + 𝑘𝑓 )

Bài 4:
1.
a) A: AgCl ; B: [Ag(NH3)2]+ D: [Ag(CN)2]-
b) 2[Ag(NH3)2]++ [Ni(CN)4]2-+2NH3=2[Ag(CN)2]-+ [Ni(NH3)6]2+
c) Thêm HNO3 và đun sôi để loại các oxit nitơ, còn thêm tiếp HCl (1:9) rồi đun sôi để
kết tủa AgCl sinh ra ở dạng keo sẽ kết tụ lại giúp cho việc lọc ít thất thoát sản phẩm
hơn
d) Do sai số chuẩn độ thấp hơn 0,2% nên nồng độ [Ag(NH3)2]+ vào khoảng
[Ag(NH3)2+] ≤ 0.010×0.2% = 2×10-5mol/L
2[Ag(NH3)2]+ + [Ni(CN)4]2- + 2NH3 = 2[Ag(CN)2]- + [Ni(NH3)6]2+
Ban đầu 0.010 c 0 0
Phản ứng -0.010 -0.0050 +0.010 +0.0050
Sau phản ứng 2×10-5 c-0.0050 2 0.010 0.0050
Theo định luật bảo toàn khối lượng ta có
(1021.1 ) 2 (108.74 ) ( 0.010) 2 ( 0.0050)
K (107.05 )(1031.3 )
 10 5.5  ( 210 5 ) 2 ( c  0.0050)( 2 ) 2
c= 6×10-3mol·L-1

Như vậy nếu tính cả sai số thì lượng Ni(II) có trong dung dịch C sẽ ≥ 6×10-3 M
2.
a) E0(Pt2+/Pt) = 1.19 V > E0(H+, NO3-/NO) = 0.96 V
Do đó Pt không thể tan trong dung dịch HNO3.
Ta có:
𝑃𝑡 2+ 𝑃𝑡 2+ 0.0592
𝐸( ) = 𝐸0 ( )+ 𝑙𝑜𝑔[𝑃𝑡 2+ ]
𝑃𝑡 𝑃𝑡 2
𝑃𝑡 2+ 0
𝑃𝑡 2+ 0.0592 [𝑃𝑡𝐶𝑙42− ]
⇒ 𝐸( )= 𝐸 ( )+ 𝑙𝑜𝑔
𝑃𝑡 𝑃𝑡 2 𝛽[𝐶𝑙 − ]4

Tại điều kiện chuẩn, [PtCl42-] = [Cl-] = 1 M, E(Pt2+/Pt) = E0(PtCl42-/Pt,Cl-). Do đó:

0
𝑃𝑡𝐶𝑙42− 0
𝑃𝑡 2+ 0.0592 1 0.0592
𝐸 ( −
) = 𝐸 ( )+ 𝑙𝑜𝑔 = 1.19 − × 16 = 0.72 𝑉
𝑃𝑡, 𝐶𝑙 𝑃𝑡 2 𝛽 2

Ta thấy E0(PtCl42-/Pt,Cl-) < E0(H+, NO3-/NO). Do đó Pt có thể tan trong nước cường thủy.
b)

Cis-Platin có moment lưỡng cực khác 0, trong khi trans-Platin có moment lưỡng cực bằng 0.
Do đó, ta có thể dựa vào moment lưỡng cực để phân biệt hai phức chất trên.

Bài 5:
1. CaCl2 + Ca = 2CaCl.
2CaCl2 + H2 = 2CaCl + 2HCl
4CaCl2 + C = 4CaCl + CCl4.
2. Tiểu phân có ánh kim màu bạc: Ca
Tinh thể không màu: CaCl2.
3. Do tổng %Ca và %Cl không đủ 100% nên rõ ràng có một nguyên tố X khác
%X = 100 – 52,36 – 46,32 = 1,32%
Dựa theo đề bài thì X chỉ có thể là hydro mà thôi. Như vậy lập tỉ lệ ta được
n(Ca) : n(Cl): n(H) = 1 : 1 : 1
Công thức thực nghiệm: CaClH
Lưu ý rằng phản ứng giữa CaCl2 và hydro không thể nào dẫn đến CaCl mà thay vào đó là
hydrua CaClH. Cấu trúc của hợp chất này đã được xác định phép phân tích phổ tia X nhưng
phương pháp này không phải là một phương pháp tốt để xác định các nguyên tố nhẹ như
hydro. Chính vì sự biến mất của hydro trên phổ tia X mà trong một thời gian dài CaClH bị
tưởng lầm là CaCl.
4. a) Các cấu trúc đồng phân:
H C C CH3
C C C

H H

b) Công thức thực nghiệm: Ca3C3Cl2.


Lưu ý rằng: Nếu tỉ lệ số nguyên tử n(Ca):n(Cl) = 1,5:1 (hay tốt hơn là 3:2 vì có thể viết lại là
CaCl2.2Ca2+ = Ca3Cl44+) và sản phẩm khử phải chứa anion C34- nên phải cần hai cation Ca2+
để trung hoà điện nên chính vì vậy nên công thức Ca3C3Cl2 được chấp nhận.
Đề 3: Hữu cơ

Bài 1:
1. Do phenolphtalein ở những pH khác nhau thì tồn tại dưới các dạng khác nhau nên ở
những vùng pH khác nhau sẽ có màu khác nhau.

Dạng In+ H2In In2− In(OH)3−

Cấu trúc

pH <0 0−8.2 8.2−12.0 >12.0

2. Vòng xiclopropan thường có sức căng vòng, xiclopropanon có lai hóa sp2 làm tăng
sức căng vòng, khi xiclopronanon cộng nước sẽ làm giảm sức căng vòng (so với
xiclopropanon) do lai hóa sp2 chuyển thành sp3 nên quá trong dung dịch
xiclopropan tồn tại dưới dạng hidrat. Ngược lại, đối với 2-hidroxietanal dạng
hemiaxetal kém bền hơn hẳn so với dạng mạch hở do ở dạng mạch vòng xuất hiện
sức căng lớn nên trong dung dịch 2-hidroxietanal không tồn tại dưới dạng
hemiaxetal.

3.

Bài 2:
1. Tổng hợp muscon:
2. Cấu trúc các chất chưa biết:

Bài 3:
1.
2.
a)

b)

c) Phản ứng tạo thành E và F có năng lượng hoạt hóa thấp hơn phản ứng tạo thành B
(chú ý: trong quá trình tạo thành B, sự hình thành chất trung gian D là bước chậm,
nên năng lượng hoạt hóa của bước đó được xem như là năng lượng hoạt hóa của cả
quá trình A → B). Do đó, ở tại điều kiện nhiệt độ thấp mà các phản ứng nghịch tương
đối chậm, tỷ lệ sản phẩm sẽ phản ánh sự cách biệt tương đối về năng lượng hoạt hóa
của hai phản ứng song song (nói cách khác, sự tạo thành E và F sẽ chiếm ưu thế). Ở
nhiệt độ cao hơn, hệ nhanh đạt đến trạng thái cân bằng, dẫn đến sản phẩm nào bền
nhiệt động học (sản phẩm B) sẽ chiếm ưu thế.

Bài 4:
1.

2.

3.

∆G0 = 6×(0.9 kcal/mol) – 4×(0.9 kcal/mol) = 1.8 kcal/mol


[𝑑𝑖𝑎𝑥] 1.8 𝑘𝑐𝑎𝑙/𝑚𝑜𝑙
= (− ) = 0.05
[𝑑𝑖𝑒𝑞] (1.987 × 10 𝑘𝑐𝑎𝑙. 𝑚𝑜𝑙 −1 . 𝐾 −1 ) × 298 𝐾
−3
4. Cấu dạng chính của muối amoni tham gia phản ứng tách loại Hoffmann:

Trong đó liên kết C-H ở nhóm ít nhóm thế đã sẵn anti-periplanar với liên kết C-N,
sẵn sàng tham gia phản ứng E2.

Bài 5:
1. Những cặn cáu trong đường ống hay trên bếp thường là những axit béo mạch dài.
Chúng là những trieste của glyxerin nên khi thủy phân trong kiềm sinh ra glyxerin
và các muối natri của axit béo vốn là những sản phẩm tan được.
Natri hydroxit nguy hiểm với người không chỉ do khả năng thủy phân este mà còn có
khả năng tấn công vào protein của võng mạc khiến nó nhanh chóng bị hủy hoại.
2. Sản phẩm được tạo thành khi chiếu sáng chính là dime của quinon, như vậy X phải là
dihydroquinon, Y là monometylete và Z là dimetylete của hydroxiquinon.
Metyl hóa hoàn toàn albutin rồi thủy phân thu được được2,3,4,6-tetra-O-metyl-D-
glucozơ, tức nhóm OH C1 đã bị thế.
Albutin chỉ bị thủy phân bằng α-glycozidaza tức liên kết là α-glycozit.
Như vậy cấu trúc các chất như sau:

3. Tổng 3Arg + 2 Trp + 1Phe = 1095, chênh 108 đơn vị so với giá trị MX thực tế, ứng với
6 phân tử nước. Hay nói cách khác một hexapeptit tạo được 6 liên kết peptit chứng
tỏ nó có cấu trúc vòng. Dựa trên cấu trúc sản phẩm thủy phân từng phần có thể kết
luận hexapeptit X có cấu trúc: xiclo – (Trp – Arg – Arg – Arg – Trp – Phe).
X không phản ứng với DNFB nên một công thức cấu tạo của X được đề xuất như sau:
Vẫn có thể có một số đáp án khác phù hợp với đề bài. Những trường hợp như
vậy vẫn được chấp nhận.
Đề 4: Đại cương. Vô cơ

Bài 1:
a) Số phối trí: 12
1 1
b) Số nguyên tử trong 1 ô đơn vị = 6x12 + 2x3 + 3 = 6

nguyên tử.
Ta có: a = 2R => OA = 2R, AH = R
Áp dụng Pytago lên tam giác OAH => OH = √3R
2 2
G là trọng tâm tam giác ở đáy => OG = OH = √3R
3 3

2
Áp dụng Pytago lên tam giác OGI => IG = 3 √6R
4
Mà h = 2IG = 3 √6R

Sđáy = 6√3R2
V = Sđáy x h = 24√2R3 = 3√2a3
c) Độ đặc khít ô mạng:
4
𝑉 𝑛𝑔𝑢𝑦ê𝑛 𝑡ử 6.( л𝑅 3 )
Độ đặc = = 3
= 0,74
𝑉 ô 𝑚ạ𝑛𝑔 24√2𝑅3

d) Theo lý thuyết, trong mạng tinh thể hoàn chỉnh:


4
ℎ √6R
𝑎
= 3
2R
= 1,63

Trong mẫu vật thì: 𝑎 = 1,58 < 1,63 => tinh thể không hoàn chỉnh.

1. Năng lượng ion hoá thứ nhất là năng lượng của QUÁ TRÌNH: Nak => Na+k + e
ℎ𝑐 ℎ𝑐
 Để ion hoá 1 nguyên tử Na khí cần photon có bước sóng : λ = = 5,1.1,6.10−19 =
𝐸

243 nm
Nhưng bức xạ có λ= 590nm (năng lượng nhỏ hơn 243nm) lại đủ khả năng bứt
electron ra khỏi bề mặt kim loại).
 Năng lượng cần để tách 1 electron ra khỏi tinh thể kim loại nhỏ hơn nhiều so với
năng lượng ion hoá thông thường (liên hệ bản chất liên kết kim loại – khí
electron)
 Ngưỡng quan điển là bước sóng cần thiết để tách 1 electron ra khỏi bề mặt kim
loại
Bảo toàn năng lượng
Ephoton = Ekim loại + 1/2mv2
6,62.10−34 .3.108 6,62.10−34 .3.108 1
= + 2. 9,1094. 10−31.842
590.10−9 𝜆𝑜

 𝜆𝑜 = 590 nm
2. Theo MO bậc liên kết của He2+ là 0,5; He2 là 0.
Bài 2:
a) Tính nhiệt độ sôi CH3OH lỏng ở 1 atm:
Xét quá trình: CH3OH(l) ↔CH3OH(k)
∆Ho = -200,7 – ( -238,7) = 38 kJ.mol-1
∆So = 239,7 -126,8 = 112,9 J.mol-1.K-1
Chấp nhận ∆Ho và ∆So không thay đổi theo nhiệt độ.
Tại điểm sôi, ∆Go = 0 => T = 63,4oC
b) Sự sai khác là do chấp nhận ∆Ho và ∆So không thay đổi trong khoảng nhiệt độ từ 25oC
đến điểm sôi.
c) Tính biến thiên entropi của quá trình chuyển 1 mol CH3OH từ 25oC đến điểm sôi
(64,51oC).
Biết Cp(CH3OH lỏng) = 81.6 J.mol-1.K-1
CH3OH(l)/25oC

𝑇2 64,51+273
∆S1 = Cpln𝑇1 = 81,6 x ln = 10,16 J.mol-1.K-1
298
𝑄 38000
∆S2 = = = 112,59 J.mol-1.K-1
𝑇 64,51+273

∆S = ∆S1 + ∆S2 = 122,75 J.mol-1.K-1


d) Xét chai khi bão hoà Metanol:
PCH3OH = P (atm)
PO2 = a (atm) PN2 = 4a (atm)
Khi hệ đạt cân bằng, Phệ = 5a + P = Pmôi trường
 5a + P = 1 atm (1)
Xét quá trình sau ở 25oC:
CH3OH(l) ↔CH3OH(k)
Dễ dàng tính được ∆Go298 = 4355,8 J =-RTlnKp
 Kp = 0,1724 atm
 PCH3OH = 0,1724 atm
Từ (1) => a = 0,1655 atm
𝑃𝑉
Số mol O2 trước khi phóng điên: nO2 = 𝑅𝑇= 3,39.10-3 mol

Số mol hơi CH3OH trước khi phóng điện: nCH3OH = 3,58.10-3 mol
Vậy trước khi phóng điện trong chai có: O2 : 3,39.10-3 mol
CH3OH: 3,58.10-3 mol
Khi phóng điện, phản ứng sau xảy ra:
3
CH3OHk + 2O2 => CO2k + 2H2Ohơi hoặc lỏng

Để xác định hơi nước tạo thành ở dạng lỏng hay hơi:
Áp suất hơi nước bão hoà ở 25oC: P* = 4,58 mmHg
Trong phản ứng trên, Oxi phản ứng hết, CH3OH dư.
Số mol H2O tạo thành: nH2O = 4,52.10-3 mol
Nếu lượng nước tạo thành hoàn toàn ở thể khí thì áp suất hơi nước sẽ là: PH2O = 168
mmHg >> P* = 4,58 mmHg nên có thể xem toạn bộ nước tạo thành đều ở dạng lỏng (Thật sự
thì 1 lượng nhỏ nước ở dạng hơi với áp suất 4,58 mmHg, còn lại ở dạng lỏng, nhưng không
đáng kể).
Phản ứng xảy ra sẽ là:
3
CH3OHk + 2O2k => CO2k + 2H2Ol
∆Ho = -285,8x2 -393,5 + 200,7 = -764,4 kJ/mol
3
=> ∆Uo = -764,4 –(1- 2 – 1) x 8,314 x 298x10-3 = -760,68 kJ.mol-1

Nhiệt phản ứng = Q = nCH3OH∆Uo = 2,26.10-3 x (-760,68) = -1,72kJ


Quá trình đẳng tích => W = 0 J
a) Khi áp suất đạt 0,1 kPa, hơi nước dẫn vào sẽ tiến hành tinh thế hoá với CuSO4 để tạo
thành CuSO4.H2O nên số mol hơi nước trong bình không thay đổi => áp suất không
đổi.
b) Số mol CuSO4 = 8,52.10-3 mol
∆m(A-B) = 0,15g => nH2O = 8,33.10-3 mol => x = 1
∆m(C-E) = ∆m(F-G) = 2∆m(A-B) => x=3 và x=5
 Các trạng thái tinh thể bền là CuSO4.H2O, CuSO4.3H2O và CuSO4.5H2O
c) ∆m(C-D) = ∆m(D-E)
 Một nửa lượng CuSO4.H2O bị chuyển hoá thành CuSO4.3H2O
 Hỗn hợp là 1 hỗn hợp đồng mol của CuSO4.H2O và CuSO4.3H2O
 Thành phần theo khối lượng:
178
CuSO4.H2O = 178+214 = 45,4%

CuSO4.3H2O = 54,6%
d) Lượng nước CuSO4 có thể hấp thụ và khối lượng mẫu vật là không thay đổi theo
nhiệt độ.
Khi nhiệt độ tăng, cùng 1 lượng hơi nước, áp suất sẽ tăng. Nên khi tăng nhiệt độ lên,
trục hoành sẽ được kéo dài (áp suất tăng) trong khi trục tung không đổi (khổi
lượng).
Bài 3:
a) Ta có:
d  NO2  d  N 2O5  v
v   k  N 2O5  
dt dt 2
 N 2O5 t vt / 2 1, 20.105 1
  
 N 2O5 o vo / 2 4,80.105 4

Vậy từ 0 đến 1386s phản ứng đã qua hai chu kỳ bán hủy. Tức 2t1/2 = 1386s  t1/2 =
693s  k = ln2/t = 1,00.10-3 s-1.
b) Ta có:
vo / 2 4,80.105
 N 2O5 o    0, 0480M
k 1, 00.103
Po   N 2O5 o RT  1,30atm

c) Phương trình thể hiện P = f(t)


P  Po e  kt  2(1  e  kt )  0,5(1  e  kt )   Po (2,5  1,5e  kt )

Với Po = 1,30 atm và k = 1,00.10-3 s-1 thì phương trình có dạng:


P = 1,30(2,5 – 1,5e-0,001t)
d) Ta có:

v2 Ea  1 1  9, 6.104 Ea  1 1 
ln     ;ln 5
     Ea  215kJ / mol
v1 R  T1 T2  9, 6.10 R  330 340 

1. Tốc độ đầu v = k[HSO3-]2[H+]2 = 9,10-10 mol.L-1.s-1  phản ứng bậc hai.


Ta lại có:

1 d  HSO3  d  HSO3 

2 2 2
v g   2k  H    HSO3   K  HSO3 
2 dt dt
K = 2k[H+]2 = 7,2.10-3 mol.L-1.s-1
Thời gian bán hủy của phản ứng này được xác định dựa trên biểu thức:
1
t1/2   2,8.105 s = 3 ngày
 

K  HSO3 
o

Bài 4:
Trong bài tập này, ta phải chú ý đến việc tổ hợp chính xác các phản ứng có thể xảy ra
hoàn toàn như sau:
1. Zn2+ + 2NH3 + H2Y2- ZnY2- + 2NH4+ K=1018.56
2. H2Y2- + NH3 HY3- +NH4+ K=103.08
Thành phần giới hạn: HY3- 0.01M
NH3 0.07M
NH4+ 0.03M
Chọn: NH3, HY3-
Theo điều kiện proton: h= -[NH4+] + [Y4-] + 0.03
→ h2(1+Ka-1[NH3]) – 0.03h – Ka4[HY3-] =0
Tính lắp 2 lần, ta được h = 2.66 * 10-10
αZn2+ = 1/(1+*β.h-1+∑βi[NH3]i) = 10-4.29
αY4- = K4/(K4+h) = 0.171
Hằng số bền điều kiện β’(ZnY2-) = β* αZn2+* αY4- = 2.747.1011
(ZnY2-)’ (Zn2+)’ + (Y4-)’ β’(ZnY2-)-1
0.01-x x x
Từ đó tính ra được [Zn2+] = [Zn2+]’ * αZn2+
Bài 5:
Phản ứng giữa Mg và axit loãng có sự tạo thành H2.
Nếu chỉ sinh ra H2 (0,54 atm) thì số mol H2 = 0,022 mol.
So với HNO3 = 400 x 1,01 x 0,021 = 8,484g  8,48 mol rất nhiều hơn H2
Vậy 1 g Mg (0,042 mol) có thể tạo ra H2.
Mặt khác khi đun nóng không có phản ứng xảy ra (không có sự thay đổi áp suất) nên
loại trừ khả năng tạo thành NO hay NO2 (do những khí này có thể bị hydro khử tạo thành N2
và nước ở nhiệt độ cao).
Khi thêm oxy = 0,85 – 0,54 = 0,31 atm  0,0126 mol vừa đủ để phản ứng 2H2 + O2 
2H2O xảy ra hoàn toàn nếu trong trường hợp 0,54 atm chỉ thuần H2. Nếu điều đó quả thật
đã xảy ra thì còn thừa lại 0,004 atm O2 trái với đề bài do áp suất còn lại trong bình khi phản
ứng xảy ra là 0,134 atm. Điều đó có nghĩa là phản ứng giữa Mg và HNO3 còn sinh ra một khí
khác nữa không phản ứng với cả H2 và O2 chỉ có thể là N2.
Các phản ứng xảy ra:
Mg + 2H+  Mg2+ + H2
5Mg + 2NO3- + 12H+  5Mg2+ + N2 + 6H2O
Số mol H2 = (2/3)[(0,85 – 0,134)/RT] = 19,53 mmol.
Như vậy số mol Oxy phản ứng là 9,765 mmol tức 0,239 atm.
Số mol N2 = 0,134 – (0,31 – 0,239) / RT = 2,58 mmol
Số mol Mg = 19,53 + 2,58 x 5 = 32,43 mmol
Khối lượng Mg = 0,03243 x 24 = 0,779g  độ tinh khiết của Mg = 77,9%
Đề 4: Hữu cơ

Bài 1:
1. Do có mặt phẳng đối xứng nên góc quay cực phải bằng 0.
2. Dựa trên các dữ kiện của đề bài có thể xác định công thức thực nghiệm của
hydrocacbon có dạng (CH)n. Theo các kết quả phân tích cấu trúc chỉ thấy toàn
cacbon bậc IV nên chỉ có ba hydrocacbon thỏa mãn điều kiện này gồm: tetrahedran
C4H4 , cuban C8H8 và dodecahedran C20H20. Trong đó chỉ cuban và dodecahedran bền
vững ở điều kiện thường còn tetrahedran vẫn chưa điều chế được.

Bài 2:
1.

2.

3. Từ trạng thái chuyển tiếp, ta dự đoán góc giữa hai liên kết C-H kế cận trong tác chất
phản ứng với benzyne cần phải tương đối nhỏ. Trong đó, xiclopentan và xiclooctan
thỏa mãn điều kiện này, còn xiclohexan thì không.
4.
5. Cơ chế (1) có vẻ phù hợp hơn, vì từ kết quả câu c ta thấy sự chuyển 2 nguyên tử
hydrogen từ vòng cyclohexane không hiệu quả.
6. Ta sử dụng chất đầu như sau:

Nếu trong sản phẩm benzenoid có 1 nguyên tử deuterium thì cơ chế đi theo (2), còn
nếu trong sản phẩm benzenoid có 2 nguyên tử deuterium thì cơ chế đi theo (1).

Bài 3:
1.

2. Quy tắc đóng vòng Baldwin:


3.

Bài 4:
1. Cấu trúc các chất:
A: B: C: D:

E: F: G: H:

2. Sơ đồ tổng hợp:
Bài 5:
1.

2.
a) Có nhóm amit CONH2
b) 395 – 398 liên kết peptit (kể cả nhóm CONH2)
c) 367 – 370 đoạn amino axit
Vẫn có thể có một số đáp án khác phù hợp với đề bài. Những trường hợp như
vậy vẫn được chấp nhận.
NĂM 2015
Đề 1: Đại cương. Vô cơ
Bài 1:

1. Có thể xác định nguyên tố đó là S (độ âm điện 2,58) dựa trên quan hệ đường chéo (Các
nguyên tố phân nhóm chính có tính chất tương tự nhau sẽ cùng nằm trên một đường chéo),
chẳng hạn Be và Al.
Thực tế cũng cho thấy hai nguyên tố này tạo ra được một số hợp chất có thành phần tương
tự nhau (SO3 và XeO3, SF4 và XeF4...). Tuy nhiên các hợp chất từ Xe có khả năng phản ứng cao
hơn, và S có thể tạo được hợp chất với C và N còn Xe thì không.
2.

a) Phương trình phản ứng phân rã:


210
84 Po  206
82 Pb + 42 

X là
206
82 Pb

t.ln2 t.ln2 t.ln2


b) T1/2 = = =
 
n  n0   m0 
ln  0  ln   
 n   n 0 -n He  ln  210 
 m0 - VHe 
 210 22,4 

Thay t = 365 ngày ; m0 = 2 g ; VHe = 0,179 l

=> T1/2 = 138 ngày.

c) Tại thời điểm t mẫu chất có chứa Pb và Po. Dựa vào đề bài, ta có được:
m Pb/t
= 0,4
m Pb/t +m Po/t
m Pb/t 206.n Pb/t 2
=> = =
m Po/t 210.n Po/t 3
n Pb/t 70
=> =
n Po/t 103
T1/2  n Pb/t  138  70 
=> t = .ln  1+ = .ln  1+ 
ln2  n Po/t  ln2  103 

=> t = 103 ngày.

3. Mạng tinh thể crystobalit như sau:

a) Lưu ý: Đề bài có đề cập đến việc ở giữa 2 nguyên tử Si là 1 nguyên tử O, đồng nghĩa với việc
nguyên tử O chỉ nằm ở giữa 2 nguyên tử Si “gần nhau nhất” trong ô mạng. Trong hình vẽ thì
2 nguyên tử Si đó chính là nguyên tử ở đỉnh hoặc ở mặt tâm của khối lập phương lớn và
nguyên tử ở hốc tứ diện của khối lập phương nhỏ.
b) Số nguyên tử Si/1ô = 8.1/8 + 6.1/2 + 4 = 8
Số nguyên tử O/1 ô = 16

=> Số phân tử SiO2/1 ô = 8

m tt 8.MSiO 8.MSiO2
Ta có công thức: D = = 3 2 => a = 3
Vtt a NA D.N A

Thay M(SiO2) = 60,1 g/mol ; D = 2,32 g/cm3 ; NA = 6,022.1023 mol-1

=> a ≈ 7.10-8 cm = 7 Å.
4. Do NO là phối tử trung hòa nên trong phức chất này sắt sẽ có số oxy hóa âm (-1) với cấu
hình d9. Do mô hình VSEPR không xét đến phân lớp d nên có thể xem như phức có dạng
AX4E0, tức phức chất có dạng tứ diện.
Bên cạnh đó mỗi phối tử trung hòa NO chỉ có thể dồn vào 2 electron nên tổng số electron
xung quanh nguyên tử sắt trung tâm sẽ là: 8 + 1 + 2x4 = 17 < 18e nên có thể khẳng định
phức chất này sẽ không ổn định.
Phụ chú: Thật ra những khẳng định của đáp án về cơ bản là không chắc chắn và quá lý tưởng khi đã
bỏ qua hiệu ứng Jahn – Teller ứng với cấu hình d9 (chính đây là nguyên nhân dẫn đến việc nhiều
phức d9 của đồng có cấu hình vuông phẳng)

Bên cạnh đó, thuyết VSEPR không áp dụng cho các phức chất các nguyên tố chuyển tiếp. Do
đối với nguyên tố chuyển tiếp, lớp hóa trị còn bao gồm cả phân lớp d và f bên trong. Nhất là đối với
phức chất, sự tham gia giữa các điện tử ghép cặp và độc thân của phân lớp bên trong trở nên rất
quan trọng trong việc xác định cấu trúc phân tử của phức chất.

Đối với phức chất này, cả 4 phối tử NO không giống nhau hoàn toàn. Có 2 phối tử NO là phối
tử trung hòa nitrosyl, còn 2 phối tử còn lại mang điện tích -1 - phối tử nitroxyl. Điều này dẫn đến số
oxi hóa của sắt là +1, cấu hình d7, hoàn toàn phù hợp với cấu trúc tứ diện mà bài báo chính thức đã
đề cập. Tính toán lượng tử cho thấy sắt trong phức chất này mang điện tích ~ +0.8, rất gần với +1.

(Rất cảm ơn bạn Lưu Nguyễn Hồng Quang đã phát hiện ra sai sót trong câu hỏi này)

Bài 2:

1. Lưu ý riêng câu 2.1: Sai cách viết số thập phân được làm tròn trừ 0,25 điểm ứng với 2
lỗi.
a) Ta có phương trình phản ứng:
PbCO3(r) + H2S(k) → PbS(r) + CO2(k) + H2O(k)
Từ đề bài tính được :
P(CO2) = 2,600.10-4 bar
P(H2O) = 4,000.10-3 bar
m H 2S
.R.T 22,4
M H 2S m H 2S .298
RT
P(H 2S) = = . =7,0.10-9 . 273 =5,0.10-9 bar
VH 2S VH 2S M H 2S 34
ΔG°phản ứng = -56,3 kJ/mol
PCO2 .PH 2O
=>ΔGphản ứng = ΔG°phản ứng + RTln = -43,1 kJ/mol < 0
PH 2S

=> Trên thực tế phản ứng trên xảy ra theo chiều thuận.
=> PbCO3 bị hoá đen (PbS có màu đen) dù với một hàm lượng rất nhỏ của H2S trong không
khí, cản trở lớn đến việc dùng chất màu này.
b) Khi dùng H2O2 để khôi phục lại màu trắng thì xảy ra phản ứng sau:
PbS + 4H2O2 → PbSO4 + 4H2O
PbSO4 có màu trắng nên có thể khôi phục màu trắng lại được.
c) Nếu để ở nơi thoáng khí:
PbS + 2O2 → PbSO4
P(O2) = 2,070.10-1 bar
ΔG°phản ứng = -718,9 kJ/mol
1
ΔG phản ứng = ΔG°phản ứng + RTln = -711,1 kJ/mol < 0
PO22

Về mặt nhiệt động học phản ứng hoàn toàn có khả năng tự diễn biến. Thực tế, ở nhiệt độ
thường phản ứng không xảy ra vì năng lượng hoạt hoá của phản ứng lớn.
2.
Quá trình bay hơi diễn ra trong bình chân không: H2O(l) → H2O(h)
Với KP = P(H2Obh) và ΔH°phản ứng = 40,68 kJ/mol
( P(H2Obh) là áp suất hơi bão hoà của nước ).
Ta có phương trình:

 K (373,15K)   P(H 2O bh )(373,15K)  ΔH°  1 1 


ln  P  =ln  =  - 
 K P (298,15K)   P(H 2O bh )(298,15K)  R  298,15 373,15 
=> P(H2Obh)(298,15K) ≈ 0,037 atm.
Để nước trong bình kín tồn tại ở trạng thái khí thì áp suất của hơi nước trong bình bé hơn
hoặc bằng áp suất hơi bão hoà của nước ở 25°C
m H 2O d H 2O .VH 2O
RT RT
M H 2O M H 2O
=> P(H2Obh) ≥ = => a ≥ 51,57 dm = 5,157 m.
a3 a3
3. a) Khả năng tạo phức của Au3+ với các ion halogenua: Br- > Cl- > I-
Lý do:
- Br- có bán kính phù hợp với Au3+ và đồng mức năng lượng nên khả năng tạo phức với Au3+
tốt hơn so với Cl-.
- I- có khả năng tạo phức kém nhất vì I- khử Au3+ xuống thành Au.
b) Ta có các cân bằng sau:
Au(OH)3 + 3H+ + 4Cl-  AuCl4- + 3H2O ΔH1 = -96,1 kJ/mol (1)
Au(OH)3 + 3H+ + 4Br-  AuBr4- + 3H2O ΔH2 = -153,8 kJ/mol (2)
Lấy (1)-(2) => AuBr4- + 4Cl-  AuCl4- + 4Br-ΔH3 = 57,7 kJ/mol (3)
Nhận thấy Qthu vào = 2,13 kJ
2,13
=> C(AuBr4-phản ứng) = = 0,037 M
57,7
=> [AuBr4-] = 0,963 M ; [Cl-] = 3,852 M ; [AuCl4-] = 0,037 M ; [Br-] = 0,148 M

[AuCl 4 - ][Br - ]4 0,037.0,1484


K3 = = = 10-7
[AuBr4 - ][Cl - ]4 0,963.3,852 4
[AuCl 4 - ]
[Au 3+ ][Cl - ]4 β AuCl - 2.1021
=> = 4
= 10 -7
=> β[AuBr4
-
] = = 2.1028
[AuBr4 - ] β AuBr - 10-7
4
[Au 3+ ][Br - ]4

Bài 3:

a) Phản ứng: 2I- + S2O82-  2SO42- + I2


b) Bậc riêng phần là 1 đối với mỗi chất phản ứng
v = k[I-][S2O82-]
k = 0,011 L.mol-1.s-1
c) Tính toán nhiệt độ:

k 25 E A  1 1 
ln    
kT R  TT T25 
k 25 42 000  1 1 
ln      ln 0.1
10  k 25 8.314  TT 298 
1 1 8.314  ln 0.1
   4.558  10 4
TT 298 42 000
1 1
 4.558  10 4   2.9  10 3
TT 298
T  344 .8 K  345 K  72 oC
d) Do phản ứng bậc hai với các chất có nồng độ bằng nhau nên:
1 1
  k t
C C0
1 1
3
  0.011  t
0.1  10 1  10 3
t  818181 s  227 h  230 h

a) Do giai đoạn 2 là giai đoạn chậm nên biểu thức tốc độ phản ứng sẽ có dạng v = k2[A][B].
Cân bằng (1) là nhanh nên ta có [A]2/[A2] = K1 (K1 = k1/k-1)

Như vậy [A] = K11/2 . [A2]1/2


Thay vào biểu thức tốc độ phản ứng ta có v = ktổng[A2]1/2[B]
b) Các đường tương ứng trên đồ thị sẽ như sau.
Trong bốn đường thì có một đường xuất phát từ 0, đó chính là đường ứng với sản phẩm P
vốn chỉ xuất hiện ở một thời điểm nào đó của phản ứng.
Đường giảm tuyến tính thứ hai ứng với chất B, do vận tốc tiêu thụ chất này tuân theo quy
luật động học bậc nhất.
Đường giảm phi tuyến là đường A2, với nồng
độ giảm theo hàm mũ.
Đường cuối cùng ứng với tiểu phân trung
gian A, vốn ban đầu tăng dần, nhưng đến khi
đạt được sự ổn định (đạt được cân bằng
nhanh) thì gần như không đổi.

Bài 4:

1. Tính pH cực đại


KIO3 + 5KI + 6H+  3I2 + 6K+ + 3H2O

IO3- + 6H+ + 5e-  ½I2 + 3H2O Eo = 1,195 V (1)

½I2 + e-  I- Eo = 0.536 V (2)


Phương trình Nernst:
RT c red
E  Eo  ln
zF c oks

E cho bán phản ứng (2):

RT [ I  ] 8,314  298
E2  E o 2  ln  0,536  ln( 0,10)  0,595V
zF 1 1  96485

E cho bán phản ứng (2) là một hàm của pH:

RT 1
E1  E o1  ln
zF [ IO3 ]  [ H  ]6

RT 1 RT 1
 E o1  ln 
 ln  6
zF [ IO3 ] zF [ H ]
RT 1 RT
 E o1  ln 
 2,303  6  pH
zF [ IO3 ] zF
8,314  298 1 8,314  298
 1,195  ln  2,303  6  pH
5  96485 0, 25 5  96485
 1,188  0, 0710 pH

pH đạt cực đại lúc E1 = E2

0,595  1,188  0,0710 pH


1,188  0,595
pH   8,35
0,071

2.
a) Giữa điểm tương đương thứ nhất ta có dung dịch đệm H3PO4 và H2PO4-

H   K H PO 

1
3 4

H PO 2 4

H 3 PO4   7,1.10 3 M
pH  2,15
b) Tại điểm tương đương thứ hai, có HPO42- nên:
[H+] = (K2K3)0,5 = 1,7.10-10M
pH = 9,77
c) HPO42- (K3 = 4,4.10-13) có tính axit không mạnh hơn H2O bao nhiêu (Kw = 1,00.10-14).
Thêm bazơ mạnh vào dung dịch HPO42- tương tự như thêm bazơ mạnh vào nước.
2. Do hằng số tạo phức của Ag(S2O3)23-, Kf = (Kd)-1 = 1,667.1013 là rất lớn nên hầu hết
Ag+ thêm vào sẽ tạo phức với S2O32- và:
[Ag(S2O3)23-] = 0,100M
số mmol S2O32- tự do = 530 – (2.20) = 490mmol.
[S2O32-] = 2,450M
Nồng độ ion Ag+ tự do được tính từ Kd

K 
Ag S O   6,0.10
 2 2
14

Ag (S O ) 
2 3
d 3
2 3 2

Ag   1,0.10
 15

Ag+ + I- → AgI
T = [Ag+][I-] = 8,5.10-17
 [I-] = 8,5.10-2M
Tức có 17,0mmol KI trong dung dịch đầu.

Bài 5:

1. Sai lầm nghiêm trọng nhất hẳn là ở ví dụ 5, sự “sản sinh ra obitan p”. Obitan vốn dĩ là một
khái niệm trừu tượng mô tả xác suất có mặt của electron trong một vùng không gian xác
định. Obitan không thể được sản sinh ra trong một phản ứng hóa học.
2. Hợp chất A ở đây là canxi photphat. Từ đó các phản ứng xảy ra như sau:
2Ca3(PO4)2 + 3SiO2 + 10C = P4 + 10CO + 3Ca2SiO4 (có thể viết CaSiO3 như SGK vẫn có điểm).
B là photpho trắng P4
C là photpho đỏ Pn
Trong nước không có oxy để oxy hóa photpho. Sự phát sáng xảy ra do quá trình oxy hóa một
phần theo phản ứng P + O2 = PO2
Pha loãng dung dịch HCl 2M hai lần thu được dung dịch HCl 1M, tức pH = 0.
P4 + 3NaOH + 3H2O = 3NaH2PO2 + PH3
Phản ứng này sinh ra sản phẩm phụ P2H4 khơi mào sự cháy của PH3
Photphin không có tính base. Tuy nhiên với axit rất mạnh sẽ xảy ra phản ứng trung hòa
khiến pH dung dịch tăng lên:
PH3 + HClO4 = PH4ClO4
Ca3(PO4)2 + 2H2SO4 = Ca(H2PO4)2 + 2CaSO4.

Phản ứng sinh ra supephotphat giúp cây cỏ tốt tươi.


Đề 1: Hữu cơ
Bài 1:

B có ít tương tác hơn nên bền hơn


Ở đây giả sử ∆𝐸 ≈ ∆𝐺 và áp dụng công thức ∆𝐺 = 𝑅𝑇𝑙𝑛𝐾𝑒𝑞
%A + %B = 100%
Giải ra sẽ được %B = 66%

%A = 80%
%B = 99.9%
A<B<C
Trung gian phản ứng ester hóa có lập thể cồng kềnh nên nếu nhóm –OH ở vị trí trục, trung
gian rất khó hình thành => C nhanh nhất
Mặt α có nhóm metyl vị trí trục nên tương tác trục lớn nhất => A chậm nhất

D phản ứng nhanh hơn C

Bài 2:
1. Cơ chế phản ứng

2. Sản phẩm các phản ứng hoán vị:


C2H4 đóng vai trò là chất khơi mào trong phản ứng, vì các diene trong phản ứng tạo ra A và C có cấu
trúc kém linh hoạt từ bộ khung kiều hoàn, C2H4 ban đầu sẽ tham gia phản ứng hoán vị mở vòng kiều
hoàn, sau đó sẽ được tái tạo từ phản ứng hoán vị sau đó.

Bài 3:
Bài 4:

1. Cấu trúc các chất chưa biết:

2. Dựa trên những dữ kiện của đề bài có thể xác định cấu trúc của caryophylen và
isocaryophylen chỉ khác nhau ở dạng hình học cis, trans (hay E, Z) của liên kết đôi trong
vòng xiclononen. Bên cạnh đó nó còn có một nối đôi exocyclic (ngoại vòng), do khi ozon
phân thu được HCHO (tức phải có C = CH2).

Lưu ý: Các dữ kiện của đề bài KHÔNG CHỈ RÕ LẬP THỂ nối đôi trong vòng xiclononen, vì thế
có thể có đáp án ngược lại. Đáp án này vẫn được chấp nhận.

Bài 5:

1. Khi cắt mạch bằng trypsin chỉ có duy nhất một phân đoạn không có đuôi C là Arg hay Lys,
phân đoạn này chắc chắn là đoạn cuối của chuỗi peptit (2).
Cộng thêm kết quả từ chymotrypsin có thể khẳng định trật tự sắp xếp của chuỗi peptit sẽ là:
His – Ser – Gln – Gly – Thr – Phe – Thr – Ser – Asp – Tyr – Ser – Lys – Tyr – Leu – Asp – Ser –
Arg – Arg – Ala – Gln – Asp – Phe – Val – Gln – Trp – Leu – Met – Asn – Thr.
2. Chuỗi tổng hợp Vitamin C
Đề 2: Đại cương. Vô cơ
Bài 1: (3,0 điểm)

1. Cs: [Xe]6s1 còn Au [Xe]4f145d106s1. Electron d và f của Au chắn kém nên Z* của Au lớn hơn
nhiều so với Cs. Điều này dẫn đến I1 của Au lớn hơn rất nhiều.
2. Cấu trúc phân tử

Góc CNS sẽ rất gần với góc tứ diện lý tưởng (109,5o). Do các lưỡng cực S=O đẩy nhau nên góc
OSO sẽ tăng lên và góc NSO sẽ giảm xuống (khoảng 107o). Cả S và N đều lai hóa sp3.

3. Ag(I) là d10 , không có e độc thân nên momen từ là 0


Ag(II) là d9, có 1e độc thân, momen từ 1,73μB
Ag(III) là d8, có 2e độc thân, momen từ 2,83 μB
Do momen từ khác nhau nên việc khảo sát tính chất từ có thể phân biệt được từng khả năng
cụ thể.
4. Từ % oxy tính được khối lượng phân tử M là 102,9 g/mol (M là Rh, HS không cần xác định
chính xác M là kim loại nào).

Với ô mạng lập phương tâm diện thì a 2  4r = 2 x 269,0.10-10 cm


Tính được a = 3,804.10-8cm.
Từ đó tính được khối lượng riêng của kim loại là 12,76 g/cm3
Phụ chú: Câu này sai đề, yêu cầu tính khối lượng riêng kim loại thay vì oxit nên rất xin lỗi các bạn.
Câu này sẽ không tính điểm trong thang điểm chính thức.

Bài 2: (4,0 điểm)

1. Caroten có 22 electron . Vậy HOMO là MO thứ 11 và LUMO là MO thứ 12.


Mức năng lượng được tính theo công thức E = n2h2 / 8mL2, và bước sóng được tính bằng
công thức ∆E = hc /

Từ đó tính được ∆E = 1,38.10-19J và  = 1,44.10-6 m

a) H r  2  H f, 298  182,5 kJ
0 0

G 0r  2  G 0f, 298  175,6 kJ

1
S0r, 298   (H 0r, 298  G 0r, 298 )
T

S0r, 298  24,62 J/K

Phản ứng không xảy ra do G r, 298 dương.


0

b) G 0r  H 0r  T  ΔS0r

G 0r  182500  24.62  T (J)

p 2 ( NO )
c) K 
p(O 2 )  p(N 2 )

(10 3 ) 2
K  6.1 10 6
0.78  0.21

ΔG 0r  RTlnK

ΔG 0r  8.31  T  ln(6.1 10 6 )  99.72  T

182500  24.62  T  99.72  T

T  1468 K

d NOBr 
d) v   k 2  NO  NOBr2 
dt
v1 k NO  Br2 
 1
v 1 k 1 NOBr2 
 NO  NOBr 
v1
v  k2 
v 1

v  k2
k1
NO2  Br2   k'NO2  Br2 
k 1

Bài 3: (4,0 điểm)

1.
a) Thời gian bán hủy của 222Rn = 0,693/3,8 ngày = 0,18 ngày-1 tức 1,3.104ph-1
Do tốc độ phân rã của 222Rn là 4,2 nguyên tử.ph-1.(100L)-1 ta có:
4,2 = kRnCRn  CRn = 3,2.104 nguyên tử.(100L)-1 = 5,5.10-22 mol.L-1
b) Do hoạt độ của 222Rn không thay đổi theo thời gian nên quá trình sinh ra Rn qua sự phân rã
của 226Ra phải cân bằng với quá trình phân rã phóng xạ của chính nó và sự mất mát do quá
trình chưa biết. Dựa trên định luật bảo toàn khối lượng ta rút ra được: 6,7 nguyên tử.ph-
1.(100mL)-1 = (kRn + kchưa biết).CRn
Tính được k = 0,79.10-4 ph-1
c) Do Rn là một khí trơ nên chỉ có thể chịu ảnh hưởng bởi các yếu tố vật lý như nhiệt độ và áp
suất trong hồ. (Thật ra sự mất Rn chính do sự khuếch tán các phân tử Rn trên bề mặt thoát ra
môi trường ngoài).
2.
a) Ta có:
1 d [ NO2 ]
v    k 2 [ N 2O 2 ][ O 2 ]
2 dt
k [ N 2O 2 ] k
K  1   [ N 2O 2 ]  1 [ NO ] 2
k 1 [ NO ] 2 k 1
kk
v  1 2 [ NO ] 2 [ O 2 ]  a  2 b  1 c  0
k 1
b) Ta có:
[ NO ]
 2  [ NO ]  2 [ O 2 ]
[ O2 ]
d [ O2 ]
 k3' [ O 2 ]( 2 [ O 2 ]) 2  4 k3' [ O 2 ] 3
dt
x  3 k  4 k3'

c) Biểu thức liên hệ giữa nồng độ oxy vào thời gian


[ O2 ] t
d [ O2 ] 1 1

[O ]
[ O2 ] 3 
 4 k3' dt
0
  
2 [ O2 ] 2 2 [ O2 ] 02
 4 k3' t ( ( 2 ))
2 0

1 1
2
  2 k3' t
[ O2 ] [ O2 ] 02
Bài 4: (5,0 điểm)

1.

RT  H 

EE  o
ln
F pH 2

a) 0,10 M HCl  [H+] = 0,10 M  E1 = -0,058 V

0,10 M CH3COOH  [H+] = (Ka.C)1/2 = 1,32.10-3 M  E2 = -0,167 V

Epin = 0,109V

b) 0,025 M HCl  [H+] = 0,025 M  E1 = -0,093 V

Dung dịch 2 gồm 0,025 M CH3COOH và 0,0375 M CH3COONa

cacid
  H    K a  1,17.105 M
cbase

E2 = -0,286V

Epin = 0,193V

c) 0,010 M HCl  [H+] = 0,010 M  E1 = -0,116 V

0,10 M CH3COOH  [H+] = (Ka.C)1/2 = 4,18.10-4 M  E2 = -0,196 V

Epin = 0,080V

d) E2 = 0,0252ln[H+] = -0,150 – 0,058 = -0,208 V  [H+] = 2,60.10-4 M

c c
 H    K a acid  2, 60.104  acid  14,8
cbase cbase

Ta có nacid + nbase = 0,010 mol  nbase = 0,010 / 15,8 = 6,30.10-4 mol

Vậy thể tích NaOH cần là 6,30 cm3

2. Gọi c là nồng độ Al2(SO4)3


Al2(SO4)3 + 12H2O → 2[Al(H2O)6]3+ + 3SO42- [SO42-] = 3c (1)
 Al ( H 2O )5 (OH ) 2   H  
[Al(H2O6)]3+ ⇌ [Al(H2O)5OH]2+ + H+ Ka   (2)
 Al ( H 2O )36 

H2O ⇌ H+ + OH- KW = [H+][OH-] (3)


2c = [Al(H2O)6]3+ + [Al(H2O)5OH]2+ (4)
3[Al(H2O)6]3+ + 2[Al(H2O)5OH]2+ + [H+] = 2[SO42-] + [OH-] (5)
Từ (3) ta có [OH-] = KW / [H+] (6)
Từ (4) ta có [Al(H2O)5OH]2+ = 2c - [Al(H2O)6]3+ (7)
Thay (1), (6), (7) vào (5) ta được.
KW
3[Al(H2O)6]3+ + 2(2c - [Al(H2O)6]3+) + [H+] = 6c +
 H  

KW
  H    2c   Al ( H 2O)36   0 (8)
 H  

(2c   Al ( H 2O )36   H  
Thay (4) vào (2): K a 
 Al ( H 2O )36 

Ka[Al(H2O)63+] = 2c[H+] – [Al(H2O)63+][H+]


Ka[Al(H2O)63+] + [H+] = 2c[H+]

2c  H  
 Al  H 2O 6 
 
3
(9)
  H    K
  a

2c  H   KW
Thay (9) vào (8):  H   2c 

 0
 H   K a

 H  

[H+]3 + Ka[H+]2 – 2c[H+]2 – 2cKa[H+] + 2c[H+]2 – KW[H+] – KWKa = 0


[H+]3 + Ka[H+]2 – (2cKa + KW)[H+] – KWKa = 0
[H+]3 + 1,123.10-5[H+]2 – 5,656.10-12[H+] – 1,123.10-19 = 0
[H+] = 5,00.10-7 M , pH = 6,30
[OH-] = 2,00.10-8M
[SO42-] = 7,54.10-7M
[Al(H2O)63+] = 2,15.10-8M
[Al(H2O)5OH2+] = 9,84.10-7M

Bài 5: (4,0 điểm)


A: NOCl B: HCl C: HNO3 D: NO E: Cl2

F: HNO2 G: H2SO4 H: NOHSO4 I: Cu J: Cu(NO3)2

K: CNCl L: (CN)2 M: NO2


Đề 2: Hữu cơ
Bài 1:

1. a) A là hợp chất thơm vì không làm mất màu dung dịch brom và dung dịch KMnO4. A có độ không
no là 9. A thơm, bền nhiệt nên dựa trên yếu tố đối xứng có thể cho rằng A gồm hai vòng benzen nối
với nhau bằng hai cầu ete ở vị trí ortho. Vậy công thức của A là:

b) A có phân tử khối lớn, có nhiều liên kết phân cực nên nó là chất rắn. Dựa trên cấu trúc thì có thể suy
ra A kém tan trong nước, tan tốt trong dung môi hữu cơ.

c) A tương đối bền khi chiếu sáng bởi hệ thơm không có liên kết nào dễ bị đứt khi chiếu sáng. A có
liên kết C sp2 – Cl nên bền vững với kiềm. A cũng bền với axit do cặp e trên oxy đã bị liên hợp với hệ
vòng nên khó phản ứng trong axit.

2. a) Nhân furan rất nhạy với môi trường có H+. Trong môi trường có H+ nhân furan bị polyme hóa
nhanh chóng, dẫn tới không thu được sản phẩm mong muốn

b) Để thực hiện phản ứng trên để thu được sản phẩm mong muốn, cần thực hiện những điều kiện
sau:

- Chuyển nhóm –OH thành một nhóm đi ra tốt

- Tạo một môi trường có anion Br- mà không có H+ để thực hiện phản ứng mở vòng ba mà nhân
furan không bị polymer hóa

Hermione đã đề xuất sử dụng các acid Lewis mang anion Br- như LiBr, MgBr2,… khan và kết quả
thực nghiệm cho thấy sử dụng MgBr2 thay cho HBr sẽ giúp thực hiện phản ứng như mong muốn với
hiệu suất khá (khoảng 30% – 40%).

Bài 2:
2.a)
b)

Bước đầu tiên nếu thí sinh sử dụng môi trường kiềm không sử dụng kèm H2O2 sẽ cho phản ứng
retro-Michael
Khử nhóm carbonyl không sử dụng hoàn nguyên Wolf – Kishner vì môi trường kiềm sẽ ưu tiên
deproton –OH của nhân phenol thay vì deproton trung gian hydrazine.

Bài 3:

Bài 4:
Chất rắn màu trắng sinh ra khi thủy phân trong axit yếu, tan kém trong nước nhiều khả năng
là một amit, tức chất đầu sẽ là nitrin (thủy phân nitrin trong axit và kiềm đều thu được axit có độ
tan tốt). Mặt khác chất rắn màu trắng chỉ xuất hiện sau khi thêm base vào dung dịch đã được axit
hóa trước đó cho thấy chất rắn màu trắng sẽ có nhóm amin.

Như vậy hợp chất chưa biết có thể có một trong ba cấu trúc sau đây

Hợp chất có màu tím đậm do để trong thời gian lâu amin thơm sẽ bị oxy hóa thành quinon.

Bài 5:

1. Cơ chế
2. Dựa trên cấu trúc glucozơ vốn có các nhóm thế nằm ở e nên X buộc phải là epimer của
glucozơ ở C2, C3 và C4. Tức X có thể là mannozơ, allozơ hay galactozơ.
Do thoái phân Wohl rồi khử thu được anditol quang hoạt nên X chỉ có thể là mannozơ hoặc
galactozơ.
Có thể xác định chính xác X bằng cách khử với NaBH4, mannozơ cho sản phẩm quang hoạt
còn galactozơ thì không.

3. Quá trình được biểu diễn như sau:


Đề 3: Đại cương. Vô cơ
Bài 1:

1.
a) Cấu trúc phức A

b) Cấu trúc phức D

Các trị số oxy hóa: A +1 B +1 C +3 D +3 E +3

c) Nguyên tử trung tâm của phức C và E lai hóa dsp3 và đều có 16e nên ta có:

Như vậy hai phức này đều thuận từ

2.
a) 4Fe, 4Cu và 8S. Công thức CuFeS2
b) Lúc này ô mạng cơ sở sẽ là « một nửa » của ô mạng chalcopyrite. Các chấm đen biểu thị 1/2Cu +
1/2Fe và chấm trắng là S. Nhìn vào thấy có 4 nguyên tử S trong ô mạng.

c) Các nguyên tử kim loại lúc này chiếm 50% số lượng hốc tứ diện
d) Cu +1 và Fe +3
Bài 2:

1. Quá trình diễn ra rất nhanh nên hỗn hợp không kịp trao đổi nhiệt với môi trường bên ngoài
=> Quá trình diễn ra là quá trình đoạn nhiệt, tức là Q = 0.
Hệ sinh công => W < 0.
i
Từ 2 điều trên suy ra ΔU < 0, mà ΔU = nRT (i là bậc tự do của phân tử khí)
2
Tức ΔT < 0, tức là nhiệt độ của hỗn hợp giảm, nước đông lại thành tuyết.
2.
a) Ta có cân bằng sau được thiết lập ở 400°C:
2HI  H2 + I2

P°: a - - (atm)

ΔP: 0,2a 0,1a 0,1a (atm)

Pcb: 0,8a 0,1a 0,1a (atm)

PH 2 .PI2 0,1a.0,1a 1
KP = = = = 0,015625
 0,8a 
2 2
P HI 64

b) Ta có các cân bằng xảy ra:

NH 4 I (k) € NH 3(k) +HI (k) K1 (1)


2HI (k) € H 2(k) +I 2(k) K 2 (2)

Lúc đầu áp suất bình tăng nhanh chóng và ổn định được một lúc là do chỉ có cân bằng (1)
được thiết lập, tức là áp suất bình tại thời điểm cân bằng tạm thời bằng tổng áp suất cân bằng tạm
0,9
thời của NH3 và HI => PNH 3 /cbtt = PHI/cbtt = = 0,45 atm
2

Từ đó tính được K1 = PNH3 /cbtt .PHI/cbtt = 0,45.0,45 = 0,2025

Tại thời điểm cân bằng vĩnh viễn thì cà hai cân bằng (1) và (2) được thiết lập.
Ta nhận thấy rằng lượng NH3 và HI tạo thành là như nhau, lượng HI mất đi bằng tổng lượng
H2 và I2 tạo thành.

Vậy có nghĩa là tại thời điểm cân bằng vĩnh viễn: n NH3 = n HI + n H2 + n I2
Hoặc: PNH3 =PHI +PH2 +PI2 (*)

Do ban đầu không có H2 và I2 nên theo cân bằng (2): PH2 = PI2 = x (atm) (**)

Đặt: PHI = y (atm)

Từ (*) và (**) => PNH3 = y + 2x (atm)

Ta có:

x2 1
K2 = 2
= => y = 8x
y 64
K1 = (2x + y)y = 0,2025
=> 80x 2 = 0,2025
=> x  0,05 => y  0,40

PNH 3 = 0,5 atm


Vậy tại thời điểm cân bằng vĩnh viễn: PHI = 0,4 atm
PH 2 = PI2 = 0,05 atm

Thử lại: Từ áp suất NH3 suy ra được lượng NH4I phản ứng, so với số mol ban đầu có thể
khẳng định được phản ứng theo chiều thuận chưa kết thúc trước khi đạt cân bằng.

Bài 3:

k -1[ES]
1. Ta có (1) là cân bằng nhanh nên: k1[E][S] = k -1[ES] => [E] =
k1[S]

 k  k [E] [S]
Và theo đề bài: [E]t = [E] + [ES] = [ES]  -1 + 1 => [ES] = 1 t
 k1[S]  k -1 +k1[S]

(2) là phản ứng chậm nên (2) quyết định tốc độ phản ứng:
d[P] k k [E] [S]
v= = k 2 [ES] = 1 2 t (Điều phải chứng minh)
dt k -1 +k1[S]

2. Ta có:
G = -RTlnK = H - TS với K = kt / kn
 H   S   En  Et 
kt     A   
Tức: K   e RT  .e R 
  t  e RT 

kn  An 

Với H = Et – En = -57,5 kJ/mol

Từ phương trình trên ta có

 S 
  A  A 
e R 
  t   S  R ln  t   41,1 J.K-1.mol-1
 An   An 

Như vậy G = H - TS = -45,3 kJ.mol-1

Bài 4:

1.
a) Ta có biểu thức:

 H 3O    X  
Ka   lg K a  lg  H 3O    lg  X    lg  HX 
 HX 
Chọn một giá trị pH bất kỳ trên đồ thị và thay vào biểu thức trên tính được lgKa = -5 hay Ka =
10-5
b) [HX].Ka = [H3O+][X-] và [X-] + [HX] = 0,1

0,1  H 3O  
Suy ra  HX  
K a   H 3O  

K a  H 3O  
ta được  X  

Tương tự cho [X-]
K a   H 3O  

Và dĩ nhiên ở bất kỳ pH nào [X-] + [HX] = 0,1


c) [HX] sau khi pha loãng = 10-5M
Có cân bằng: HX + H2O ⇌ H3O+ + X- Ka = 10-5
Lúc cân bằng (10-5 – x) x x

Giải được x = 6,2.10-6

Vậy pH = 5,2 và [HX] = 3,8.10-6 M

2.
a) Ksp = 4x3  [Cu2+] = x = 2,0.10-7M
[OH-] = 2x = 4,0.10-7M  [H+] = 2,5.10-8M, pH = 7,60
b) Cu2+ + 2A- = CuA2 K = 8,0.1018
Coi như phản ứng xảy ra hoàn toàn nên [CuA2] = 0,064M
α(A-) = 5.10-4 ở pH = 6.
Đặt [Cu2+] = x thì [A-] = 10-3x và 2[Cu2+] = [A-]/α(A-)
0, 064
K  8, 0.1018  Cu 2   2, 0.105 M ;  A   2, 0.10 8 M
106 x3

CuA 
K1   2, 0.108  CuA   8, 0.105 M
Cu   A 
2 

c) Nồng độ [Cu2+] tăng.


d) Nồng độ [Cu2+] giảm.

Bài 5:

Khi cho dung dịch A tác dụng với Pb(NO3)2 thì tạo thành kết tủa trắng và trong dung dịch chỉ
còn lại muối kali  trong thành phần của hai chất A và B có chứa kali. Kí hiệu anion phức xiano trong
hợp chất A là X, ta có:
1,325
n Pb(NO3 )2 : n A = : 0,1.0,02 = 2:1  Anion phức trong A là X4-
331,21

2Pb2+ + X4-  Pb2X↓

n (mol) 4.10-3 2.10-3

1,2527 = 626,35 (g)  M X 4- = 211,97 (g)


M Pb2X =
2.10-3

2Fe2+ + X4-  Fe2X 

1,5192 1, 6184
n FeSO4  = 0,01 (mol) n Fe2X = 0,005 (mol) M Fe2X = = 323,68 (g/mol)
151,92 0, 005
323,68 . 0,5177
Kết tủa Fe2X chứa 51,77% khối lượng là Fe  n Fe =  3  trong thành phần
55,85
4-
của anion phức xiano X có chứa 1 nguyên tử Fe  X4- có dạng: [Fe(CN) a ], mà M = 211,97 (g)
X 4-

211,97 - 55,85
 a= . Vậy C là Fe2[Fe(CN)6]; A là K4[Fe(CN)6].
12,01 + 14,01

A và B có cùng thành phần nguyên tố, chứa anion phức xiano, trong đó A (là hợp chất của Fe2+)
tác dụng được với dung dịch Fe3+ (Fe2(SO4)3), còn B phản ứng với dung dịch Fe2+ (FeCl2)  B là hợp
chất của Fe3+  B là K3[Fe(CN)6]  D là Fe4[Fe(CN)6]3 và E là Fe3[Fe(CN)6]2.

K4[Fe(CN)6] + 2Pb(NO3)2  Pb2[Fe(CN)6] + 4KNO3


K4[Fe(CN)6] + 2FeSO4  Fe2[Fe(CN)6] + 2K2SO4

3K4[Fe(CN)6] + 2Fe2(SO4)3  Fe4[Fe(CN)6]3 + 6K2SO4

2K3[Fe(CN)6] + 3FeCl2  Fe3[Fe(CN)6]2 + 2KCl

A (kali hexaxianoferat(II)) là hợp chất của Fe2+; B (kali hexaxianoferat(III)) là hợp chất của Fe3+.
Đề 3: Hữu cơ
Bài 1:

1. Trong dung môi phân cực, dạng tồn tại sẽ là dạng có momen lưỡng cực lớn giúp dung môi
solvat hóa tốt, hoặc tạo liên kết hydro liên phân tử với hợp chất
Trong dung môi không phân cực, dạng tồn tại sẽ có momen lưỡng cực nhỏ, tránh lực đẩy
tĩnh điện nội phân tử, hoặc có thể tạo liên kết hydro nội phân tử.
Như vậy:

Ưu thế trong Ưu thế trong


octan e metanol
2. Ở đây tính bazơ giảm do ảnh hưởng hai nhóm t-Bu cồng kềnh làm giảm khả năng solvat hóa.

Bài 2:

1.
2.

Bài 3:
Bài 4:

1.

2.
Theo đề bài thì B chuyển thành dietyl ete khi hydro hóa tức B phải có công thức phân tử
dạng C4HxO.

Có phản ứng: C4HxO + [(10 – x) / 2]H2 = C4H10O

Ta có:

0, 448 0, 04 (3,92  3, 22)


nH 2   0, 02mol  nB   MB 
22, 4 10  x 0, 04
10  x
 64  x  17,5(10  x)  x  6

Vậy B có công thức C4H6O.

B + H2 thu được Et2O nên B phải có dạng ROEt

Do B có 3 nguyên tử H khác nhau theo tỉ lệ 1:2:3 nên B phải là HC≡COCH2CH3


(etoxyaxetylen, etynyletyl ete)


HC≡COC2H5 + H2O 
H
CH3COOH + C2H5OH

Từ đó suy ra C phải tạo ra được 2 phân tử C2H5OH và 1 phân tử CH3COOH khi thủy phân.
Tức trong C phải có 2 nhóm C2H5.

Lúc này C đã có 2 loại nguyên tử H với tỉ lệ 4 : 6 (2 nhóm C2H5). Như vậy C chỉ còn 2 nguyên
tử H khác loại với 2 nhóm C2H5. Như vậy C chỉ có thể là dietyl malonat CH2(COOCH2CH3)2

Khi thủy phân hỗn hợp X trong axit thu được hai sản phẩm là axit axetic và etanol với tỉ lệ 1 :
2. Để tỉ lệ đó được đảm bảo thì A buộc phải có 3 nhóm C2H5, lúc này trong A chỉ còn 3 nguyên tử H
khác loại với 3 nhóm C2H5. Từ đó suy ra A chính là trietylorthoaxetat CH3(OCH2CH3)3

Bài 5:

1.
2.
Đề 4: Đại cương. Vô cơ
Bài 1:
1. Do OH dễ dàng thay thế F (β lớn) nên OH là một ligand mạnh hơn F. Tuy nhiên OH lại khó
thay thế H2O (β nhỏ) nên OH phải là một ligand xếp sau H2O. Từ đó ta kết luận F- < HO- <
H2O.
2.
a) Mâu thuẫn: Các giá trị hằng số chắn S của nguyên tử các nguyên tố He, Be, B, C đều khác
rất nhiều so với quy tắc kinh nghiệm Slater (He: 0,66 thay vì 0,3 ; Be: 2,34 thay vì 2,05 ;
…).
b) Lý do xảy ra mâu thuẫn: Do hiểu nhầm về năng lượng ion hoá của phản ứng
*Cụ thể: Cho quá trình ion hoá sau:
M  M+ + 1e I1
Năng lượng ion hoá thứ nhất được tính như sau: I1 = E M+ - E M
Trong đó: Năng lượng của nguyên tử hoặc ion được tính bằng tổng năng lượng của tất cả
các electron có trong nguyên tử hoặc ion đó
2
N N
 Z* 
Với: E M =  E i =   13,6   i  (eV) (Với N là tổng số electron của nguyên tử M)
i=1 i=1  ni 
Biểu thức trên cũng có thể được áp dụng đối với năng lượng của ion bất kỳ có N
electron.
** Riêng đối với nguyên tử chỉ có 1 electron ở phân lớp ngoài cùng (ns1 hoặc ns2np1):
Quá trình ion hoá xảy ra:
M  M + + 1e I1
(ns1 ) (ns0 )
Hay: (ns2 np1 ) (ns2 np0 )
Cả nguyên tử M và ion M+ đều có cùng CẤU HÌNH ELECTRON ở các phân lớp trong và chỉ
khác nhau ở duy nhất một electron ở phân lớp ngoài cùng. Vậy electron ở các phân lớp
trong TƯƠNG ỨNG của cả M và M+ đều có năng lượng bằng nhau, nên khi tính toán I1 thì
năng lượng của các electron này đã triệt tiêu lẫn nhau, và cuối cùng thì ta có năng lượng
2
 Z* 
ion hoá I1 = -E N = 13,6  N  (eV)
 nN 
Phù hợp với công thức được trích dẫn ở đầu bài. Đó là lý do vì sao hằng số chắn của Li và
B không bị lệch nhiều so với Quy tắc kinh nghiệm Slater.
Ta có thể khẳng định rằng: Với nguyên tử có một electron nằm ở phân lớp ngoài cùng thì
công thức được trích dẫn là đúng, tuy nhiên ta không thể khái quát hoá công thức đó cho
mọi trường hợp ; công thức I1 = E M+ - E M luôn luôn đúng, tuy nhiên nếu chỉ dựa vào
năng lượng ion hoá thứ nhất của nguyên tử có nhiều hơn 1 electron để suy ra hằng số
chắn của nguyên tử thì đó là điều không tưởng, mà ta phải dựa vào năng lượng ion hoá
thứ hai, thứ ba … để tính hằng số chắn của từng ion, rồi từ đó quy về hằng số chắn của
nguyên tử.
c) Nếu bạn là độc giả của tài liệu tham khảo trên: Cần khái quát hoá công thức tính hằng số
chắn S cho mọi trường hợp, và cần phải dựa vào các giá trị năng lượng ion hoá còn lại để
tính toán cho phù hợp.
3. XeOF2: AX3E2 ; dạng bập bênh
TeOF2: AX3E1 ; dạng tháp
BrF4-: AX4E2 ‘ vuông phẳng
PO33-: AX3E1 ; dạng tháp

Bài 2:
1. Theo các thí nghiệm 1,2 và 3 thì khi tăng gấp đôi nồng độ của H2O2 và giữ nguyên nồng độ
của I- thì tốc độ phản ứng tăng gấp đôi, điều đó có nghĩa là phản ứng tỉ lệ thuận với nồng độ
H2O2. Nói một cách khác phản ứng là bậc 1 đối với H2O2.
Từ các thí nghiệm 2, 4, 5 ta nhận thấy tốc độ phản ứng tỉ lệ thuận với [I-]. Điều đó có nghĩa
phản ứng cũng là bậc 1 đối với I-.
2. 2H2O2 → 2H2O + O2
v = kC(H2O2).C(I-)
3. Nồng độ ban đầu:
Khi pha loãng lên 3 lần thì Co = 1%, nghĩa là 10g H2O2/L
10 g / L
Co  = 0,294 mol/L.
34 g / mol

Phản ứng diễn ra chậm tới mức mà trong thời gian ngắn có thể bỏ qua sự giảm nồng độ.
Sau 4 phút sẽ hình thành 4.8,5mL = 34mL O2
Khi đó; n(O2) = 1,390.10-3 mol
Lúc ban đầu có no = 3/34 mol H2O2
sau 4 phút chỉ còn: n4 = no – 2n(O2) = 0,0854 mol
Với C4 = n4/0,3L ta có C4 = 0,285mol.L-1
Nếu tính ra nồng độ sau 1 phút và coi nồng độ đó là Co đối với phút thứ hai v.v… thì sau 4
phút người ta cũng đi đến cùng kết qủa C4.
4. Đối với trường hợp bước thứ nhất quyết định tốc độ thì hằng số của bước thứ nhất k1 nhỏ
hơn hằng số tốc độ của bước thứ hai k2. Tốc độ phản ứng tổng hợp bằng tốc độ phản ứng
của bước thứ nhất và như vậy là v = kC(H2O2).C(I-).
Phản ứng (1) chậm và quyết định tốc độ.
Phản ứng (2) nhanh hơn
(Nếu bước thứ hai quyết định tốc độ thì k2 < k1. Như vậy bắt buộc bước thứ nhất phải là
thuận nghịch, điều đó cũng có nghĩa là k2 < k1. Tốc độ phản ứng tổng hợp lại được quyết định bởi tốc
độ của bước chậm nhất v = kC(H2O2).C(IO-). Nhưng C(IO-) không được biết. Do phản ứng cân bằng
trước nó cho nên C(IO-) có thể biểu thị qua nồng độ của H2O2 và I-
C(IO-) = k1/k-1.C(H2O2).C(I-)
Khác với quy luật đã quan sát được ta có:
v = k1k2 /k-1.C(H2O2)2.C(I-)
5. Vì rằng trong tất cả các thí nghiệm, nồng độ của các chất tham gia bằng nhau cho nên quan
hệ giữa các tốc độ phản ứng giống như quan hệ giữa các hằng số tốc độ tương ứng. Khi đó
v2/v1 = k2/k1. Nếu đặt v1 bằng tốc độ tương đối là 1 thì v2 = k2/k1.
k1 = Ae-73300(8,314.295)
k2 = Ae-56200(8,314.295) v2(tđ) = k2/k1 = 1,07.103.
k3 = Ae-42400(8,314.295) v3(tđ) = k3/k1 = 2,96.105.
k4 = Ae-1750(8,314.295) v4(tđ) = k4/k1 = 4,69.1012.
6. Gọi x là nhiệt độ của 0oC tính bằn Kelvin thì khi đó –x là nhiệt độ của điểm không tuyệt đối
tính bằng oC.
vo = ko.C(H2O2).C(I-) k25 = 8,27k0; k50 = 49,3k0 (1)
v25 = k25.C(H2O2).C(I-)
v50 = k50. C(H2O2).C(I-)
Theo Arrhenius thì k0 = Ae-Ea/RT
hoặc đơn giản hơn là: k0 = Ae-B/T.
Theo (1) ta có: Ae-B/(x+25) = 8,27Ae-B/x
Ae-B/(x+50) = 49,3Ae-B/x
Logarit hóa hai biểu thức trên và giải hệ phương trình ta thu được gía trị x = 272,7 và như
vậy giá trị của điểm không tuyệt đối là –272,7oC.

Bài 3:
1.
a) Anh sinh viên kia quên mất rằng, đối với những chất tan điện ly thì trong các công thức liên
quan đến Định luật Raoult, áp suất thẩm thấu và áp suất hơi bão hoà phải thêm vào công
thức hệ số đẳng trương (hay hệ số Van’t Hoff) i. Trong trường hợp này hệ số đẳng trương i
của đường bằng 1; của NaCl bằng 2.
b) Vậy công thức tính độ tăng nhiệt độ sôi sẽ là:
Ts = k s (C mGlucose + 2C m NaCl )
n Glucose + 2n NaCl
Hay: Ts = k s
m H 2O
VH 2O D H 2O Ts 0,1.0,998.3,58
=> n Glucose + 2n NaCl = = = 0,7 mol
ks 0,51
Mà mhỗn hợp = 180nGlucose+58,5nNaCl = 35,55 (g)
=> nGlucose = 0,1 mol ; nNaCl = 0,3 mol.
2.
a) Ta có các cân bằng sau ở 273°C(546K):
A(r) +B(k) € C(k) + D(k) (1) K P1 = 6
9
C(k) +B(k) € E(k) + D(k) (2) K P2 =
5
22,4
0,55. .546
n o RT 273
Tại thời điểm ban đầu: P = B o
B = = 1,1 atm
V 22, 4
Tại thời điểm cân bằng: PB + PC + PD + PE = 2,9 atm
*Nhận xét: Trong quá trình phản ứng, lượng chất B mất đi bằng với lượng chất D tạo thành.
Hay nói cách khác, tổng lượng chất B và D tại thời điểm cân bằng bằng với lượng ban đầu của chất B.

Vậy ta được: PB + PD = PBo = 1,1 atm

*Mặt khác:

PE PD
K P2 PP P
= C B = E2
K P1 PC PD PC
PB
K P2 6 2 10 2
=> PE = PC2 = P = PC
K P1 9 C 3
5
Từ các phương trình trên ta có được:

10 2
PC + PC = 2,9 - 1,1 = 1,8
3
=> PC = 0,6 atm
=> PE = 1,2 atm

PB P 0,6
*Tính PB và PD dựa vào K1: = C = = 0,1 và PB + PD= = 1,1 atm
PD K P1 6

=> PB = 0,1 atm ; PD = 1,0 atm


**Tính lại để biết rằng phản ứng theo chiều thuận có kết thúc trước khi đạt cân bằng hay
không:
Từ 2 cân bằng (1) và (2) ta nhận thấy: Lượng chất A mất đi bằng tổng lượng chất C tạo
thành, mà tổng lượng chất C này đã chuyển hoá một phần vào E nên có thể kết luận rằng:

(PC + PE )V 1,8.22,4
Δn A = n C + n E = = = 0,9 mol
RT 22,4
.546
273
Lúc này ΔnA < nA nên phản ứng (1) theo chiều thuận chưa kết thúc trước khi đạt trạng thái
cân bằng.
b) Nếu nA = 0,1 mol thì do ΔnA > nA nên phản ứng (1) theo chiều thuận đã kết thúc trước khi đạt
được trạng thái cân bằng. Lúc này áp suất bình tại thời điểm cân bằng sẽ khác đi.
***Lưu ý: Do đề bài không yêu cầu tính lại áp suất bình nếu nA=0,1 mol nên chỉ cần có câu kết luận
trên thì đã được trọn điểm, nếu thí sinh có tính toán lại cũng không được tính điểm.

Bài 4:
1.
Thế của nửa phải pin E = 0,7946V
Thế của nửa trái pin E = 0,1204V
Vậy thế của pin lúc pin hoạt động E = 0,6742V
Như vậy phản ứng xảy ra theo chiều hướng oxy hóa U4+ và khử Fe3+

U4+ + 2Fe3+ + 2H2O ⇌ UO22+ + 2Fe2+ + 4H+


2.
n
a) pH = pK A − log n s = 1.29 − log(1) = 1.29
B
[CHCl2 COO− ]·[H3 O+ ]
b) KA = [CHCl2 COOH]
−1.29 x·x
10 = 0.1−x
2 −1.29 −2.29
x + 10 · x − 10 =0

10−1.29 10−1.29 2
x1.2 = − 2
± √( 2
) + 10−2.29

x1 = 5.04 · 10−2 (x2 = −1.02 · 10−1 )


[H3O+] = [CHCl2COO-] = 5.04·10-2 mol/L
pH = -log(7.92·10-2) = 1.297
[CHCl2COOH] = 0.1 – 5.04·10-2 = 4.96·10-2 mol/L
[OH-] = 10-(14-1.297) = 1.98·10-13 mol/L
c) [H3O+] = 5.04·10-2 – y
[CHCl2COO-] = 5.04·10-2 + 0.1 – y = 1.504·10-1 – y
[CHCl2COOH] = 4.96·10-2 + y
(1.504·10−1 −𝑦)·(5.04·10−2 −𝑦)
10−1.29 = 4.96·10−2 +𝑦

(y1 = 2.302·10-1) y2 = 2.19·10-2


[H3O+]exact = 5.04·10-2 – 2.19·10-2 = 2.85·10-2 mol/L
pH = -log(2.85·10-2) = 1.545
[H3O+]Henderson = 10-1.29 = 5.13·10-2 mol/L
5.13·10−2 ·100
𝑝= 2.85·10−2
= 179.8%
Bài 5:
2NH3 + 2Na  2NaNH2 + H2
2NaNH2 + N2O  NaN3 + NaOH + NH3
NaN3 + HCl  NaCl + HN3
N2H4 + HN3  [N2H5]+[N3]-
12[N2H5]+[N3]-  3N2H4 + 16NH3 + 19N2
N2 + 3H2  2NH3
2NH3 + H2O2  N2H4 + 2H2O
(NH2)2CO + NaOCl + 2NaOH  N2H4 + H2O + NaCl + Na2CO3
Đề 4: Hữu cơ
Bài 1:

1. Xét hai cấu trúc base liên hợp:

Xiclopentadien có cấu trúc không thơm, nhưng base liên hợp của nó có cấu trúc thơm. Trong
khi đó, bản thân pyrole đã có cấu trúc thơm. Base liên hợp của pyrole chỉ khiến cho hê thơm
ổn định hơn. Tức chênh lệch năng lượng khi phản ứng với base của xiclopentadien sẽ lớn
hơn pyrole. Vì thế xiclopentadien có tính axit mạnh hơn
2. Imidazole sẽ có nhiệt độ sôi cao hơn vì Imidazole tồn tại dưới dạng chuỗi polymer nhờ liên
kết hydro liên phân tử. Trong khi đó cấu trúc của pyrazole chỉ cho phép chúng tồn tại liên
kết hydrogen liên phân tử của 2 đến 3 phân tử với nhau

3. Với cấu trúc cồng kềnh án ngữ lập thể của hai nhóm tBu, 2,6-di-tert-butyl pyridin sẽ không
thể phản ứng với BF3, và phản ứng chậm với HCl.
Như vậy, để nhận biết BF3 và HCl bằng 2,6-di-tert-butyl pyridine, ta làm như sau:
- Trích mỗi chất một ít cho vào ống nghiệm làm mẫu thử
- Nhỏ vào mỗi ống nghiệm vài giọt 2,6-ditert-butyl pyridine và lắc đều
Dung dịch nào có hiện tượng tách lớp (tách thành 2 lớp dung dịch) thì chất ban đầu là
BF3
Dung dịch nào tạo thành hệ dung dịch đồng nhất thì chất ban đầu là HCl

Bài 2:

1. Ozon phân terpen C10H16 (∆ = 3) thu được HCHO và một chất C9H14O chứng tỏ trong terpen
phải có hai vòng và một nối đôi C = CH2 ngoài vòng. Từ các dữ kiện oxy hóa sản phẩm thu
được có thể xác định rằng sản phẩm có chứa vòng xiclopropan. Như vậy terpen ban đầu sẽ
là sabinen (1-isopropyl-4-metylenbixiclo[3.1.0]hexan.
Phản ứng thứ nhất, ozon phân và khử hóa ozonit tạo ra một xeton có 9 nguyên tử cacbon (Y)
và fomanđehit:

Khi tác dụng với KMnO4/H+, Y sẽ bị oxi hóa tạo thành axit đicacboxylic:

Axit này khi tác dụng bởi H2 có xúc tác niken, nung nóng sẽ tạo thành hỗn hợp ba diaxit như
hình vẽ

2. Các phản ứng xảy ra:

Bài 3:

a)
b)

Bài 4:

1. Cơ chế phản ứng:


2. Sản phẩm các phản ứng:

Bài 5:

1. Polysaccarit là một glucose polymer phân nhánh. Do sản phẩm chiếm ưu thế là 2,3,4-tri-O-
metyl-D-glucose nên liên kết glycozit chiếm ưu thế nhất là (1  6). Sự tạo thành 2,4-di-O-
metyl-D-glucose cho thấy sự phân nhánh xảy ra trên C3. Dựa trên tỉ lệ sản phẩm cho thấy cứ
20 mắt xích glucose sẽ có một lần phân nhánh. Đầu 2,3,4,6-tetra-O-metyl-D-glucose vốn bắt
nguồn từ đầu không khử của polysaccarit. Như vậy có thể kết luận polysaccarit này các mắt
xích glucose gắn với nhau theo kiểu (1  6) với sự phân nhánh (1  3) cứ 20 mắt xích lại
xảy ra một lần.
2. Peptit (Lys – Ala)3 sẽ tan tốt hơn ở pH = 7 do lúc này nó có khá nhiều đầu NH3+ phân cực.
Peptit (Asn – Ser – His)5 ở pH = 3 có đầu COOH trung hòa, còn nguyên tử N imidazole trong His đã
hoàn toàn bị proton hóa để trở thành NH+ dương điện. Chính vì thế nó sẽ tan tốt hơn.
NĂM 2016
Đề 1: Đại cương. Vô cơ
Bài 1:

1. Có thể xác định được vị trí của nguyên tử Se trong bảng tuần hoàn như sau

Từ đó có thể ước lượng giá trị khối lượng riêng của Se là trung bình cộng của bốn nguyên tố
xung quanh, tức giá trị ước lượng sẽ là 4,29 g/cm3.

Giá trị thực nghiệm đo được là 4,81 g/cm3, như vậy sai số của phép ước lượng trên sẽ là:
(4,81 – 4,29)/4,81 = 0,11 tức 11%

2. Cấu hình electron của các tiểu phân trên theo thuyết MO:
N2: (σ2s)2(σ2s*)2(π2p)4(σ2p)2

O2: (σ2s)2(σ2s*)2(π2p)4(σ2p)2(π2p)4(π2p*)2

N22-: (σ2s)2(σ2s*)2(π2p)4(σ2p)2(π2p)4(π2p*)2

N2-: (σ2s)2(σ2s*)2(π2p)4(σ2p)2(π2p)4(π2p*)1

O2+: (σ2s)2(σ2s*)2(π2p)4(σ2p)2(π2p)4(π2p*)1

Tiểu phân có năng lượng ion hóa nhỏ nhất chính là tiểu phân có e dễ bị mất đi nhất. Từ cấu
hình e theo MO của các chất thì có thể loại N2 do e có mức năng lượng cao nhất nằm ở mức π2p là MO
liên kết. Trong khi 4 tiểu phân còn lại đều có e chót nằm ở mức MO phản liên kết nên sẽ dễ mất
electron hơn nhiều so với N2.

Tiểu phân dễ bị mất electron nhất sẽ là N22- do ion này có điện tích âm lớn, tức lực đẩy giữa
các e ở đây sẽ lớn hơn đáng kể so với N2- . Với O2+ do số e ít hơn số điện tích hạt nhân nên sẽ xuất
hiện lực hút mạnh giữa hạt nhân với e ngoài cùng. Còn lại giữa N2- và O2 thì số proton giữa O2 sẽ lớn
hơn so với N2- nên có thể dự đoán rằng lực hút của hạt nhân lên 2e ở mức π* của O2 sẽ lớn hơn N2-.

Như vậy thứ tự năng lượng ion hóa thứ nhất của các tiểu phân theo thứ tự tăng dần sẽ là:
N22- < N2- < O2 < O2+ < N2
3. Ô mạng nguyên tử của KBr cũng có dạng giống như ô mạng nguyên tử NaCl. Có thể biểu diễn
ô mạng nguyên tử KBr như hình vẽ dưới đây:

Độ dài cạnh a của ô mạng cơ sở được tính như sau:

4.M KBr 4.119


 KBr  a 3 23
 6, 60.108 cm  660 pm
V. NA 2, 75.6, 022.10

Trong ô mạng KBr thì các ion K+ và Br- tiếp xúc nhau dọc theo chiều dài cạnh, như vậy bán
a 660
kính của ion Br- sẽ là: rBr    rK    138  192 pm
2 2

Hai câu 1.1 và 1.2 nếu lập luận hợp lý nhưng khác đáp án vẫn cho đủ điểm

Bài 2:

1.
a) Phản ứng xảy ra theo phương trình:
C4H6(g) + 5,5O2(g) = 4CO2(g) + 3H2O(g)
∆Ho = 4∆Hos(CO2) + 3∆Hos(H2O) - ∆Hos(C4H6) = -2464,6 kJ/mol. Quy về 0,01 mol C4H6 thì
nhiệt lượng tỏa ra sẽ là -24,64 kJ.
b) Tổng số mol khí trước khi nạp but-1-in vào bình: n = PV/RT = 4,090 mol
Số mol oxy lúc này = 4,090/5 = 0,818 mol, số mol N2 = 3,272 mol
Sau khi đốt cháy thì số mol nitơ vẫn giữ không đổi 3,272 mol. Số mol CO2 = 0,01 x 4 = 0,04
mol. Số mol H2O = 0,03 mol.
Số mol O2 = 0,818 – 0,01x5,5 = 0,763 mol.
c) Gọi Tx là nhiệt độ cực đại của bình sau khi quá trình đốt cháy xảy ra hoàn toàn. Do sự đốt
cháy là đoạn nhiệt nên không có sự trao đổi nhiệt với bên ngoài, tức ở đây Q = ∆H = 0. Từ đó
ta có chu trình sau:

Như vậy ta có ∆Ho4 + ∆Ho5 + ∆Ho6 + ∆Ho7 + ∆H(298) = 0. Với các giá trị ∆Ho tương ứng lần lượt là:

∆Ho4 = 0,04 x 46,6(Tx – 298)

∆Ho5 = 0,03 x 41,2(Tx – 298)

∆Ho6 = 0,763 x 32,2(Tx – 298)

∆Ho7 = 3,272 x 27,6(Tx – 298)

∆Ho298 = -24,64 kJ = 24,64.103J

Giải phương trình tìm được Tx = 507K.

2. Áp dụng phương trình Clausius – Clapeyron ta có:

p2 H h  1 1 
ln    
p1 R  T1 T2 
p 20, 25.103  1 1 
ln 2    
1, 000 8,314  244 312 
 p2  9, 07 atm
Bài 3:

1.
[HP2 O37 ][H  ]
K1 
a)
[H 2 P2 O72 ]

[P2 O74 ][H  ]


K2 
[HP2 O37 ]

c  [H2P2O72 ]  [HP2O37 ]  [P2O74 ]

 [H  ] [H  ]2  4
b) c  1    [P2 O7 ]
 K2 K1 K 2 

[P2 O 72 ] K1 K 2
 
c K1 K 2  K1[H  ]  [H  ]2

10 6,12 10 8,95


  6,12 8,95  0,00979
10 10  10 6,12 10 7  (10 7 ) 2

k 7* 0,0010 s 1
k   0,102 s 1
7 0,00979

10 6,12 10 8,95


c)  6   0,000484
10 6,12 10 8,95  10 6,12 10 6  (10 6 ) 2

10 6,12 10 8,95


 8  6,12 8,95  0,0997
10 10  10 6,12 10 8  (10 8 ) 2

k6*   6 k  0,000484  0,102 s 1  4,93 10 5 s 1

k8*   8 k  0,0997  0,102 s 1  1,0 10 2 s 1

2.
a) vo = kt.Co(H2).Co(I2)

Co(H2) = 0,112/2.1,12 = 0,05mol.L-1

Co(I2) = 14,224/254.1,12 = 0,05mol.L-1.


Kt = 9.10-5/0,05.0,05 = 0,036L.mol-1.ph-1.

K = kt/kn = C(HI)2/C(H2).C(I2) = 9

Kn = 0, 036/9 = 0,004L.mol-1.ph-1.

b) vt(HI) = vt – vn = ktC(H2).C(I2) – kC(HI)2

vt(HI) = 2,6.10-5 mol.L-1.ph-1.

Bài 4:

1.
a) Xét dung dịch hỗn hợp gồm hai đơn axit yếu HA (nồng độ C1) và HB (nồng độ C2). Áp dụng
điều kiện proton ta có biểu thức sau:
K C K C K
 H     A    B     HO    HA 1  HB 2  w
 H   H   H 

Từ đó suy ra  H    K HAC1  K HBC2  K w

Bỏ qua [H+] do nước phân ly (do quá bé) thì biểu thức thu được sẽ trở thành:
 H    K HAC1  K HBC2

Từ đó ta có:

mHA mHB
 H    K HA  K HB
M HA .V M HB .V
Áp dụng với hệ hai axit yếu 3,4-DHBA (HA) và 3,5-DHBA (HB) thu được [H+] = 3,6.10-3M, tức
pH = 2,45.
b) Ta có hệ phương trình hai ẩn:
mHA + mHB = 30,80
2 mHA mHB
 H    K HA  K HB
M HA .V M HB .V
Với pH = 2,40 và MHA = MHB thu được m(HA) = 6,31 g và m(HB) = 24,49 g
c) Tính tương tự với hai trị pH 2,38 và 2,42 thu được các kết quả tương ứng sau:
pH 2,38 2,42
m(HA) 2,25 10,01
m(HB) 28,55 20,79
C(HA) / C(HB) = m(HA) / m(HB) 0,08 0,48
Do tỉ lệ nồng độ chênh lệch khá lớn khi pH lệch 0,02 đơn vị (trong giới hạn cho phép của
máy đo) nên khả năng xác định chính xác thành phần dung dịch đầu dựa trên giá trị pH đọc được
trên máy đo hoàn toàn không thực hiện được.

2. Pin có thể được biểu diễn như sau:


Ni | Ni2  ( c Ni2  ) || (c Cu 2  ) Cu 2  | Cu với || ký hiệu cho cầu muối.

Thế chuẩn của pin là Eo  0.599 V .

Thế của pin trước khi thêm CuCl2 được biểu diễn như sau:

R T c Ni2 
E  Eo  ln
2F cCu 2 

Sau khi thêm CuCl2 thì thế mới của pin E' được biểu diễn như sau:

R T c Ni2 
E'  Eo  ln '
2F cCu 2

Với c'Cu 2 là nồng độ mới của Cu2 . Thế tăng một đại lượng, E  E'  E ứng với lượng

CuCl2 thêm vào được biểu diễn dưới dạng:

'
R T cCu 2 
E  E '  E  ln
2F cCu 2 

Như vậy không cần phải biết nồng độ của dung dịch Ni 2 để xác định sự thay đổi thế của pin
sau khi thêm CuCl2 .

Biến đổi biểu thức thức trên thu được biểu thức cho phép xác định nồng độ mới của Cu2 :
2 E F 
c Cu 2  c Cu 2 exp   0.020 M
'

 R T 

Từ đây tính được biến thiên nồng độ sau khi thêm CuCl2 là cCu2  0. 010 M và số mol

CuCl2 thêm vào dung dịch là 1. 00  103 mol .

Vậy khối lượng CuCl2 thêm vào: mCuCl 2  n CuCl 2  MCuCl 2  1.344 g .
Bài 5:

Gọi công thức của oxit lúc này là MxOy. Như vậy ta có:

y %M y 68, 42
M (M )   16   16
x %O x 31,58

Chỉ có x = 2 và y = 3 thì phù hợp (M = 52, Cr). Vậy oxit là Cr2O3 (M = 152)

Các phương trình phản ứng:

2Cr2O3 + CCl4 = 3CO2 + 4CrCl3 (chất rắn màu tím)

CO2 + Ca(OH)2 = CaCO3 + H2O

n(CO2) = 3/2nCr2O3  nCr2O3 = 2/3nCO2  Moxit = 152 g/mol. Oxit là Cr2O3

CaCO3 + 2HCl = CaCl2 + H2O + CO2

2CrCl3 + Cl2 = 2CrCl4 (%Cl = 73,2%)

2CrCl4 = 2CrCl3 + Cl2

Cl2 + 2KI = 2KCl + I2

Khi phản ứng với nước thì CrCl4 bị oxy hóa thành CrCl3, chất này bị thủy phân trong nước tạo môi
trường axit theo phản ứng: CrCl3 + H2O = CrCl2(OH) + HCl.
Đề 1: Hữu cơ
Bài 1:

1. Hợp chất có tính bazơ mạnh nhất là N,N-dimetyl-2,6-dimetylanilin. Ở đây tương tác +C giữa
cặp e của N với vòng benzen nhỏ nhất do tương tác đẩy giữa hai nhóm phenyl với nhóm
NMe2 khiến cho khả năng xen phủ của cặp electron trên N với các obitan π của vòng thơm
trở nên kém hơn hẳn so với ba chất còn lại.
2. Cấu trúc của bixiclo[6.2.0]-2,4,6,8,10-pentaen như hình vẽ:

Hệ thống này phẳng, có 10 electron, nhưng trong cả hai cộng hưởng đều tồn tại vòng
xiclobutadien kém bền. Để vòng được ổn định thì phải chuyển 2e từ vòng này sang vòng kia (thành
2+ và 2-). Việc di chuyển một lượng lớn điện tích để tạo thành hệ thơm là điều hết sức khó khăn,
điều này dẫn đến sự mất ổn định vòng. Tuy nhiên việc tạo thành hệ thơm khi dịch chuyển điện tích
khiến cho tính ổn định của vòng tăng lên. Kết quả giữa hai xu hướng đối nghịch nhau này là
bixiclo[6.2.0]-2,4,6,8,10-pentaen chỉ được bền hóa yếu. Điều này đã được thực nghiệm xác nhận
(Xem thêm D. Cremer, T. Schmidt, and C. W. Bock, J. Org. Chem., 50, 2684 (1985).)

3. Cấu trúc các chất như ở sơ đồ dưới:

Bài 2:
a) Trong BF3, bo lai hóa sp2. Nguyên tử trung tâm còn một AO p trống có khả năng nhận cặp e
từ các base Lewis nên BF3 là một axit Lewis.
b) Bo có cấu hình [He]2s22p1. Khi bo lai hóa sp sẽ hình thành hai obitan lai hóa sp, một obitan
chứa 1 electron xen phủ với 1s của hydro hình thành liên kết B-H. AO lai hóa sp còn lại chứa
một cặp e, và chính nhờ cặp điện tử này mà hợp chất bo đã cho có tính nucleophin. Hai AO p
trống được ổn được ổn định nhờ hai ligand.
c)

d)

Bài 3:

1. Sự tạo thành các sản phẩm P1, P2, P3 qua trung gian M như sau:
2. Cơ chế phản ứng như sau:

Bài 4:

1. Sơ đồ tổng hợp
2. Chuỗi phản ứng:
Br OH
HBr C2H5ONa H CH3
Cl2
C2H5OH H2O Cl

NaOH

HO CH3O
O
CH3ONa H+ HO
CH3O
CH3OH CH3OH

3. B có công thức phân tử C4H8O2, phản ứng với PCC thu được furan. Như vậy B chỉ có thể là cis
- but-2-en-1,4-diol. Furan được hình thành bằng cách đóng vòng but-2-en-1,4-dial (sản
phẩm khi ancol bị PCC oxy hóa) trong môi trường axit (phản ứng Paal – Knoor).
Thủy phân A trong môi trường axit thu được diol B và axeton. B có khả năng làm mất màu
nước brom và không phản ứng với NaOH, như vậy B phải là một xetal có công thức như sau:

Do đề bài chưa thực sự chặt chẽ nên vẫn tồn tại một số đáp án khác phù hợp với giả thiết (hình
dưới). Vì thế nên nếu lập luận hữu lý vẫn được cho nguyên điểm:

Bài 5:

1. Cấu trúc các chất như sau:


2. Aminoaxit đầu N lẫn đầu C đều là Met.

BrCN đóng vai trò cắt mạch peptit sau Met, và biến đoạn Met bị cắt ra thành homoserin
lacton theo cơ chế:

Chymotrypsin cắt mạch sau aminoaxit thơm nên cấu trúc pentapeptit phải là (Met - ? -? - ? –
Phe).

Việc cắt bằng trypsin cho Arg tự do và hai tripeptit có chứa Met chứng tỏ Arg phải là
aminoaxit thứ 4.

Dựa trên dữ liệu cắt mạch bằng pepsin có thể suy ra được peptit phải có trật tự sắp xếp như sau:
Met – Asp – Lys – Arg – Phe – Gly – Met.
Đề 2: Đại cương. Vô cơ
Bài 1:

1. Cấu trúc và dạng lai hóa của nguyên tử trung tâm:

2. Giản đồ MO của NO-

N NO- O

Khi tương tác với H+ thì mức tương tác là obitan bị chiếm có mức năng lượng cao nhất
(HOMO), ở đây chính là mức π* vốn dĩ có năng lượng rất gần với N. Như vậy mật độ e trên N lúc này
sẽ cao hơn so với O nên H+ ưu tiên gắn vào phía N. Do H+ lúc này tương tác với MO π* nên có thể dự
đoán rằng phân tử HNO sẽ có dạng góc (Hình vẽ)

3. Công thức các chất chưa biết:


A: [Fe(NEt3)6][Fe(CO)4]; B: [HOCOFe(CO)4]; C: [(CO)4Fe=C(OLi)CH3]; D: Fe(CO)4Br2; E:
Fe2(CO)9

Bài 2:

1.

2. Đối với phản ứng A → B + nH+ năng lượng Gibbs chuẩn theo hóa sinh liên hệ với năng lượng
chuẩn theo nhiệt động học như sau:
ΔGo’ = ΔGo + nRT lnc(H+) = ΔGo – 2.3nRTpH = ΔGo – 16.1nRT
Trong phản ứng đầu tiên n = 1:
ΔGo’ = -23.6 kJ/mol
Trong phản ứng đồng phân hóa ion H+ không tham gia vào phản ứng, n = 0
ΔGo’ = ΔGo = +1.7 kJ/mol
3. Glucozơ + ATP4- → glucozơ-6-P2- + ADP3- + H+
ΔG = ΔGo + RT[ln c(glucozơ-6-P2-) + ln c(ADP3-) + ln c(H+) – ln c(glucozơ) – ln c(ATP4-)]
ln c(H+) = -8.88
pH = 3.86
4. K = 0.504
Hằng số cân bằng phản ứng đồng phân hóa có thể được biểu diễn theo phần mol ở trạng thái
cân bằng của các chất:
K = [FP]/[GP] = x(FP)/x(GP) = x(FP)/(1-x(FP))
x(FP) = K/(K+1) = 0.335 = 33.5%
Thành phần tương đối của các đồng phần trong hỗn hợp cân bằng không phụ thuộc vào
nồng độ ban đầu của chúng và chỉ được xác định bằng hằng số cân bằng
5. C6H12O6 + 2ADP3- + 2HPO42- + 2H+ → 2CH3CH(OH)COOH + 2ATP4- + 2H2O
Tương tự như nhiệt phản ứng, năng lượng tự do Gibbs có thể tính được qua năng lượng tạo
thành Gibbs của sản phẩm và các chất tham gia phản ứng:
ΔG(thủy phân ATP) = -53.3 kJ/mol
ΔG(đường phân) = -19.4 kJ/mol

Bài 3:

1. Các phản ứng phân rã xảy ra:


40K  40Ca + e

40K + e  40Ar

2. Khả năng xác định hàm lượng dựa trên canxi cho kết quả không đáng tin cậy do canxi cũng
là một thành phần của mẫu đá. Khi tính toán tuổi dựa trên hàm lượng khí nên cân nhắc đến
lượng khí vẫn chưa thoát hết ra khỏi mẫu vật.
3. T = ln2/(k1 + k2) = 1,31.109 năm
N ( K  Ca) T2 1
4.   8,1  N ( K  Ar)  .100  11
N ( K  Ar) T1 1  8,1

5. Hiện tại tổng lượng 40K trong vỏ trái đất là:

5.1025.0, 015.0, 000119


n  2, 2.1018 mol
40
Tổng lượng 40K ở thời điểm ban đầu (cách đây 5 tỉ năm về trước):

vo  v.e( k1 k2 )t  3,1.1019 mol


Như vậy số nguyên tử 40K đã phân rã sẽ là: 3,1.1019 – 2,2.1018 = 2,9.1019 mol 40K. Trong đó bao
gồm 2.9.1019.0,11 = 3,2.1018 mol Ar.
Từ đó tính được V(Ar) = nAr.22,4.10-3 = 7,2.1016 m3.
Lượng Argon trong khí quyển Vatm(Ar) = 0,01.40.109 = 4.1017 m3
Như vậy 1/6 lượng Ar trong khí quyển xuất phát từ 40K.
1000.0, 0324.0, 000119
6. Tổng lượng 40K trong mẫu: n   9,9.105 mol
39
5,9.1015
Hàm lượng argon: n  23
 9,8.10 9 mol
6, 02.10
Như vậy tổng lượng 40K đã phân rã ở thời điểm t = 9,8.10-9 / 0,11 = 8,9.10-8 mol
 ( k1  k2 ) t
Từ đó ta có: n( K )  (n phanra  n)e
40
 (n phanra  n)et ln 2/T

Thay số tính được t = 1,7.106 năm

Bài 4:

1. Viết các phương trình hóa học xảy ra


Al3  3NH3  3H2O ƒ Al(OH)3  3NH4 K1  1018,12

0,01 0,53

0,5 0,03

Cu 2  4NH3 ƒ [Cu(NH3 )4 ]2 β1  1011,75

0,02 0,5

0,42 0,02

Ag   2NH3 ƒ [Ag(NH3 )2 ] β2  107,24

0,1 0,42

0,22 0,1

Pb2  2NH3  2H2O ƒ Pb(OH)2  2NH4 K 2  105,38

0,03 0,22

0,16 0,06

Mg 2  2NH3  2H2O ƒ Mg(OH)2  2NH4 K3  101,38


2. Hiện tượng: Xuất hiện kết tủa keo trắng Al(OH)3, dung dịch chuyển sang xanh
Vì K3  101,38 không lớn nên kiểm tra điều kiện xem có kết tủa Mg(OH)2 hay không

Tính COH từ hệ gồm Al(OH)3 , Pb(OH) 2 , [Cu(NH3 )4 ]2 , [Ag(NH3 )2 ] , Mg 2  0,01M, NH3
'

0,16M, NH 4 0,09M.

C NH3 0,16
pHhệ  pK a  lg  9,24  lg  9, 49
C NH 0,09
4

4,51
 COH  10
'
(M)

4,51.2
 CMg2+ .COH  0, 01.10
' '2
 1011,02  K Mg(OH)2  10 10,9

 Mg(OH)2 không tách ra.

3. Tính nồng độ cân bằng của các ion kim loại trong hỗn hợp B.
K S(Al(OH)3 ) K S(Pb(OH)2 )
[Al3+ ]   3
 1, 35.10 19 (M) [Pb 2+ ]   2
 1, 32.10 6 (M)
[OH ] [OH ]

K S(Mg(OH)2 )
[Mg 2+ ]   1, 32.10 2 (M)
[OH  ]2

[Cu(NH3 ) 4 2+ ]
[Cu 2+ ]  4
 5, 43.1011 (M)
lgβ[ ] 2
Cu(NH3 )4
.[NH3 ]

[Ag(NH3 ) 2 + ]
[Ag + ]  2
 2, 25.107 (M)
lgβ ]+
.
[Ag(NH3 )2
[NH 3
]

4. Trình bày phương pháp hóa học để tách và nhận biết các ion trong dung dịch A.
5. Tính thế của điện cực Cu nhúng trong hỗn hợp B.
ECu = 0,033V

6. Tính SAl(OH)3 trong hỗn hợp B.

pHB  9, 49  [OH ]B  104,51 (M)

Al(OH)3 ƒ Al3  3OH KS  1032,4

C S

Al3  H2O ƒ [Al(OH)]2+  H+ β1*  104,3

C S

 β1*  19  104,6 


S  C Al3+  [Al3+ ]  [Al(OH) 2+ ]  [Al3+ ].  1    1, 35.10 .  1  9,49 
 2, 091.10 14 (M)
 [H + ]   10 

7. Thiết lập pin:


E oCu 2 Cu
 0, 337V < E Cal  0, 242V

(a) Cu | Hỗn hợp B || KCl bão hòa | Hg2Cl2 | Hg (c)

[Cu(NH3 )4 ]2 0, 02M


NH3 0,16M

Anot Cu  4NH3 ƒ [Cu(NH3 )4 ]2  2e

Catot Hg 2Cl2  2e ƒ 2Hg  2Cl

Bài 5:

Do X phản ứng được với Ca nên X phải là một phi kim. Trong dung dịch kiềm X hòa tan sinh ra
một muối tan và một khí. Nguyên tố X có mặt trong cả hai thành phần ấy. Trong hợp chất khí tồn tại liên
kết X – H. Như vậy chỉ có thể có ba khả năng là silan, photphin và amoniac. X sinh ra khi cho than cốc tác
dụng với muối C (có chứa X) và SiO2 nên X chỉ có thể là photpho.

Các phản ứng xảy ra như sau (Có thể viết dạng ion thu gọn)

P4 + 3NaOH + 3H2O  3NaH2PO2 + PH3

P4 + 6Ca  2Ca3P2

2NaH2PO2 + 4CaOCl2  Ca3(PO4)2 + CaCl2 + 2NaCl + 4HCl

2Ca3(PO4)2 + 10C + 6SiO2  6CaSiO3 + 10CO + P4

3Ca3(PO4)2 + 16Al  3Ca3P2 + 8Al2O3

Ca3P2 + 6HCl  3CaCl2 + 2PH3

2Ca3(PO4)2 + 6SiO2  6CaSiO3 + P4O10

P4O10 + 6H2O  4H3PO4 (trong dung dịch axit loãng)

P4O10 + 12NaOH  4Na3PO4 + 6H2O (có thể viết phản ứng tạo muối axit)
Đề 2: Hữu cơ
Bài 1:

1.
a) Chưa tính liên kết hidro: CH2(OH)2 → CH2=O + H2O
ΔH hình thành: -2400 - 1606 -862 (kJ/mol)

ΔH phản ứng: -1606 – 862 – (-2400) = -68 (kJ/mol)

Thêm liên kết hidro: [CH2(OH)2...6H2O]; [CH2=O...2H2O]

ΔH hình thành: - 2400 + 8 x (-30 ) = 2580; -1606 + 2 x (-30 ) = 1666 (kJ/mol).

ΔH phản ứng: -2580 – (-1666) = 914 (kJ/mol).

b) Do fomon có thể trùng hợp tạo thành polymer theo phản ứng sau
n CH2=O → -(-CH2-O-)-n
c) CH2=O không những có tác dụng diệt vi sinh vật mà còn có tác dụng khâu mạch các protein
và các hợp chất hữu cơ khác khiến chúng trở nên bền vững khó bị phân hủy.
a) CH2=O phản ứng khâu mạch với các nhóm OH của tinh bột làm cho nó dai hơn. Khi đó nó
không còn ở trạng thái tự do dễ phát hiện. Vào cơ thể nó có thể được giải phóng ra và phản
ứng bừa vào các nhóm OH, NH ở các hợp chất có trong tế bào.
2.
a. Có 4 C*. Cấu trúc có hai dạng endo và exo.

b. Anhidrit 2,3-dimetyl-7-oxabixiclo[2.2.1]heptan-2,3-dicacboxilic

c. Do có mp đối xứng đi qua nguyên tử oxi và qua giữa liên kết C5-C6 và liên kết C2-C3 nên
phân tử không có tính quang hoạt.

Bài 2:

1. Z là trietylamin Et3N. Vai trò của trietylamin trong phản ứng này là chất đầu độc xúc tác để
làm giảm hoạt tính xúc tác. Nếu không có trietylamin, nhóm hydroxyl của các ancol có cấu
trúc giống với ancol benzylic sẽ có khả năng bị cắt đứt trong quá trình hydro hóa.
Sản phẩm thu được nếu không có trietylamin:
2.
a)

Vị trí α có HOMO mở rộng sẽ ưu tiên những electrophin mềm, ít dương điện; trong khi đó, vị
trí β âm điện hơn sẽ ưu tiên những electrophin cứng, dương điện hơn.
AlCl3 là một axit Lewis mạnh hơn rất nhiều so với BF3. Trong quá trình phản ứng, Ac2O sẽ
hình thành phức hoạt động với axit Lewis. Axit Lewis càng mạnh, tâm electrophin càng dương điện
 Khi dùng AlCl3 sẽ ưu tiên axyl hóa ở vị trí β, còn BF3 sẽ ưu tiên vị trí α.
b)

Khi đun trong TFA, trung gian acylium ion sẽ hình thành. Vì trung gian này rất dương điện
nên sẽ chuyển vị sang vị trí β. Vị trí β cũng trống trải về mặt lập thể hơn (sản phẩm bền nhiệt động).

Bài 3:

1. Cơ chế phản ứng xảy ra:


2. Cơ chế hình thành sản phẩm:
Khi có mặt HCHO sẽ như sau:
Bài 4:

1. Sơ đồ tổng hợp như sau

2. Các phản ứng xảy ra như sau:


3. Dãy chuyển hóa:

Bài 5:

1. Tinh bột và xenlulozơ là những hợp chất cao phân tử, lại có vô vàn liên kết hiđro liên phân tử làm
cho lực hút giữa các phân tử rất lớn, đến nhiệt độ cao vẫn không rời nhau ra (không nóng chảy) mà
bị cháy nếu có oxi hoặc bị phân hủy nếu không có mặt oxi.
2. Cấu trúc các chất chưa biết
CH2OH CHO H O
H OH H OH H OCH3
H
H OH H
H OH
OH OH
CH2OH CH2OH
Q P N

3. Tổng hợp glyxin trong điều kiện Trái đất khởi thủy:
Đề 3: Đại cương. Vô cơ
Bài 1:

1.
a) 5 đồng phân hình học của phức [Cr(H2O)2(NH3)2Br2]+ là:

OH2 OH2 OH2


Br NH3 Br OH2 Br NH3
Cr Cr Cr
H3N Br Br NH3 Br NH3
OH2 NH3 OH2

A:
trans-điamin-trans-điaqua- B: cis-điamin-cis-điaqua- C: cis-điamin-trans-điaqua-
-trans-đibrom com(III) -cis-đibrom crom(III) -cis-đibrom crom(III)

NH3 OH2
Br OH2 H3N OH2
Cr Cr
Br OH2 H3N OH2
NH3 OH2

D: trans-điamin-cis-điaqua- E: cis-điamin-cis-điaqua-
-cis-đibrom crom(III) -trans-đibrom crom(III)
Trong 5 đồng phân hình học trên thì B có hai đồng phân quang học có cấu tạo B1, B2 như
sau:

OH2 OH2
Br OH2 H 2O Br
Cr Cr
Br NH3 H 3N Br
NH3 NH3

B1 B2
b) Giải thích hình dạng bát diện đều của phức:
Cr3+ có cấu hình electron: [Ar]3d34s04p04d0:
Tương tự vì NH3, Br-, H2O đều là các phối tử trường yếu nên cả 3 electron tự do trên 3 obitan
3d của Cr3+ không bị ghép đôi. Khi tham gia tạo phức với các phối tử này có sự lai hóa giữa 2 obitan
3d với 1 obitan 4s và 3 obitan 4p, tạo thành 6 obitan lai hóa trong d2sp3, hướng về 6 đỉnh của hình bát
diện có tâm là Cr.

2.
a) V = a2csinα = 8,98.10-23 cm3
M
  8.00 g/cm3
N A .V
6M H
b)    112 g/dm3
N A .V
So với không khí

m p.M H 2
   81 g/cm3 = 0,081 g/dm3
V R.T
Như vậy rõ ràng LaNi5 là một vật liệu có khả năng hấp phụ hydro rất tốt.

Bài 2:

1)
3H2 N2 2NH3 Σ
t=0 n0 n0 0
t=∞ n0 − 3yn0 n0 − 3yn0 2yn0 2n0 − 2yn0
x∞ n0  3 yn0 1  3y n0  yn0 1 y 2 yn0 2y 1
  
2n0  2 yn0 2  2 y 2n0  2 yn0 2  2 y 2n0  2 yn0 2  2 y

2y
xNH3 
2  2y
2 2
pNH xNH p2 (2  2 y ) 2  4 y 2 1 16  (1  y )  y 2
Kp  3
 3
  
pH3 2 pN2 xH3 2 p 3 xN2 p (1  3 y )3  (1  y ) p 2 (1  3 y )3 p 2
2y 2  0.1111
xNH3   0.1111  y   0.1000
2  2y 2  2  0.1111

16  (1  y)  y 2 16  (1  y)  y 2 16  0.9  0.12
Kp   p    51.2 bar
(1  3 y)3 p 2 (1  3 y)3 K p 0.73  1.60  104 bar 1

2)
a. Đối với phản ứng OH- + H3O+  2H2O
∆H = -117,3/0,00210 = -55,9kJ/mol

Như vậy đối với qúa trình tự phân ly của nước ta có nhiệt phản ứng là +55,9kJ/mol

b. Nhiệt lượng đo được trong thí nghiệm phụ thuộc vào hai qúa trình:
NH3 + H2O → NH4+ + OH- ∆H = x1 (1)

OH- + H3O+  2H2O ∆H = -55,9kJ/mol (2)

Phản ứng của 0,200L . 0,00100M = 2,00.10-3 mol NH3 giải phóng ra -83,4J, phản ứng của một
mol NH3 sẽ giải phóng ra -83,4/(2,00.10-3) = -41,7kJ/mol.

Khi đó ta có: x1 + (-55.9) = -41,7  x1 = 14,2 kJ/mol.

c. Ở đây cần phải xuất phát từ chỗ là đã có một phần NH3 bị phân ly và chỉ có phần còn lại mới
tác dụng theo phương trình (1), trong khi đó lượng OH- tác dụng theo (2) là giống như trên
Tính phần đã tự phân ly:

Theo (1) ta có: K b 


NH   1,77.10
 2
5

NH 
 2

NH   NH  0,00100  NH 


4 4
 
3 4 4

Phương trình bậc hai hình thành sẽ cho kết qủa: [NH4+] = 4,12.10-4M

Đó chính là phần 4,12.10-4/0,00100 = 0,0412 đã tồn tại dưới dạng NH4+, trong khi đó một
phần là 0,9588 vẫn tồn tại ở dưới dạng NH3

Nếu áp dụng những suy luận như ở phần b ta sẽ thu được: x2.0,9588 - 55,9 = -49,7  x2 =
14,8kJ/mol.

Bài 3:
1.
a)
14
7 N  01n  14
6 C  11 H

14
6 C 14
7 N

b) Sự phụ thuộc của hoạt độ A vào thời gian:

a = a0et

a0
ln = t ;
a
ln2
= = 1.245  104 nam 1
t '1/2
230
ln
t= 480  1000 / 3600 = 4380 nam
1.245  104
c) Hoạt độ 230 Bq/kg ứng với tỉ lệ 14C/12C :
m ln 2 m
a = NAk w = NA w
M  12 C  t1/ 2 M  12 C 

(bỏ qua lượng 13C)

at1/ 2 M  12 C  230  5730  365  24  3600  12


w= = = 1.20 1012
N A m ln 2 6.02  10  1000  ln 2
23

Do 6.010–13 / 1.2010–12 = 1/2, mẫu của ta đã trải qua một chu kỳ bán hủy (chúng ta sử dụng
giá trị 5568 năm để xác định tuổi). Nhà khảo cổ học nghĩ rằng chất bột này được tạo thành xấp xỉ cũng
vào thời điểm 3560 năm trước công nguyên.

Tuy nhiên nhóm phenoxiaxetyl được tạo thành từ axit phenoxiaxetic vốn được tổng hợp từ
các quá trình hóa dầu và chuyển hóa than. Nó không hề chứa cacbon phóng xạ. Vì thế chỉ 8 cacbon
trong tổng số 16 là cacbon tự nhiên (tạo ra từ cơ thể sống), như vậy lượng 14C sẽ gấp đôi so với trong
tự nhiên, và rút ra được w = 1.210–12, có nghĩa là chất bột này được tạo ra ở thời hiện đại.
2.
d[C2 H 6 ]
a) = k 2 [CH 4 ] [.CH3] (1)
dt
d[H.]/dt = k1[CH4] + k2[CH4][.CH3] - k3[CH4][H.] - k4[H.][.CH3] = 0 (2)

d[.CH3]/dt = k1[CH4] - k2[CH4][.CH3] + k3[CH4][H.] - k4[H.][.CH3] = 0 (3)

Cộng (2) và (3)  k1[CH4] = k4[H.][.CH3] → [H.] = k1[CH4]/k4[.CH3] (4)

Lấy (2) trừ đi (3), ta có: k2[CH4][.CH3] = k3[CH4][H.] → [.CH3] = k3[H.]/k2 (5)

k1k 3[CH 4 ]
Thay (4) vào (5) và biến đổi đơn giản, ta thu được: [.CH3] = (6)
k 2k 4

3
d[C 2 H 6 ] kk k
Thay (6) vào (1), ta có:  k[CH 4 ] 2 với k = 1 2 3
dt k4

3 1 1 1
[C] 1- - mol - cm3
b. k = n
= [C] 1-n
[t] -1
= [C] 2 [t] -1 = [C] 2 [t] -1 = ( 3
) 2 s = (
-1
) 2  s-1
[t].[C] cm mol

Bài 4:

1.
1
β[Ag(CN)  ? K S(AgI) ? β[Ag(NH + ? β[Ag(NH +
2] 3 )2 ] 3 )]

a) Viết các quá trình xảy ra.


2CN  Ag + ƒ [Ag(CN)2 ] β  1021

I  Ag + ƒ AgI KS1  1016

b) Vai trò của KI và NH3.


Vai trò của KI: chất chỉ thị xác định điểm dừng chuẩn độ.

Vai trò của NH3: tạo môi trường bazơ, tránh phản ứng tạo ra HCN rất độc và hòa tan kết tủa
AgCN (nếu có).

c) Tính CAgNO3 và C KCN biết [CN  ]  3,82.109 (M)


Vì lượng AgI không đáng kể  tại tương đương dừng chuẩn độ.

0, 06
[I  ]  C   10 4.  1, 25.10 7 (M)
I
48, 06

10 16
[Ag + ]  7
 8.10 10 (M)
1, 25.10

 10 9 2 5
 [Ag(CN)2 ]  10 .8.10 .(3,82.10 )  1,16.10 (M) ? [Ag ]
21 +

 C'Ag+  1,16.105 (M)

1,16.105.48, 06
 C AgNO3   2.10 5 (M)
28

1,16.105.2.28
 n CN  2n Ag  CCN   3, 248.10 5 (M)
20

2.
a) Viết phương trình hóa học các phản ứng xảy ra.
[Fe(CN)6 ]3  e  [Fe(CN)6 ]4

C6 H12O6  H2O  C6 H11O7   3H  2e

H+ + OH  H2O

C6 H12O6  2[Fe(CN)6 ]3  3OH  C6 H11O7   2[Fe(CN)6 ]4  2H2O (1)

2[Fe(CN)6 ]3  3I ƒ 2[Fe(CN)6 ]4  I3 K ? (2)

I3  2S2O32 ƒ S4O62  3I  (3)

b) Tính E Fe(OH)3 ; E oFe(CN)3 . Có thể thay K3Fe(CN)6 bằng Fe3+ trong quy trình trên
o
Fe(OH)2 6 Fe(CN)64

được không? Vì sao?


KS(III)
E oFe(OH)3 Fe(OH)2  E oFe3+ Fe 2+  0, 0592 lg  0, 526(V)
KS(II)
β(II)
E oFe(CN)3 Fe(CN)64
 E oFe3+ 2+  0, 0592 lg  0, 3556(V)
6 Fe
β(III)

 E oFe(CN)3 Fe(CN)64
 E oFe(OH)3 Fe(OH)2
6

Nếu dùng Fe3+, trong dung dịch pH = 7,4 sẽ xuất hiện Fe(OH)3  xuất hiện Fe(OH)3 Fe(OH)2

 Fe(OH)3 sẽ không oxi hóa được C6H12O6  không thay K3Fe(CN)6 bằng Fe3+ trong quy
trình trên được.

c) Tính hằng số cân bằng của quá trình:


2[Fe(CN)6 ]3  3I ƒ 2[Fe(CN)6 ]4  I3 K ? (2)

Từ đó cho biết vai trò của Zn2+.

nEo

K  10 0,0592
 106,078

 vai trò của Zn2+

2K+ +Zn 2+ +[Fe(CN)6 ]4 ƒ K 2 [ZnFe(CN)6 ] 

(trắng)

 làm cho (2) chuyển dịch sang phải

d)
1 1 1
n C6 H12O6  (n[Fe(CN) 3  n[Fe(CN) 3 ) (n[Fe(CN) 3  2n I )  (n[Fe(CN) 3  n S O 2 )
6] 6 ] du 6] 6]
2 2 3
2 2 3

1
 n C6 H12O6  (5.4, 012  3, 28.4)  3, 47.106 (mol)
2

3 0, 6246.10 3
 m C6 H12O6  0, 6246.10 (g)  C C6 H12 O6   3,123(g/ l)
0, 2.103

Bài 5:
Đề 3: Hữu cơ
Bài 1:

1. Antraxen có các cộng hưởng sau:

Các liên kết C2 – C7, C2 – C3 và C5 – C6 đều có độ dài 144pm do trong bốn cộng hưởng đưa
ra thì có đến ba cấu trúc các liên kết này đều là liên kết đơn. Liên kết C1 – C2, C3 – C4 có độ dài
140pm do có hai cộng hưởng liên kết đôi và hai cộng hưởng liên kết đơn. Còn liên kết C4 – C5 và C6
– C7 có độ dài 137pm do có ba cộng hưởng nó mang liên kết đôi và một cộng hưởng liên kết đơn.

2. Khi proton hóa nguyên tử nitơ imin sẽ dẫn đến một cation thơm có tính đối xứng cao.

Bài 2:

1. Phản ứng tổng hợp chất X:

Cặp e trên hai nguyên tử N của X có tính base yếu nên chất này không tác dụng với axit, hợp
chất azo cũng không có khả năng cộng electrophin với brom. Khi chiếu sáng hợp chất này phân hủy
tạo thành carben CH3 – C: – CH3 ngay lập tức bị đồng phân hóa thành propen, còn khi có mặt etylen
nó sẽ tấn công vào nối đôi để tạo thành 1,1-dimetylxiclopropan.

2. Sơ đồ phản ứng tổng hợp Y như sau:


Bài 3:

1. Cơ chế tạo thành adenin:

2. Cấu trúc các chất chưa biết và cơ chế chuyển từ C thành D như sau:
Bài 4:
Bài 5:

1. Toàn bộ quá trình phân tích được biểu diễn như hình dưới:

Dựa vào sản phẩm cắt mạch disaccarit bằng HIO4 có thể thấy rằng disaccarit này được tạo
thành từ một đơn vị pyranose nối với một đơn vị furanose qua liên kết α – glycozit giữa hai đầu C1.
Chỉ có ba đường tạo cùng một osazon với D-glucose là D-glucose, D-mannose và D-fructose.
Do disaccarit này chiếm vai trò quan trọng trong cuộc sống cũng như trong công nghiệp nên nó chỉ
có thể là sucrose (saccarozơ) gồm một đơn vị glucopyranose nối với một đơn vị fructofuranose.

Không thể thu được aminoaxit bằng quy trình đó, do ở pH = 4 nhóm NH2 của aminoaxit đã bị proton
hóa khiến cho aminoaxit không thể tan được trong DCM.
Đề 4: Đại cương. Vô cơ
Bài 1:

3. Trong phức trên, coban có số oxy hóa +2 với cấu hình [Ar]3d7
Cấu hình e của phức ở 350K là t2g5eg2 . Phức sẽ có 3e độc thân.
Cấu hình e của phức ở 200K là t2g6eg1 . Phức sẽ có 1e độc thân.
Do ở cả hai phức đều tồn tại bước chuyển từ mức t2g lên eg nên các phức này đều có màu.
Ở 350K, với cấu hình spin cao, hiệu ứng Jahn – Teller ảnh hưởng tới eg1
Ở 200K, với cấu hình spin thấp, hiệu ứng Jahn – Teller ảnh hưởng tới t2g5
eg là mức năng lượng mang bản chất MO phản liên kết trong phức, trong khi t2g mang bản
chất của mức không liên kết. Như vậy khi hiệu ứng Jahn – Teller xảy ra với eg, sự biến dạng độ dài
liên kết sẽ lớn hơn nhiều khi hiệu ứng Jahn – Teller xảy ra ở t2g.
4. Dạng hình học của MeNCS và SiH3NCS:

Trong hợp chất Silic tồn tại liên kết π pπ - dπ giữa AO 3d của Si vào AO p của N. Điều này
khiến cho phân tử silyl isoxianat có cấu trúc phẳng, do đó góc SiNC sẽ lớn hơn góc CNC của metyl
isoxianat có cấu trúc góc.

5. Ta có:
mX 1 ln 2
P .F .k .Q( )  g .5, 41.106.96485 J  144W / g
MX 210 g 138, 4.24.60.60s
1000
mt   6,94 g
144
mt  mo .e  kt  mo  12, 7 g

Bài 2:

1. ∆Go = -RTlnKp . Thay số tính được Kp = 2,37.102


2. Ở thời điểm cân bằng lượng CO2, H2, metanol và nước trong bình lần lượt là x, 3x, (1-x) và
(1-x) mol. Tổng lượng khí trong bình lúc này sẽ là 2 + 2x.
Với khí lý tưởng, khi áp suất bình bằng áp suất tiêu chuẩn (1000hPa = 105Pa) thì phần mol
sẽ chính là áp suất riêng phần. Như vậy ta có:
3
 3   x 
   
2  2x   2  2x  27 x 4
Kp   
 1  x   1  x  (2  2 x) 2 (1  x) 2
  
 2  2x   2  2x 
2
Khai căn hai vế thu được x 2  . Thay số vào tính được x = 0,925 (chỉ lấy giá trị
2  27 / K
dương). Tức ở điều kiện đã cho 92,5% metanol sẽ bị phân hủy thành hydro.
3. Khi thêm 20 mol N2 vào thì tổng số mol lúc này trở thành 22 + 2x. Tính toán tương tự như
câu 2 thu được x = 0,986. Tức 98,6% metanol đã chuyển hóa thành hydro.
4. Để tổng nhiệt lượng bằng 0 thì phản ứng oxy hóa hydro thành nước cần tỏa ra -53 kJ. Như
vậy số mol H2 cần để tỏa ra một lượng nhiệt như vậy là 53/(0,5.485) = 0,22 mol hydro.
Phản ứng chuyển hóa metanol thu được 3 mol hydro, trong đó 0,22 mol hydro đã bị chuyển
hóa thành nước. Tức %H2 chuyển hóa = (0,22/3).100% = 7,3%.

Bài 3:

1. Axit bị tiêu thụ ở giai đoạn 2 và giai đoạn 3. Tức


-d[B]/dt = k2[B][H2O2] + k3[B][HO.]
Nồng độ của gốc tự do hydroxyl trung gian có thể được xác định từ nguyên lý nồng độ ổn
định như sau:
d[OH.]/dt = k1[H2O2][Fe2+] – k3[HO.][B] – k4[[H2O2][OH-] – k5[OH.][Fe2+] = 0

Từ đó rút ra được:

k1  H 2O2   Fe 2 
 HO  
g

k3  B   k4  H 2O2   k5  Fe 2 

Thay biểu thức nồng độ gốc tự do hydroxyl vào biểu thức tiêu thụ B thu được:

d  B k3  B  .k1  H 2O2   Fe 2 
  k2  B  H 2O2  
dt k3  B   k4  H 2O2   k5  Fe 2 
2. Thế của phản ứng oxy hóa Fex+ + Cl2-. = Fey+ + 2Cl- = E – E(Cl2-./Cl-) = 0,771V
Giá trị này cho thấy cả hai cặp Fe2+/Fe và Fe3+/Fe đều không thỏa mãn.
Từ thế của hai cặp trên có thể tính được thế chuẩn của cặp Fe3+/Fe2+ như sau Eo(Fe3+/Fe2+) =
3Eo(Fe3+/Fe) – 2(Fe2+/Fe) = 0,771V.
Như vậy phản ứng oxy hóa xảy ra trong dung dịch sẽ là Fe2+ + Cl2-. = Fe3+ + 2Cl-
nE o

K  10 0,059
 6,96.1025
3. Có thể tính được nhiệt sinh của HCOOH(l) từ phản ứng cháy của nó. HCOOH(l) cháy theo phản
ứng: HCOOH(l) + 0,5O2(k) = CO2(k) + H2O(l). Do nhiệt tạo thành của O2 là 0 nên nhiệt sinh của
HCOOH sẽ là
∆Hos = ∆Hos(CO2) + ∆Hos(H2O) - ∆Hcháy = -424,76 kJ.mol-1
Gọi x là sinh nhiệt của oxy nguyên tử. Từ phản ứng oxy hóa axit hydroxibenzoic ta có:
3∆Hos(HCOOH) - 4∆Hos(CO2) - ∆Hos(B) – 11x = -4994,69.
Giải ra x = 249,17 kJ/mol. Năng lượng liên kết của O – O là 2x = 498,34 kJ.mol-1

Bài 4:

1. Khi phenolphtalein chuyển màu (hồng → không màu) pH = 8


pK a1  pK a2 6,35  10,33
pH HCO    8,34  pH phenolphtalein
3
2 2

2
 chuẩn độ hết nấc 1 của CO3

CO32 + H+ ƒ HCO3

10. Co1 12.0,03

 Co1 = 0,036(M)

Khi metyl da cam chuyển màu → chuẩn độ hết nấc 2

 thành phần của hệ: CO2, H2O.

CO2 + H2O ƒ HCO3  H+ K = 106,35

C 0,03

[] 0,03 – x x x
4
 [H ] = 1,155.10  pH TĐ 2 = 3, 94  4  pH metyl da cam
+

Phản ứng chuẩn độ:

CO32 + 2H+ ƒ H2O + CO2

HCO3 + H+ ƒ H2O + CO2

n HCl  2n CO2  n HCO


3 3

 0,03.(18,67 + 12) = (2Co1 + Co2).10  Co2 = 0,02(M)

2. Dung dịch A gồm có: Cl 0,01M; CrO24 5.10 M. Thêm dần dung dịch AgNO3 vào dung dịch
 3

A. Cho biết hiện tượng.


K S(AgCl) 1010
*Điều kiện xuất hiện kết tủa AgCl: C'Ag+ (1)    108 (M)
C'Cl 0,01

K S(Ag 2CrO4 )
*Điều kiện xuất hiện kết tủa Ag2CrO4: C Ag + (2) 
'

C'CrO2
4

CrO24  H2O ƒ HCrO4  OH K b  107,5

C 5.103

C’ 5.103  x x x

5 3 3
 x  1, 27.10 (M) = 5.10 (M)  CCrO2  5.10 (M)
'
4

5
 CAg + (2)  2.10 (M)
'

 AgCl (trắng) kết tủa trước Ag2CrO4 (đỏ gạch).

 
3. Tính CCrO2 cần có trong dung dịch để khi chuẩn độ Cl bằng Ag thì Ag2CrO4 xuất hiện
4

đúng điểm tương đương.


1
Cl  Ag + ƒ AgCl K S(AgCl)  10
10

CrO24  2Ag + ƒ Ag 2CrO 4 K S(Ag


1
2 CrO4 )
 1011,7
Để Ag2CrO4 xuất hiện đúng điểm tương đương tức là tại điểm tương đương chỉ có quá trình

AgCl ƒ Ag +  Cl K S(AgCl)  1010

[Ag + ]  [Cl  ]  K S(AgCl)  105 (M)

K S(AgCl) K S(Ag 2CrO4 )


Khi Ag2CrO4 xuất hiện [Ag ]TĐ 
+
  105 (M)
[Cl ] 
[CrO 24 ]

2
 [CrO4 ]  10  0, 02(M)
1,7

Bài 5:

Cấu trúc các hợp chất chưa biết:

A: CaCO3 B: CO2 D: CaO E: CaC2


G: CO J: C2H2 K: Ca(OH)2 L: CaCN2
X: NH4OCN Y: ClCN
Các phản ứng xảy ra trong quá trình trên:

CaCO3 = CaO + CO2

CaO + 3C = CaC2 + CO

CaC2 + 2H2O = Ca(OH)2 + C2H2

C2H2 + 2[Cu(NH3)2]+ = Cu2C2 + 2NH3 + 2NH4+

CaC2 + N2 = CaCN2 + C

CaCN2 + 2H2O + CO2 = CaCO3 + (NH2)2CO

NH4OCN = (NH2)2CO

ClCN + NH3 + H2O = (NH2)2CO + HCl


Đề 4: Hữu cơ
Bài 1:

1. Thủy phân nonactin tinh khiết thu được hỗn hợp raxemic của axit nonactic chứng tỏ cấu
trúc của nonactin gồm các đôi đối quang của axit nonactic ghép lại qua cầu este, như vậy
chất này sẽ có cấu trúc như sau:

2. Các phản ứng xảy ra theo sơ đồ sau:

3. Trong dãy ankan độ bền liên kết C – H giảm từ metyl đến tert-butyl do gốc tự do sinh ra
được ổn định bởi hiệu ứng siêu liên hợp khiến cho việc phân cắt liên kết C – H bậc cao bằng
con đường đồng ly được thuận lợi hơn. Tuy nhiên với liên kết C – F độ bền liên kết lại tăng
lên từ metyl đến tert-butyl do liên kết C – F phân cực mạnh, gốc R lúc này đã mang một phần
tính chất carbocation. Như vậy carbocation tương ứng càng bền thì tương tác ion giữa R+ và
F- càng lớn dẫn đến sự tăng năng lượng liên kết C – F từ metyl đến tert-butyl.

Bài 2:

1. Cơ chế sự đồng phân hóa đi qua sự dịch chuyển 1,5-hydrua tạo thành sản phẩm bền hơn:

2. Dựa theo các dữ kiện của đề bài thì cấu trúc các hydrocarbon A, B, C như sau:

3. Dựa theo đề bài thì cấu trúc các chất như sau:

Bài 3:

1. Cấu trúc các hợp chất chưa biết:


2. Cơ chế phản ứng chuyển hóa C và chuyển hóa F
3. Cơ chế phản ứng tạo thành 1

4. Tổng hợp 2

5. NaH đóng vai trò một base lấy H ở OH tạo thành O- tấn công nucleophin vào vòng thơm tạo
thành Oxametoxymarmycin có cấu trúc như sau:
Bài 4:

1. Dựa theo đề bài thì C có thể có mười cấu trúc từ C1 đến C10 như sau:

Do C là chất có pKa thấp nhất nên C3 sẽ là cấu trúc chính xác của C.

F (C4H4O4) sinh ra khi cho dẫn xuất của E tác dụng với etylenglycol. F không còn nhóm OH
nên F chỉ có thể là 1,4-dioxan-2,3-dion. Như vậy E sẽ là axit oxalic.

D sinh ra từ quá trình ozon phân khử B nên D chính là HOCH2 – CHO hoặc (CHO)2. Tuy nhiên
khả năng D là glyoxal không phù hợp với giả thiết conyferin có 2 nhóm OH bậc 1 (1 nhóm của phần
CH2OH trong gốc đường).

Do conyferin là đường không khử nên hợp chất này sẽ có công thức như sau:
2. Cấu trúc các hợp chất chưa biết trong dãy phản ứng:

Hai chất còn lại D và Z được tạo thành theo cơ chế:

Do Y và X (C4H7NO2) có cùng bộ khung carbon nên X chỉ có thể là axit 1-aminoxiclopropan


carboxylic có cấu trúc:

Bài 5:

- Theo nguyên lý nồng độ ổn định:


v = k2[I][A]
d I 
 0  k1[M ][D]  k1[I ]  k 2 [I ][A]
dt
k1[M ][D]
 [I ] 
k1  k2 [A]

k1k2 [M ][D][A]
Tức v 
k1  k2 [A]

- Theo phương pháp cân bằng nhanh:


k1 [I ] k [M ][D]
K   [I ]  1
k1 [M ][D] k1
k1k2 [M][D][A]
Tức v 
k1
Trong trường hợp k-1 >> k2 thì hai biểu thức trên tương đương nhau:
Do các kết quả thực nghiệm cho thấy cơ chế có bậc hai so với andehit nên có thể đề nghị cơ
chế như sau:

You might also like